You are on page 1of 72

수학 하

정답과 풀이
고등
Top of the Top
01 집합 ㄷ. BAAC ( ◯ )
ㄹ. B-A=B이므로 AD(B-A)=ADB= ( ◯ )

STEP 1 1등급 준비하기 p. 6 ~7 따라서 옳은 것은 ㄷ, ㄹ

01 ④ 02 3개 03 8 04 ②
05 ⑤ 06 ② 07 15개 08 ⑤
09 ① 10 ⑤ 05 ^⑤
GUIDE
AB=(ACB)D(ADB)C을 벤 다이어그램 U
01 ^④
으로 나타내면 오른쪽과 같다. 즉
A B
GUIDE
AB=(A-B)C(B-A)
a€+2 a€+2
aGA이면 GA이고 n(A)=1이므로 a=
3 3

ㄱ. AAC=(A-AC)C(AC-A)=ACAC=U ( ◯ )
a€+2
㈏에서 a= , 즉 a€-3a+2=0이 되는 실수 a값은 1 또는 ㄴ. AB=(A-B)C(B-A)
3
2이고, ㈎ 에서 n(A)=1이므로 A={1} 또는 A={2} =(ADBC)C(BDAC)
따라서 옳은 것은 ④ =(BCDA)C(ACDB)
=(ACDB)C(BCDA)
=(AC-BC)C(BC-AC)
02 ^ 3개
=ACBC ( ◯ )
GUIDE
B={z¡€+z™€|z¡GA, z™GA}는 집합 A의 임의의 두 원소를 각각 제곱
ㄷ. A(ADB)={A-(ADB)}C{(ADB)-A}
하여 더한 것을 원소로 가진다. =(A-B)C=A-B
AD(AB)를 벤 다이어그램에서 구해 보면 A-B와 같다.
A={i,-1,-i, 1}이고 zGA인 z에 대하여 z€=1, -1이므로
 A(ADB)=AD(AB) ( ◯ )
B={z¡€+z™€|z¡GA, z™GA}={-2, 0, 2}
따라서 집합 B의 원소는 3개

06 ^②
GUIDE
03 ^8
ADB={5, 6}이므로 {5, 6}AX일 때 ㈐가 성립한다.
GUIDE
따라서 {5, 6}AXAA가 되는 X를 생각한다.
(A-B)CX=X에서 A-BAX, ADX=X에서 XAA이므로 X
는 A-B를 포함하는 A의 부분집합이다.
㈏, ㈐에서 {5, 6}AXAA이다.

A={x|1<x<5}, A
㈎에서 A+X이므로 X가 될 수 있는 집합은 25-2-1=7(개)
X
B={x|3<x<7}에서
A-B
A-B={x|1<x<3}이고
ADX=X,
1 3 q 5 x
07 ^ 15개
GUIDE
(A-B)CX=X에서 A-BAXAA이므로 64
가 자연수이므로 64의 양의 약수 x를 생각한다.
위 그림에서 3<q<5 x

따라서 q의 최솟값 m=3, 최댓값 M=5  m+M=8 1GA이면 64GA, 2GA이면 32GA
4GA이면 16GA, 8GA이면 8GA

04 ^② 원소 개수를 기준으로 집합 A를 다음과 같이 생각한다.


GUIDE 1개 ⇨ {8}
(A-B)C(BC-A)=(ADBC)C(BCDAC) 2개 ⇨ {1, 64}, {2, 32}, {4, 16}
=BCD(ACAC)=BC
3개 ⇨ {1, 8, 64}, {2, 8, 32}, {4, 8, 16}
(A-B)C(BC-A)=BC이므로 U 4개 ⇨ {1, 2, 32, 64}, {2, 4, 16, 32}, {1, 4, 16, 64}
A-{(A-B)C(BC-A)} A B 5개 ⇨ {1, 2, 8, 32, 64}, {2, 4, 8, 16, 32}, {1, 4, 8, 16, 64}
=A-BC=ADB= 6개 ⇨ {1, 2, 4, 16, 32, 64}
즉 전체집합 U의 두 부분집합 A, B 7개 ⇨ {1, 2, 4, 8, 16, 32, 64}
는 오른쪽 벤 다이어그램과 같다. 따라서 모두 15개

02 정답과 풀이
다른 풀이
STEP 2 1등급 굳히기 p. 8~13
64
xGA일 때, GA가 되는 쌍을 찾으면 (1, 64), (2, 32) 01 {4, 8, 16} 02 8 03 5 04 ③
x
05 ③ 06 ⑤ 07 28 08 13
(4, 16), (8, 8)로 모두 4개이므로 공집합을 제외하면 이 4개로
09 ② 10 ③ 11 ① 12 ③
만들 수 있는 집합은 모두 2›-1=15(개)
13 10개 14 ② 15 ⑤ 16 ②
17 ③ 18 24개 19 ④ 20 108
21 ⑴ 672 ⑵ 636 22 ③ 23 ②
08 ^⑤ 24 -20 25 ③ 26 32개
GUIDE
2A={X|XAA}에서 2A는 A의 부분집합을 원소로 가지는 집합이다. 01 ^ {4, 8, 16}
GUIDE
2A의 원소는 , {1}, {2}, {{1, 2}}, {1, 2}, {1, {1, 2}}, A={a, b, c} (단, a<b<c)라 놓고, B를 구한다.
{2, {1, 2}}, {1, 2, {1, 2}}이다.
A의 세 원소를 a<b<c라 하면 표를
① {{1, 2}}는 집합 2A의 원소이다. ( ◯ ) _ a b c
A 이용해 B의 원소를 찾을 수 있다. a a€ ab ac
② {1, 2}가 집합 2 의 원소이므로 {1, 2}만을 원소로 갖는 집합
b b€ bc
 B={a€, ab, ac, b€, bc, c€}
{{1, 2}}는 집합 2A의 부분집합이다. ( ◯ ) c c€
그런데 집합 B의 원소가 5개이고,
⑤ A={1, 2, {1, 2}}G2A이고 {{1, 2, {1, 2}}}A2A이다. 즉
a€<ab<ac<bc<c€이므로 b€=ac이어야 한다.
{A}A2A가 옳다. ( _ )
가장 큰 원소가 256이므로 c=16
참고 가장 작은 원소가 16이므로 a=4
A A
공집합은 모든 집합의 부분집합이므로 G2 이고, A2 이기도 하다. b€=ac=64에서 b=8
따라서 A={4, 8, 16}

09 ^①
GUIDE
n(BDC)=0에서 n(ADBDC)=0임을 이용한다.
02 ^8
GUIDE
집합 X의 원소는 다음 표를 이용해 구할 수 있다.
n(A)=a라 하면 n(ADB)=a-8, n(CDA)=a-13
+ 1 2 3 4 a
또 n(B)=12, n(C)=18, n(BCC)=30이므로
1 2 3 4 5 a+1
n(BDC)=0, 이때 n(ADBDC)=0 3 4 5 6 7 a+3
따라서 35=a+12+18-a+8-a+13에서 a=16 5 6 7 8 9 a+5

X={2, 3, 4, 5, 6, 7, 8, 9, a+1, a+3, a+5}이므로


10 ^⑤ n(X)=10이 되는 자연수 a의 최댓값은 a+1=9에서 a=8
GUIDE 이때 X={2, 3, 4, 5, 6, 7, 8, 9, 11, 13}으로 n(X)=10
A와 C 영화만 본 학생 수가 최소일 때와 최대일 때를 생각한다.
따라서 최댓값은 8
또 B와 C 영화만 본 학생 수가 최소일 때와 최대일 때를 생각한다.

참고
세 영화 A, B, C를 본 학생의 집합을 A, B, C라 하고, C 영화만
➊ a+1이 2부터 9 사이의 값 중 하나와 같으면서 가장 큰 값일 때를 생
본 학생 수의 최솟값과 최댓값을 벤 다이어그램에서 생각해 보자.
각한다.
A A
0 3 ➋ a가 9 이상인 자연수이면 n(X)=11이므로 a<9이다.
3 3 3 0
7 7
0 1 5 1 0 9
B C B C
최솟값을 가지는 경우 최댓값을 가지는 경우
03 ^5
GUIDE
따라서 최솟값은 5, 최댓값은 9이므로 그 곱은 45 {x, y, z}={xy, yz, zx}이므로 세 원소끼리의 합, 세 원소끼리의 곱이
각각 같다. 즉 x+y+z=xy+yz+zx, x_y_z=xy_yz_zx
참고

C 영화만 본 학생이 최소일 때:n(ADC), n(BDC) 값이 최대이다. 주어진 조건에서 xy+yz+zx=x+y+z=6


C 영화만 본 학생이 최대일 때:n(ADC), n(BDC) 값이 최소이다. 또 xyz=(xyz)€에서 xyz=0 또는 xyz=1

1. 집합 03
1 xyz=0이면 t ‹-6t €+6t=0, 즉 t(t €-6t+6)=0의 세 근 A와 B가 모두 무한집합일 때 집합 C는 유한집합일 수도 있고
이 x, y, z이고, 그 근은 0, 3 '3이다. 무한집합일 수도 있다. 즉 ㄷ, ㄹ은 거짓이다.
이때 {x, y, z}+{xy, yz, zx}이므로 불가능하다. ㄱ. f(x)+g(x)=0의 해가 무한하고 f(x)-g(x)=0의 해가
2 xyz=1이면 t ‹-6t €+6t-1=(t-1)(t €-5t+1)=0의 유한한 경우 성립하지 않는다. ( _ )
5-'ß21 ㄴ. A가 유한집합이면 두 집합 B, C도 유한집합이다. ( ◯ )
세 근이 x, y, z이고, 그 근은 1, 이다.
2
ㅁ. B 또는 C가 무한집합이면 집합 A는 무한집합이다. ( ◯ )
따라서 가장 큰 원소 a는 t€-5t+1=0의 근이므로
1
a€-5a+1=0에서 a+ =5
a
07 ^ 28
GUIDE
13=4+9=2€+3€이므로 a£=4, a¢=9이고, a¡=2, a™=3이다.
04 ^③
집합 A의 나머지 원소 a=p로 놓고 ACB를 구한다.
GUIDE
3GA에서 ㈏를 이용해 집합 A를 생각해 보고, ㈐를 이용해 집합 B를 주어진 조건에서 a£+a¢=13=2€+3€
생각해 보자.
즉 ADB={2€, 3€}={4, 9}이므로
① ㈎에 따라 3GA이고, ㈏를 이용하여 집합 A의 원소를 생각 A={2, 3, 4, 9, p}, B={4, 9, 16, 81, p€}이라 하면
해 보면 A={3, 6, 9, 12, y}와 같이 3의 양의 배수 전체의 ACB={2, 3, 4, 9, p, 16, 81, p€}이므로
집합이다. 따라서 A는 무한집합이다. ( _ ) p€+p+115=225에서 p€+p-110=0
② 같은 방법으로 ㈎에 따라 3GB이고, ㈐에서 집합 B의 원소를  p=10 ( p는 자연수)
생각해 보면 B={y,-9,-6,-3, 0, 3, 6, 9, y}, 즉 3의 배 따라서 A={2, 3, 4, 9, 10}이고, 모든 원소의 합은 28
수 전체의 집합이므로 B는 무한집합이다. ( _ )
③ AAB이므로 ADB=A, 무한집합이다. ( ◯ )
④ A-B=이므로 A-B는 유한집합이다. ( _ )
08 ^ 13
GUIDE
⑤ ACB=B이므로 ACB는 3의 배수의 집합이다. ( _ ) n(AB)=n(A)+n(B)-2n(ADB)임을 이용한다.

n(AB)=n(A)+n(B)-2n(ADB)=20 yy ㉠
05 ^③ n(BC)=n(B)+n(C)-2n(BDC)=24 yy ㉡
GUIDE
n(CA)=n(C)+n(A)-2n(CDA)=30 yy ㉢
두 집합 B, C를 다음과 같이 생각할 수 있다.
n(A)+n(B)+n(C)-n(ADB)-n(BDC)-n(CDA)
B + a b c C _ a b c
a a+b a+c a ab ac 를 p라 하면 ㉠+㉡+㉢에서 2p=74  p=37
b b+a b+c b ba bc n(ACBCC)=p+n(ADBDC)=50이므로
c c+a c+b c ca cb
n(ADBDC)=50-p=50-37=13

a<b<c라 하면 B={a+b, a+c, b+c}={5, 7, 8} 다른 풀이


마찬가지로 C={ab, ac, bc}={6, 10, 15}
오른쪽 그림에서 A
집합 B에서 세 원소의 합을 생각하면 a+b+c=10
a+b+f+g=20 …… ㉠ a
a=2, b=3, c=5
b+c+e+g=24 …… ㉡ e g
집합 C에서 세 원소의 곱을 생각하면 abc=2_3_5 d
a+c+e+f=30 …… ㉢ b f
c
즉 A={2, 3, 5}이므로 보기 중 옳은 것은 ㄱ, ㅁ B C
(㉠+㉡+㉢)/2에서
참고 a+b+c+e+f+g=37이므로 d=50-37=13

a+b<a+c<b+c에서 a+b=5, a+c=7, b+c=8이다.


마찬가지로 ab=6, ac=10, bc=15이다.
09 ^②
GUIDE

06 ^⑤ ➊ 결합법칙과 드 모르간의 법칙 등을 생각한다.


➋ AB=(A-B)C(B-A)=(B-A)C(A-B)=BA
GUIDE
A에서 { f(x)+g(x)}{ f(x)-g(x)}=0 ⇨ A=BCC
ㄱ. A=(A-)C(-A)=AC=A ( ◯ )

04 정답과 풀이
ㄴ. AU=(A-U)C(U-A)=CAC=AC ( _ ) 
1
+
1
+y+
1
C C C f(1) f(2) f(5)
ㄷ. AB =(A-B )C(B -A)
=(ADB)C(BCDAC) ={1-;2!;}+{;2!;-;3!;}+y+{;5!;-;6!;}
C C
=(A DB )C(BDA)
=1-;6!;=;6%;
=(AC-B)C(B-AC)=ACB ( ◯ )
ㄹ. 벤 다이어그램으로 나타내 보면 다음과 같다.
(A-B)C (AC)-(BC)
A A
12 ^③
GUIDE
➊ n(XDY)의 범위는
n(X)+n(Y)-n(U)<n(XDY)<min {n(X), n(Y)}
B C B C ➋ n(XDY)가 최대일 때 n(XY)는 최소이고, n(XDY)가 최소일
때 n(XY)는 최대이다.
 (A-B)C+(AC)-(BC) ( _ )
n(U)=6, min{n(X), n(Y)}=4이므로 3<n(XDY)<4
1등급 NOTE 간단한 예를 이용하기
XY=(XCY)-(XDY)에서
A={1, 2, 3, 4}, B={2, 3, 5, 6}, A
C={3, 4, 6, 7}이라 하고 주어진 1
n(XY)=n(X)+n(Y)-2n(XDY)=9-2n(XDY)
보기의 내용들을 확인해도 된다. 따라서 n(XY)의 최솟값 m은 n(XDY)=4일 때 1이고,
2 4
즉 A-B={1, 4}이므로 3
n(XY)의 최댓값 M은 n(XDY)=3일 때 3이므로
5 7
(A-B)C={1, 3, 6, 7} 6
B C
m+M=1+3=4
또 AC={1, 2, 6, 7}, BC={2, 4, 5, 7}
이므로 (AC)-(BC)={1, 6}
LECTURE
따라서 (A-B)C+(AC)-(BC)
n(XCY)=n(X)+n(Y)-n(XDY)에서
참고 n(XDY)=9-n(XCY)이므로 n(XCY)의 최솟값과 최댓값을 생
각하면 n(X)+n(Y)-n(U)<n(XDY)<min{n(X), n(Y)}
대칭차집합에서 분배법칙은 성립하지 않는다.

10 ^③ 13 ^ 10개
GUIDE GUIDE
집합 D는 원소 0을 포함하므로 C+D임을 주의한다. {1, 2, 3, 4}의 부분집합 중 공집합과 {1, 3}, {1, 2, 4}처럼 원소가 연속
한 자연수가 아닌 경우를 제외한 것이 집합 S이다.
ㄱ. AB=(A-B)C(B-A)=B이면 A= ( _ )
ㄴ. 집합 D의 원소는 b+0일 때 순허수, b=0일 때 0이므로 S={1}일 때는 (0, 2)로 유일하다. 마찬가지로 집합 S가 각각
C-D=, D-C={0} {2}, {3}, {4}일 때 결정되는 (m, n)의 순서쌍도 1개씩이다.
CD=(C-D)C(D-C)={0} ( _ ) 즉 n(S)=1일 때, (m, n)의 순서쌍은 4개
ㄷ. 두 집합 A, B가 서로소이면 A-B=A, B-A=B이므로 n(S)>2이면 집합 S의 원소는 연속한 자연수 꼴이어야 한다.
AB=ACB ( ◯ ) 즉 S가 각각 {1, 2}, {2, 3}, {3, 4}일 때 (m, n)의 순서쌍도 각
각 1개씩이다.
또 S가 각각 {1, 2, 3}, {2, 3, 4}, {1, 2, 3, 4}일 때 (m, n)의 순
11 ^①
서쌍도 각각 1개씩이다.
GUIDE
따라서 순서쌍은 모두 4+3+3=10(개)
집합 An의 최소 정수와 집합 Bn의 최소 정수를 비교한다.

집합 An의 최소 정수는 (n€+n-1)+1=n€+n


집합 Bn의 최소 정수는 2n€+1 14 ^②
이때 2n€+1-(n€+n)=n€-n+1>0이 GUIDE
문제에서 주어진 조건을 벤 다이어그램에서 U
모든 자연수 n에 대하여 성립한다. A B
생각할 수 있다.
또 An의 최소 정수 n€+nIAnDBn이므로
이때 n(A)+0, n(B)+0,
1 1 1 n{(ACB)C}+0이다.
f(n)=n€+n=n(n+1)에서 = -
f(n) n n+1

1. 집합 05
n{(ACB)C}의 값이 1 또는 2일 때만 n(A)+0, n(B)+0이 17 ^③
된다. GUIDE
1 n {(ACB)C}=1, 예를 들어 (ACB)C={a}일 때 집합 A ➊ 8이 집합 X의 최소 원소가 되려면 X는 {8, 9, 10}의 부분집합 중 원
소 8을 반드시 포함하는 것이다.
로 가능한 것은 {b}, {c}, {d}, {b, c}, {b, d}, {c, d}, 즉 6가
➋ n이 집합 X의 최소 원소가 되려면 X는 {n, n+1, y, 10}의 부분집
지이고, 이 각각에 대하여 결정되는 집합 B는 유일하다.
합 중 원소 n을 반드시 포함하는 것이므로 f(n)=2(10-n+1)-1=210-n
C
(ACB) 가 {b}, {c}, {d}일 때도 마찬가지이다.
따라서 두 집합 A, B의 순서쌍은 모두 4_6=24(개) ㄱ. f(n)=210-n이므로 f(8)=210-8=4 ( ◯ )
2 n{(ACB)C}=2, 예를 들어 (ACB)C={a, b}일 때 ㄴ. [반례] f(9)=2, f(10)=1이므로 f(9)>f(10) ( _ )
집합 A로 가능한 것은 {c}, {d}이고, 이 각각에 대하여 결정 ㄷ. f(1)+f(3)+f(5)+f(7)+f(9)
되는 집합 B는 차례로 {d}, {c}이다. =2·+2‡+2—+2‹+2–=682 ( ◯ )
C
(ACB) 가 {a, c}, {a, d}, {b, c}, {b, d}, {c, d}일 때도 마
찬가지이다.
18 ^ 24개
따라서 두 집합 A, B의 순서쌍은 모두 6_2=12(개)
GUIDE
1, 2에서 순서쌍은 모두 24+12=36(개) AGK(U)이고, BGK(U)이므로 n(A)=n(B)=2이다.
벤 다이어그램에서 조건의 내용을 생각한다.

15 ^⑤ n(A)=n(B)=2이고 n(ADB)=1 U
GUIDE A B ㉣
이므로 전체집합 U의 네 원소 1, 2, 3,
f(1)은 ADB=에서 생각하고, f(2)는 n(ADB)=1인 경우부터
4가 각각 벤 다이어그램의 ㉠, ㉡, ㉢, ㉠ ㉡ ㉢
생각한다.
㉣에 하나씩 있어야 한다.
1 f(1)이면 n(A)=1이고 U 즉 경우의 수는 4_3_2_1=24이
ADB=이므로 오른쪽 벤 다이 A B 고, 각각에 대하여 순서쌍 (A, B)가 유일하다.
어그램에서 집합 A의 원소를 1개 1

뽑는 사건과 나머지 네 원소를 집


합 B와 (ACB)C에 나눠 주는 경
19 ^④
원소 4개 GUIDE
우를 함께 생각한다. 전체집합을 U, n(ADBDC)=x, n{(BDC)-A}=y라 놓고, 벤 다
  f(1)=5_2›=80 이어그램의 각 영역의 조건에 맞는 학생 수를 표시하는 것을 생각한다.
2 f(2)는 다음과 같이 생각한다. U
A B 벤 다이어그램의 각 영역에 원소 개수를 표시하면 다음과 같다.
1 ADB의 원소 정하기:5가지
U(100)
2 A-B의 원소 정하기:4가지 1 1 A
7
3 나머지 세 원소를 B-A 또는 15+x

(ACB)C에 나눠주기:2‹가지 15-x 10-x


원소 3개 x
   f(2)=5_4_2‹=160 20-y
y
42-y
1, 2에서 f(1)+f(2)=240 B C

이때 n(ACBCC)=102-y=93이므로 y=9
16 ^② x의 범위는 0<x<10이므로 두 문제 이상 맞힌 학생 수
GUIDE
25-x+y, 즉 34-x의 최솟값은 x=10일 때 24(명)
f(P)가 될 수 있는 값은 1, 2, 4, 8, y이다. f(A)=8이므로
f(ADB)=f(B)=4이다.

f(ADB)+f(B)=8이므로 f(ADB)=f(B)=4이고 20 ^ 108


GUIDE
n(ADB)=n(B)=2이다.
SDMC=에서 SAM이고, n(SDN)=2이므로 집합 S는
따라서 B=ADB이므로 집합 B로 가능한 것은 {1, 2, 3, 4}에서 두 개만 뽑은 것을 원소로 가지는 M의 부분집합이다.
{1, 2}, {2, 3}, {1, 3}이다. ※ 조건에 맞는 집합 S는 24개이고, n(S)는 4 이하라는 점도 생각하자.

참고 1 n(S)=2인 경우 {1, 2}, {1, 3}, {1, 4}, {2, 3}, {2, 4}, {3, 4}
부분집합의 개수는 2의 거듭제곱이므로 8=4+4만 가능하다. 를 하나씩 S¡, y, S§에 대응시킨다.

06 정답과 풀이
이때 a¡+a™+y+a§=3(1+2+3+4)=30 ㄴ. A¢={4, 8, 12, 16, 20}, A§={6, 12, 18}
2 1에서 구한 집합 각각에 0만 더 포함해 n(S)=3인 경우를 f(A¢)>f(A§) ( _ )
S¶, y, S¡™이라 하면 각 집합에 속한 원소의 합은 1에서 구한 ㄷ. A™DA£=A§이므로
것과 같다. 즉 a¶+ao+y+a¡™=30 f(A™CA£)=f(A™)+f(A£)-f(A§) ( ◯ )
3 1에서 구한 집합 각각에 -1만 더 포함해 n(S)=3인 경우를
S¡£, y, S¡o이라 하면
a¡£+a¡¢+y+a¡o=30-6=24 23 ^②
4 1에서 구한 집합 각각에 0과 -1을 더 포함해 n(S)=4인 GUIDE
경우를 S¡ª, y, S™¢이라 하면 적당한 자연수 n에 대하여 An, Bn을 구해 본다.

a¡ª+a™º+y+a™¢=30-6=24
ㄱ. 3 이하의 소수는 2, 3이므로 A£={2, 3}이고,
1~4 에서 30+30+24+24=108
4의 양의 약수는 1, 2, 4이므로 B¢={1, 2, 4}
 A£DB¢={2} ( ◯ )
다른 풀이
ㄴ. aGAn이면 a<n이고 a는 소수이다.
집합 S는 -1과 0을 각각 포함할 수도, 하지 않을 수도 있고 1, 2,
이때 a<n<n+1이고 a는 소수이므로 aGAn+1
3, 4 중에는 2개만 포함한다.
 AnAAn+1 ( ◯ )
1 -1이 포함된 집합의 개수는 0의 포함 여부와 1, 2, 3, 4 중 2
ㄷ. BmABn이면 m은 n의 약수이다. ( _ )
개만 포함하는 것을 따지면 2_6=12
2 1이 포함된 집합의 개수는 -1, 0의 포함 여부와 2, 3, 4 중 1개
만 포함하는 것을 따지면 2_2_3=12
24 ^ -20
3 2, 3, 4에 대해서도 마찬가지이므로
GUIDE
(-1+1+2+3+4)_12=108 ➊ S={a, b}이므로 aGS이면 a€+a-4GS는 a€+a-4=a 또는
a€+a-4=b임을 뜻한다.
➋ a+b, ab의 값을 구한다.

21 ^ ⑴ 672 ⑵ 636 aGS이면 a€+a-4=a 또는 a€+a-4=b


GUIDE 그런데 a€+a-4=a이면 a= 2이므로 ㈎에 어긋난다.
⑴ 예를 들어 U의 부분집합 중 반드시 원소 1을 포함하는 것은 모두
따라서 a€+a-4=b yy ㉠
26-1=2—(개)이다.
마찬가지로 생각하면 b€+b-4=a yy ㉡
⑵ ‘~적어도’가 있으므로 그 반대인 경우를 생각한다.
㉠-㉡에서 a€-b€+a-b=b-a, (a-b)(a+b+2)=0
⑴ U의 모든 부분집합 개수는 2˜이므로 n=2˜=64이다. 즉 a+b=-2 ( a+b)
이때 U의 한 원소를 반드시 포함하는 부분집합은 2—개이므로 ㉠+㉡에서 a€+b€+a+b-8=b+a
S(A¡)+S(A™)+y+S(A64) 즉 a€+b€=8
=(1+2+y+6)_2—=672 a€+b€=(a+b)€-2ab=4-2ab=8에서 ab=-2
⑵ A¡, A™, y, An 중 짝수인 원소가 없는 것은 {1, 3, 5}의 부분  a‹+b‹=(a+b)‹-3ab(a+b)=-8-12=-20
집합과 같고, 그 개수는 2‹=8
이 부분집합을 C¡, C™, y, Co이라 하면
S(C¡)+S(C™)+y+S(Co)=(1+3+5)_2€=36
25 ^③
 S(B¡)+S(B™)+y+S(Bv)=672-36=636 GUIDE
6으로 나눈 나머지가 0, 1, 2, y, 5이므로 Aº, A¡, y, A5를 생각한다.

Aº={6n|n은 1<n<16인 정수}  n(Aº)=16


22 ^③ A¡={6n+1|n은 0<n<16인 정수}  n(A¡)=17
GUIDE A™={6n+2|n은 0<n<16인 정수}  n(A™)=17
A={5, 10, 15, 20}이므로 f(A)=5+10+15+20이다.
A£={6n+3|n은 0<n<16인 정수}  n(A£)=17
f(U)=f(A¡)=1+2+y+20
A¢={6n+4|n은 0<n<16인 정수}  n(A¢)=17
C C
ㄱ. A =U-A이고, ADA =이므로 A={6n+5|n은 0<n<15인 정수}  n(A)=16
C
f(A )=f(U)-f(A) ( ◯ ) ㄱ. A¡에서 n=6일 때, 6_6+1=37  37GA¡ ( ◯ )

1. 집합 07
ㄴ. ( _ ) 02 ^ 336
ㄷ. aGA™, bGA£, cGA¢이므로 GUIDE
a=6p+2, b=6q+3, c=6r+4라 하면 ㈎에서 1, 2, 3, 4 중 두 수만 집합 P의 원소가 될 수 있다.
또 두 집합이 어떤 경우일 때 P-B=가 되는지 생각해 본다.
(p, q, r는 0부터 16까지의 정수)
3a+bc=3(6p+2)+(6q+3)(6r+4) n(PDA)=2이므로 집합 A의 원소 중 2개만 집합 P에 속한다.
=6(6qr+3p+4q+3r+3) 이때 A의 원소 중 P에 속하는 원소들 합의 최댓값은 3+4=7,
이므로 3a+bcGAº ( ◯ ) 최솟값은 1+2=3이므로 P의 원소 중 A에 속하지 않는 원소들
의 합을 S라 하면
21<S<25 yy ㉠
26 ^ 32개
또 P-B=에서 PAB이고, 집합 B의 원소 중 A에 속하지
GUIDE
A✽B=(U-A)C(U-B)=ACCBC이고
않는 원소는 5, 6, 7, 8이므로
C C
(U-A)✽(U-B)=(A ) C(B ) =ACB C C 1 5, 6, 7, 8이 P에 포함될 때
집합 P의 원소의 합은 26이므로 ㉠의 범위 밖이다.
A✽B=ACCBC이고, (U-A)✽(U-B)=ACB
2 5, 6, 7, 8 중 3개만 P에 포함될 때
또 U-(A✽B)=UD(ACCBC)C=UD(ADB)=ADB
6+7+8=21이므로 ㉠의 범위에 맞는 것은 6, 7, 8 뿐이다.
이때 (U-A)✽(U-B)=U-(A✽B), 즉
즉 ㈐에 따라 {3, 4, 6, 7, 8}은 집합 P가 될 수 있다.
ACB=ADB에서 A=B
3 5, 6, 7, 8 중 2개만 집합 P에 포함될 때
따라서 전체집합 U={1, 2, 3, 4, 5}의 부분집합 개수만큼
㉠의 범위 밖이다.
A=B인 순서쌍 (A, B)가 존재하므로 개수는 2—=32
1~3에서 P={3, 4, 6, 7, 8}만 가능하므로
P-A={6, 7, 8}
따라서 P-A의 모든 원소의 곱은 6_7_8=336

03 ^ 1440
GUIDE
1부터 10까지 모든 자연수를 약수로 가지는 가장 작은 자연수

STEP 3 1등급 뛰어넘기 p. 14~17 2‹_3€_5_7에서 생각한다.

01 6 02 336 03 1440 04 ⑤ 1부터 10까지 모든 자연수를 약수로 가지는 가장 작은 자연수는


05 18 06 9 07 32 08 ③
2‹_3€_5_7이므로 소인수의 지수를 바꿔 약수 2개를 뺄 수 있
09 ③ 10 ㄴ, ㄷ 11 ⑤ 12 {3, 4, 6}
는 경우를 생각해 본다.
13 ⑤ 14 80 15 ④
1 2_3€_5_7=630일 때 4, 8이 빠지지만 범위를 벗어난다.
01 ^6 2 2€_3€_5=180일 때 7, 8이 빠지므로 가능
GUIDE
3 2€_3_5_7=420일 때 8, 9가 빠지므로 가능
A={a, b, c}일 때 만들 수 있는 모든 세 자리 자연수의 합을 나타내 본다.
4 2‹_3_5=120일 때 7, 9가 빠지므로 가능
세 수 a, b, c로 만들 수 있는 세 자리 자연수는 또 2›_3_5=240, 2—_3_5=480도 마찬가지로 7, 9가 빠
abc, acb, bac, bca, cab, cba, 즉 지므로 가능
100a+10b+c, 100a+10c+b, 100b+10a+c, 따라서 조건에 맞는 수는 180, 420, 120, 240, 480이므로 합은
100b+10c+a, 100c+10a+b, 100c+10b+a이므로 1440
이 6개 자연수의 합은 222(a+b+c)
이때 1554<222(a+b+c)<1998에서

04
7<a+b+c<9이므로
^⑤
1 a+b+c=7:A={1, 2, 4} GUIDE
2 a+b+c=8:A={1, 2, 5}, A={1, 3, 4} ➊ AnDXn=Xn에서 XnAAn이다.
3 a+b+c=9:A={1, 2, 6}, A={1, 3, 5}, A={2, 3, 4} ➋ n(A)=n일 때, 특정한 원소 k(k<n)개를 포함하지 않는 집합 A의
부분집합은 모두 2n-k(개)
따라서 가능한 집합 A는 모두 6개

08 정답과 풀이
ㄱ. An={1, 2, 3, y, 2n}, Bn={1, 3, y, n} 또는 2a_2b-2a-2b=0, (2a-1)(2b-1)=1
Bn={1, 3, y, n-1}이므로 BnAAn이다. ( ◯ ) 이때 2a, 2b 모두 자연수이므로
ㄴ. X£은 A£의 부분집합 중 B£의 원소를 적어도 하나 포함해야 2a=2b=2  a=1, b=1
하므로 1, 3을 모두 포함하지 않는 부분집합을 제외하면 따라서 n(ADB)=9
2˜-2›=48(개) ( ◯ )
1등급 NOTE
ㄷ. n이 짝수일 때 An={1, 2, 3, y, 2n-1, 2n}이고,
2 +2a=2b 꼴이 되는 경우는 a=10, b=11일 때이다.
10

n
Bn={1, 3, 5, y, n-1}이므로 n(Bn)= 이다.
2
BnAAn, BnDXn+에서 집합 Xn은 An의 부분집합 중
n
Bn의 원소 개를 포함하지 않는 부분집합을 제외한 것이므로
2 07 ^ 32
GUIDE
2n 2n-;2N; n 3n
그 개수는 2 -2 =4 -2 2 (◯)
n
k=1, 2, 3, y일 때 “ ‘=4에서 자연수 n을 찾는다.
k
참고 홀수의 개수

➊ 홀수 ‘1, 3, 5, y, n’의 개수는


n+1
개 k=1일 때 [n]=4에서 n=4
2
n n
➋ 홀수 ‘1, 3, 5, y, n-1’의 개수는
n
개 k=2일 때 “ ‘=4에서 4< <5 ∴ 8<n<10
2 2 2
1부터 시작하여 k번째 홀수는 2k-1이므로 2k-1=(마지막 홀수)에서 n n
k=3일 때 “ ‘=4에서 4< <5 ∴ 12<n<15
k값을 구하면 알 수 있다. 3 3

즉 자연수 n의 값은 4, 8, 9, 12, 13, 14, y이므로

05 ^ 18 m=13이고, 이때 A13={13, 6, 4, 3, 2, 1}
GUIDE 따라서 {4}AXAA13인 집합 X는 26-1=32(개)
자연수를 4로 나눈 나머지는 0, 1, 2, 3 중 하나이고 n(B)=4이므로
집합 B={0, 1, 2, 3}이다. 또 n(ADB)=2이므로 B의 원소 중 집합
A의 원소가 될 수 있는 것은 1, 2, 3 중 2개이다.

1, 2 또는 2, 3이 집합 A의 원소이면 ㈐에 어긋나므로 1, 3이 A 08 ^③
의 원소가 되어야 한다. GUIDE
집합 A의 원소에서 나머지가 0, 나머지가 2인 것이 있어야 집합 T는 과 {a, b, c}를 포함하고 있으므로 X와 부분집합 중에서 
과 X가 아닌 것을 3개를 골라야 n(T)=5가 된다.
B={0, 1, 2, 3}이 될 수 있다.
즉 2, 4, 6, 8 중에서 2, 4는 ㈐에 어긋나므로 A={1, 3, 6, 8} {a}, {b}, {c}, {a, b}, {a, c}, {b, c} 중 3개를 골라야 한다.
(ACCBC)CC(ADBC)=(ADB)C(ADBC) 이때 ㈏에 따라
=AD(BCBC) 1 원소가 1개인 부분집합에서 2개를 고르면 원소가 2개인 부분
=ADU=A 집합 1개는 자동으로 결정된다.
따라서 집합 A의 원소의 합은 1+3+6+8=18 이때 경우의 수는 3
2 원소가 2개인 부분집합에서 2개를 고르면 원소가 1개인 부분
집합 1개는 자동으로 결정된다.
06 ^9 이때 경우의 수는 3
GUIDE 1, 2에서 조건에 맞는 집합 T는 모두 6개
A B
n(A-B)=a, n(B-A)=b라 하면
n(ADB)=10-a이다. a 10-a b 참고
이때 N(A)=210, N(B)=210-a+b
➊ 원소가 1개인 부분집합에서 3개를 고르면 ㈏의 ‘AGT, BGT이면
ACBGT’에 어긋난다. 또 원소가 2개인 부분집합에서 3개를 고르
n(A-B)=a, n(B-A)=b라 하면 면 ㈏의 ADBGT에 어긋난다.
n(ADB)=10-a, n(B)=10-a+b, n(ACB)=10+b ➋ 예를 들어 1에서 원소가 1개인 A={a}와 B={b}를 골랐다면 원소
㈎ 에서 210+210-a+b=210+b 가 2개인 집합은 {a, b}가 되어야 ㈏의 ‘AGT, BGT이면
ACBGT’이 성립한다. 즉 {, {a}, {b}, {a, b}, {a, b, c}}AT이
2b
양변을 210으로 나누면 1+2-a+b=2b, 1+ =2b 지만 {, {a}, {b}, {b, c}, {a, b, c}}ET이다. 2도 마찬가지이다.
2a

1. 집합 09
09 ^③ ㄴ. A™의 원소는 소수부분이 1-;4!;=;4#;이고,
GUIDE
1002
X={a, b, c}이므로 {A|AAX}를 원소나열법으로 나타내면 =250+;4@;이므로 소수부분이 ;2!;이다. ( _ )
4
{, {a}, {b}, {c}, {a, b}, {a, c}, {b, c}, X}이다.
ㄷ. 1+2이므로 ㄱ에서 A¡DA™= ( _ )
ㄱ. T={, {a}, {b}, {c}, {a, b}, {a, c}, {b, c}, X}이면
ㄹ. A£의 원소는 소수부분이 1-;6!;=;6%;이고, -;6&;=-2+;6%;이
㈎, ㈏, ㈐ 모두 성립함을 알 수 있다. ( ◯ )
므로 A£의 원소이다. ( ◯ )
예 A={a, b}, B={b, c}일 때
X-A={c}GT ⇨ ㈏
ACB={a, b, c}GT ⇨ ㈐
ㄴ. X-=XGT, X-X=GT ⇨ ㈏ 12 ^ {3, 4, 6}
GUIDE
XC=XGT ⇨ ㈐ ( ◯ )
ADB={6}이므로 A={a, b, 6}으로 놓을 수 있다.
ㄷ. X-{a}={b, c}GT, X-{b, c}={a}GT ⇨ ㈏
이때 B={ka, kb, 6k}이고 6k+6 ( k+1)이므로 ka 또는 kb가 6이
{a}C{b, c}=XGT이고, {a}C={a}GT, 어야 한다.
{b, c}C={b, c}GT, {a}CX=XGT,
A={a, b, 6}, kb=6이라 하면 ACB={a, b, 6, ka, 6k}
{b, c}CX=XGT ⇨ ㈐ ( ◯ )
또 M(A)=6ab=72이므로 ab=12
ㄹ. [㈏의 반례] X-{b}={a, c}IT ( _ )
M(ACB)=6€a€bk€=2°_3‹이므로 ak€=16
이때 ab=12 (a, b는 자연수)인 (a, b), 즉 (1, 12), (3, 4), (4, 3),
(12, 1)에 대하여 ak€=16이 되는 경우는 (4, 3)일 때 뿐이다.

10 ^ ㄴ, ㄷ 즉 a=4, b=3일 때
GUIDE kb=6에서 k=2이므로 ak€=16
x가 임의의 실수이므로 [x]는 임의의 정수이다. 따라서 n[x]는 n의 배수 따라서 a=4, b=3이므로 A={3, 4, 6}
이고, 집합 An은 n의 배수 전체의 집합이다.
1등급 NOTE
ㄱ. A™는 2의 배수의 집합, A¢는 4의 배수의 집합이므로
6GA이므로 72=6_12에서 A의 나머지 원소를 3, 4 또는 1, 12로 생
A™BA¢ ( _ ) 각할 수 있다.
ㄴ. A£은 3의 배수의 집합, A는 5의 배수의 집합이므로
A£DA는 15의 배수의 집합이다. ( ◯ )
ㄷ. 1, 2, 3, y, n의 최소공배수를 k라 하면 13 ^⑤
A¡DA™DA£DyDAn=Ak이므로 GUIDE
A¡DA™DA£DyDAn=인 n은 존재하지 않는다. ( ◯ ) 소수 p에 대하여 p€과 서로소가 아닌 수는 p의 배수이고, p€의 약수는
1, p, p€이므로 Ap€={p, 2p, 3p, y, p€}, Bp€={p, p€}

ㄱ. A¢={2, 4}, B¢={2, 4} ( ◯ )


ㄴ. m이 n의 양의 약수이면 AmAAn ( ◯ )

11 ^⑤ ㄷ. Ap€={p, 2p, 3p, y, p€}, Bp€={p, p€}이므로


GUIDE Ap€-Bp€={2p, 3p, y, (p-1)p}이다.
1   n(Ap€-Bp€)=p-2 ( ◯ )
x가 정수이면 <x>-x=0인데 +0이므로 정수는 집합 An의 원소가
2n
될 수 없다.
x=n+a (n은 정수, 0<a<1)이라 하면
1 1 14 ^ 80
<x>=n+1이고, <x>-x=1-a=  a=1-
2n 2n GUIDE
{1, 2, 3, 4, 5}의 특정 원소 1개를 반드시 포함하는 부분집합은
1 25-1=16(개)이고, 이 원소를 포함하지 않는 부분집합도 16개이므로 특정
집합 An의 원소는 소수부분이 1- 인 실수이다.
2n
원소를 포함하는 부분집합과 포함하지 않는 부분집합을 짝지을 수 있다.
1
ㄱ. Ai의 원소는 소수부분이 1- 이고, Aj의 원소는 소수부분
2i {1, 2, 3, 4, 5}의 2—개의 부분집합 중 가장 큰 원소인 5를 포함하
1 지 않는 부분집합을 XA라 하고, 5를 포함하는 부분집합을 XB라
이 1- 이다. 즉 i+j이면 AiDAj= ( ◯ )
2j
하여 다음과 같이 짝지어서 m(XA)+m(XB)를 구해 보자.

10 정답과 풀이
XA $# XB m(XA)+m(XB)
02 명제
 $# {5} 0+5=5
{1} $# {1, 5} 1+(5-1)=5 STEP 1 1등급 준비하기 p. 20 ~21
{2} $# {2, 5} 2+(5-2)=5
01 {-3} 02 ④ 03 ④ 04 ①
{3} $# {3, 5} 3+(5-3)=5
05 ④ 06 ⑤ 07 7개 08 ②
{1, 2} $# {1, 2, 5} (2-1)+(5-2+1)=5
09 -3 10 ③
⋮ ⋮

01
(4-3+2-1)
{1, 2, 3, 4} $# {1, 2, 3, 4, 5} ^ {-3 }
+(5-4+3-2+1)=5
GUIDE
 m(X¡)+m(X™)+y+m(X31)=5_16=80 ‘p이고 ~q’의 진리집합은 PDQC

조건 p, q의 진리집합을 각각 P, Q라 하면
P={-3, 3}, Q={0, 3}
따라서 ‘p이고 ~q’의 진리집합은 PDQC=P-Q={-3}
15 ^④
GUIDE
백화점 A, B, C를 이용한 주부의 집합을 각각 A, B, C라 하고 조건의
02 ^④
내용을 기호로 나타낸 다음 벤 다이어그램을 이용해 본다.
GUIDE
ㄱ. n(ADBDC)>25인 예를 찾아본다.
‘모든 ~ 이다.’의 부정은 ‘어떤 ~ 아니다.’이다.

조건에서 n(A)=88, n(B)=75, n(C)=50


‘모든 무리수 a에 대해 a는 무한소수이다.’는 무리수의 집합 A가
ㄱ. [반례] n(ADBDC)=50 A
무한소수의 집합 B에 포함된다는 것과 같다.
n(ADBCDCC)=25, 25
AAB의 부정 AEB는 A의 원소 중 B에 포함되지 않는 것이
n(ACDBDCC)=12, 13
50 있다는 것이므로 주어진 명제의 부정은 ‘어떤 무리수 a에 대해 a
n(ADBDCC)=13 ( _ ) 12 는 유한소수이다.’이다.
B C
ㄴ. n(A)=88이므로
n(ACBCC)>88이다. A
C C
따라서 n(A DB DC )<12이므 C 8 03 ^④
GUIDE
로 A, B, C 어느 곳도 이용하지 않 30 5
45 진리집합의 포함 관계를 통하여 참인 명제를 찾는다.
은 주부는 12명을 넘을 수 없다. 0 0
0
B C PEQ이므로 p 46, q이다.
오른쪽 벤 다이어그램은
n{(ACBCC)C}=12인 예이다. ( ◯ ) QARC이므로 q %%, ~r이다.
ㄷ. n(ADBDC)=p, A 따라서 참인 것은 ④
n(ADBDCC)=a, 88-(a+c+p)
C
n(A DBDC)=b, a c
C
p 04 ^①
n(ADB DC)=c 75-(a+b+p) 50-(b+c+p)
b GUIDE
B C
라 하면 다음과 같다. ‘p이면 ~q이다.’가 거짓이면 ‘p이면 q이다.’는 참이다.
88-(a+c+p)>0, 75-(a+b+p)>0,
‘p이면 ~q이다.’가 거짓임을 보여 주는 원소는 ‘p이면 q이다.’가
50-(b+c+p)>0에서
참임을 보여 주는 원소이다. PDQ={2, 3}이므로 2+3=5
a+c+p<88, a+b+p<75, b+c+p<50
세 부등식을 더하면 2(a+b+c)+3p<213
한편 벤 다이어그램 각 영역의 값을 모두 더했을 때 100을 넘 05 ^④
을 수 없으므로 213-(a+b+c+2p)<100, GUIDE
즉 a+b+c+2p>113 진리집합의 포함 관계가 PAQ인 것을 찾는다.

부등식의 양변에 2를 곱하면 2(a+b+c)+4p>226,


ㄱ. 두 조건 p, q의 진리집합을 각각 P, Q라 하면 PAQ ( ◯ )
226<2(a+b+c)+3p+p<213+p
ㄴ. [반례] x='2, y='2 일 때, xy=2는 유리수이다. ( _ )
 p>226-213=13
ㄷ. x=2n (n은 자연수)이면 x€=4n€이므로 x가 짝수이면 x€
따라서 세 백화점을 모두 이용한 주부는 최소 13명이다. ( ◯ )
도 짝수이다. ( ◯ )

2. 명제 11
06 ^⑤ 09 ^ -3
GUIDE GUIDE
명제가 참이면 대우도 참이다. PAQ가 성립하려면 a는 P의 가장 작은 원소 2보다 작아야 하고 a€은
P의 가장 큰 원소인 7보다 커야 한다.
대우 ‘~q이면 ~p이다.’도 참이므로
x+1=0일 때 x€-ax+2=0이다. P={2, 3, 5, 7}, Q={x|a<x<a€, x는 정수}에서 PAQ이려

(-1)€-a_(-1)+2=0  a=-3 면 다음과 같아야 한다.


Q

a 2 3 5 7 a€ x

즉 a<2이고 a€>7에서 공통 부분은 a<-'7


07 ^ 7개
따라서 이 범위에 있는 가장 큰 정수는 -3
GUIDE
진리집합을 구한다. 범위를 구할 때는 등호에 주의한다.

조건 p, q의 진리집합을 P, Q라 하면 10 ^③
GUIDE
조건 ~q:x<-1 또는 x;4A;의 진리집합 QC에 대하여
소수의 개수가 유한하다고 가정했을 때 모순이 되는 경우를 찾는다.
C
명제 p !# ~q가 참이기 위해서는 PAQ 에서
소수의 개수가 유한하다고 가정하고, 소수들을 크기순으로 배열
-a<-1이고, ;4A;2이어야 한다. 하여 a¡, a™, y, an이라 하자.

QC QC
이때 ㈎ a¡_a™_y_an+1 은 n개의 소수 중 어느 것으로도
P P
나누어떨어지지 않으므로 ㈎ a¡_a™_y_an+1 은
-a -1 2 x
;4A; ㈏ 소수 이다.

따라서 1<a8 범위에 있는 자연수는 7개 또한 ㈎ a¡_a™_y_an+1 > ㈐ an 이므로


가장 큰 소수가 an이라는 가정에 모순된다.
다른 풀이 따라서 소수의 개수는 무한하다.
대우인 명제 q !# ~p가 PC
참이 되도록 하려면 QAP C Q

-a -1 2 x
즉 -a<-1, ;4A;<2에서 ;4A;
STEP 2 1등급 굳히기 p. 22~26

1<a<8
01 ③ 02 ③ 03 ③ 04 ④
05 384 06 ③ 07 ⑤ 08 ③
1등급 NOTE
09 ⑴ {x|3<x<5} ⑵ 5<k<8 10 ⑤
등호를 포함한 경우일 때 참이 되는지 확인한다. 위 경우 a=1, a=8일
11 9개 12 ⑤ 13 ③
때를 대입해 보면 a+1, a=8임을 알 수 있다.
14 ⑴ -3 ⑵ -2 15 19 16 ①
17 ⑴ a€+b€+c€ ⑵ 홀수
18 ⑴ 실수 ⑵ z” ⑶ < ⑷ zz”>0 19 ②
20 ⑤
08 ^②
GUIDE 01 ^③
p가 q이기 위한 필요조건이면 q는 p이기 위한 충분조건이므로 QAP GUIDE
또 ~p가 r이기 위한 필요충분조건이면 PC=R ‘모든’을 포함하는 명제는 반례가 존재하면 거짓이고, ‘어떤’을 포함한 명
제는 조건이 참이 되는 예가 하나만 있어도 참이다.
p가 q이기 위한 필요조건이므로 U
P ①의 부정은 ‘모든 실수 x에 대하여 x<|x|’ [반례] x=0
QAP이고, ~p가 r이기 위한 필 Q
요충분조건이므로 PC=R이다. R=PC ②의 부정은 ‘모든 실수 x에 대하여 x+;x!;2’ [반례] x=2
따라서 오른쪽과 같은 벤 다이어
③의 부정은 ‘모든 실수 x에 대하여 x€>x-1’
그램으로 나타내면 항상 성립하는
것은 PCDR=R 즉 x€-x+1={x-;2!;}€+;4#;>0이므로 참

12 정답과 풀이
④의 부정은 ‘어떤 실수 x에 대하여 x€+6x+100’ 참고

판별식 D<0이므로 모든 실수 x에 대하여 x€+6x+10>0 두 집합 P, Q의 예를 오른쪽과 같이 Q Q


수직선에서 생각할 수 있다. P
따라서 거짓
⑤의 부정은 ‘어떤 실수 x, y에 대하여 |x+y|>|x|+|y|’ -1 1 4 5 x
P-Q
모든 실수 x, y에 대해서 |x+y|<|x|+|y|이다.
따라서 거짓
05 ^ 384
1등급 NOTE GUIDE
주어진 명제가 거짓이면 그 명제의 부정은 참이다. 자연수 n에 대하여 n€이 12의 배수이려면 n은 2의 배수이면서 3의 배수

따라서 주어진 명제 중에서 거짓인 것을 찾으면 된다. 이어야 한다.

n€이 12, 즉 2€_3의 배수이면 n은 2의 배수이면서 3의 배수, 즉


6의 배수이다.
02 ^③
따라서 6의 배수이지만 12의 배수가 아닌 것이 모두 반례이므로
GUIDE
100보다 작은 것은 6, 18, 30, 42, 54, 66, 78, 90이고, 그 합은
ADB=B ⇨ BAA, 이때 ACABC
6+18+30+42+54+66+78+90
C C C
(P CQ)D(PCQ )=PCQ 이므로 =6_(1+3+5+7+9+11+13+15)
C C
(PCQ )A(P CQ) =6_64=384
C C C
이때 P와 P 는 서로소이므로 PAQ이고 Q AP 이다.
따라서 명제 p !# q가 참이다. 참고

12k+6<100 (k는 0 이상인 정수)에서 k=0, 1, 2, 3, y, 7이므로


참고 (12_0+6)+(12_1+6)+y+(12_7+6)
C C C C C C =12(0+1+2+y+7)+6_8
(PCQ )A(P CQ)이므로 (PCQ ) B(P CQ)
C C =12_28+48=384
따라서 (PDQ )A(QDP ), 즉 P-QAQ-P에서 PAQ임을 알 수
있다.

06 ^③
GUIDE
03 ^③
GUIDE ➊ 유리수 조건에서 0과 같이 특수한 경우를 생각한다.

➊ A-B=ADB C ➋ (실수)€>0

➋ ADBDC=A ⇨ AAB이고 AAC


ㄱ. x€+6x+9=(x+3)€이므로 x가 유리수인 -3인 경우를 제
(PDQ)-R=PDQDR =P이므로 PAQ이고 PAR C C
외하고 항상 x€+6x+9>0이다. ( ◯ )
따라서 p !# q, r !# ~p는 항상 참이다. m€
ㄴ. a= (단, m은 0 이상의 정수, n은 자연수)이면
n€
참고 'a도 유리수이고, a도 유리수이다. ( ◯ )
다음과 같은 벤 다이어그램을 생각할 수 있다.
ㄷ. 무리수 a에 대하여 ab=2가 성립하면 b=;a@;
Q R Q RC
P 이때 a가 무리수이므로 b도 무리수이다. ( ◯ )
P
ㄹ. [반례] a=0이면 ab=2가 되는 유리수 b는 없다. ( _ )
ㅁ. a가 음수이면 b€=a가 되는 무리수 b는 없다. ( _ )
따라서 참인 것은 ㄱ, ㄴ, ㄷ으로 3개

04 ^④
GUIDE 07 ^⑤
AAB일 때 ADB=A임을 이용한다. GUIDE
(유리수)_(유리수)=(유리수)
명제 ~p !# q의 대우 ~q !# p가 참이므로 QCAP
또 명제 ~p !# q의 역인 명제 q !# ~p가 거짓이므로 ② p이고 q이면 xGQ이고 x€GQ이므로 x‹=x_x€GQ ( ◯ )
C C C C
QEP , 즉 Q FP  P-Q=PDQ =Q ④ ~p!#(~q 또는 ~r)의 대우는 (q이고 r)!#p이다.

2. 명제 13
q이고 r이면 x€GQ이고 x‹GQ이므로 10 ^⑤
x‹
=xGQ (x+0) GUIDE
[ x€ (◯) p가 q이기 위한 필요조건이면 명제 q !# p가 참이고 명제 p !# q는 거
xGQ (x=0)
짓이다.
⑤ [반례] x='2 이면 x€GQ이지만 x‹IQ ( _ )
ㄱ. p:ab=0이면 a=0 또는 b=0
q:z€<0이면 z는 순허수이므로 a=0, b+0

08 ^③ 즉 p 46, q이고 q %%, p이므로 p는 q이기 위한 필요조건이다.


GUIDE ㄴ. p:z€=z€” 이면 a€-b€+2abi=a€-b€-2abi
ㄴ. XA(ACB)이면 X의 원소 x가 A 또 A B  ab=0
는 B에 속하면 된다. 모든 원소가 A에 X q:z는 순허수이므로 a=0, b+0
속하거나 모든 원소가 B에 속하지 않아
즉 p 46, q이고 q %%, p이므로 p는 q이기 위한 필요조건이다.
도 된다.
ㄷ. p:a€=b€이면 a=b 또는 a=-b
ㄱ. 두 실수 x, y에 대하여 x€+y€=0이면 x=0, y=0 q:z”z=0이면 a€+b€=0  a=0, b=0
이때 x€+y€=xy이므로 명제가 참이다. 즉 p 46, q이고 q %%, p이므로 p는 q이기 위한 필요조건이다.
ㄴ. 세 집합 A, B, X에 대하여 XA(ACB)이면 X의 원소 x 따라서 p가 q이기 위한 필요조건인 것은 ㄱ, ㄴ, ㄷ
가 A 또는 B에 속하면 되고, X의 모든 원소가 A에 속하거
나 모든 원소가 B에 속하지 않는 경우가 있으므로 명제는 거
짓이다.
한편 XAA 또는 XAB이면 X의 모든 원소는 ACB에 속 11 ^ 9개

한다. 따라서 명제의 역은 참이다. GUIDE


p가 q이기 위한 필요조건이면 명제 q!#p가 참이므로 두 조건의 진리
ㄷ. 두 실수 x, y에 대하여 x€+y€>0이면 x 또는 y가 0이 아니
집합 P, Q에 대하여 QAP이다.
다. 즉, x, y 중 하나는 0일 수 있으므로 명제는 거짓이다.
한편, xy+0이면 x, y 모두 0이 아니므로 x€+y€>0이다. 조건 p, q의 진리집합을 각각 P, Q라 하면
따라서 명제의 역은 참이다. p:|x-5|<a에서 5-a<x<5+a이므로
ㄹ. |a|+|b|>|a+b|에서 부등식의 양변 모두 0 이상이므로 P={x|5-a<x<5+a, a는 한 자리 자연수}
주어진 부등식과 (|a|+|b|)€>|a+b|€은 같다. q:|x-b|<7에서 b-7<x<b+7이므로
(|a|+|b|)€>|a+b|€에서 |ab|>ab Q={x|b-7<x<b+7, b는 한 자리 자연수}
즉 ab<0일 때 성립하므로 명제는 참이다. p가 q이기 위한 필요조건이 되려면 QAP이어야 하므로
[역의 반례] a=-1, b=0이면 |a|+|b|=1, |a+b|=1 5-ab-7, b+75+a
이므로 1>1이 되어 거짓이다. 즉 a+b12, a-b2가 되는 순서쌍 (a, b)는
ㅁ. a=-3, b='3 일 때 명제는 성립하지 않으므로 거짓이다. a=9일 때 (9, 3), (9, 4), (9, 5), (9, 6), (9, 7)
한편 a=b=0이면 a+b'3=0이므로 명제의 역은 참이다. a=8일 때 (8, 4), (8, 5), (8, 6)
따라서 명제는 거짓이지만 역이 참인 것은 ㄴ, ㄷ, ㅁ으로 3개 a=7일 때 (7, 5)
따라서 조건에 맞는 순서쌍은 모두 9개

09 ^ ⑴ {x|3<x<5} ⑵ 5<k<8
GUIDE
'a
=-Æ
a
⇨ a=0 또는 a>0, b<0
12 ^⑤
'b b GUIDE
p가 q이기 위한 충분조건이지만 필요조건이 아니면 P는 Q의 진부분집
'ßx-3 x-3 합이다.
⑴ =-æ 에서 x=3 또는 x-3>0, x-5<0이므
'ßx-5 x-5
로 조건 p의 진리집합을 P라 하면 P={x|3<x<5} p가 q이기 위한 충분조건이지만 필요조건이 아니므로
⑵ 조건 q의 진리집합을 Q라 하면 PAQ이고, QEP이다.
Q={x|k-5<x<k} r는 ~q이기 위한 충분조건이지만, ~q는 r이기 위한 충분조건
명제 p !# q가 참이 되려면 PAQ이어야 하므로 이 아니므로 RAQC이고, QCER이다.
k-5<3이고 5<k   5<k<8 따라서 벤 다이어그램으로 나타내면 다음과 같다.

14 정답과 풀이
U p가 q이기 위한 필요충분조건이므로 두 진리집합 P, Q는 P=Q
Q R 이다.
P
따라서 두 집합 P, Q의 원소의 합과 곱은 같다.
a+b+c=ab+bc+ca=4 yy ㉠
abc=(abc)€ yy ㉡
① RAPC ( ◯ )
㉡에서 abc(abc-1)=0  abc=1 ( abc+0)
② PARC이므로 P-RC= ( ◯ )
㉠에서 a€+b€+c€=(a+b+c)€-2(ab+bc+ca)
③ PAQ이므로 PCCQC=(PDQ)C=PC ( ◯ )
=4€-2_4=8
④ RDQ= ( ◯ )
따라서
⑤ QCER ( _ )
a‹+b‹+c‹
=(a+b+c)(a€+b€+c€-ab-bc-ca)+3abc

13 ^③ =4_(8-4)+3_1=19
GUIDE
➊ 조건이 모든 x에 대하여 참이면 진리집합은 전체집합 U이다.
➋ 조건이 모든 x에 대하여 거짓이면 진리집합은 공집합 이다.
16 ^①
조건 (~p 또는 q)가 모든 x에 대하여 U GUIDE
Q
참이므로 PCCQ=U ➊ ACDB=B-A=B이면 ADB=
P
따라서 ➋ AC-BC=ACDB=B-A=이면 BAA

(PCCQ)C=PDQC=P-Q=
(PDQ)CDR=R-(PDQ)=R ⇨ RD(PDQ)=
이므로 PAQ
QC-RC=QCDR=R-Q= ⇨ RAQ
ㄱ. ( ◯ )
세 집합을 벤 다이어그램으로 나타내면 다음과 같다.
ㄴ. 조건 p는 조건 q이기 위한 충분조건이다. ( _ )
P Q
ㄷ. PDQC=이므로 조건 (p이고 ~q)는 모든 x에 대하여 거 R
짓이다. ( ◯ )

ㄱ. QER ( _ ) ㄴ. RAQ ( ◯ )
14 ^ ⑴ -3 ⑵ -2
ㄷ. RA(PCQ) ( ◯ ) ㄹ. (PCR)EQ ( _ )
GUIDE
㈎ 조건 q가 조건 p이기 위한 충분조건이지만 필요조건이 아니면 Q는 P ㅁ. (PDR)AQ ( _ )
의 진부분집합이다.
㈏ 조건 p가 조건 ~r이기 위한 필요조건이지만 충분조건이 아니면 RC
는 P의 진부분집합이다.
17 ^ ⑴ a€+b€+c€ ⑵ 홀수
세 조건 p, q, r의 진리집합을 각각 P, Q, R라 하면 GUIDE
Q={1}, RC={b}이고 ➊ ‘a, b, c가 모두 홀수이다.’의 부정은 ‘a, b, c 중 적어도 하나는 짝수이
다.’이다.
㈎에서 QAP, PEQ, ㈏에서 RCAP, PERC
➋ 홀수의 제곱은 홀수이다.
⑴ QAP이므로 x=1은 집합 P의 원소이다.
1+a+2=0  a=-3 ⑴ ‘a€+b€=c€이다.’의 부정이므로 ‘a€+b€+c€이다.’
C
⑵ R AP이므로 x=b는 집합 P의 원소이다. ⑵ a, b, c가 모두 홀수이면 a€, b€, c€은 모두 홀수이다.
b‹-3b+2=0, (b-1)€(b+2)=0에서 b=1 또는 b=-2 이때 a€+b€은 짝수이고, c€은 홀수이므로 a€+b€+c€이 성립
그런데 q는 ~r이기 위한 충분조건이 아니므로 한다. 즉 대우가 참이므로 주어진 명제도 참이다.
C
QER , 즉 b+1이다.  b=-2

15 ^ 19 18 ^ ⑴ 실수 ⑵ z” ⑶ < ⑷ zz”>0
GUIDE GUIDE
p가 q이기 위한 필요충분조건이면 진리집합 P, Q는 서로 같은 집합이고, ➊ 이차방정식의 근과 계수의 관계를 이용한다.
원소의 합과 곱은 같다. ➋ z가 실수가 아닌 복소수이면 항상 zz”>0이다.

2. 명제 15
z-
a
z
=k (k는 실수, a>0)라 하면 z€-kz-a=0에서 20 ^⑤
GUIDE
z는 x에 대한 이차방정식 x€-kx-a=0의 근이다. 한 사람의 말이 참인 경우 나머지 두 사람의 말은 거짓이므로 서로 모순
이때 k와 -a가 모두 ⑴ 실수 이므로 z가 실수가 아닌 복소수 이 되지 않는 것을 찾는다.

라면 이차방정식의 다른 한 근은 ⑵ z” 이다.
1 승원이 말이 참인 경우, 참인 말은
근과 계수의 관계에서 zz ” ⑶ < 0
승원:나는 ‘서’씨이다.
그런데 z가 실수가 아닌 복소수라면 ⑷ zz”>0 이므로
용현:나는 ‘서’씨이다.
주어진 조건에 모순된다. 따라서 z는 실수이다. 정은:나는 ‘사공’씨이다.

참고 따라서 승원이 말은 참이 아니다.


➊ x€-kx-a=0에서 2 용현이 말이 참인 경우, 참인 말은
D=(-k)€-4_1_(-a)=k€+4a>0 ( a>0)
따라서 이차방정식 x€-kx-a=0은 서로 다른 두 실근을 갖는다.
승원:나는 ‘한’씨 또는 ‘사공’씨이다.

➋ 귀류법, 즉 가정을 부정한 부분이 증명 과정 끝에 있는 ‘z가 실수가 아 용현:나는 ‘한'씨 또는 ‘사공’씨이다.


닌 복소수라면~’에 해당한다. 이 경우 zz”<0인 것에 모순이 생기므 정은:나는 ‘사공’씨이다.
로 z는 실수여야 한다는 것을 보이고 있다.
따라서 용현이 말은 참이 아니다.
3 정은이 말이 참인 경우, 참인 말은

19 ^② 승원:나는 ‘한’씨 또는 ‘사공’씨이다.


GUIDE 용현:나는 ‘서’씨이다.
상품을 하나씩 샀다고 가정하고, 다른 상품의 구매 여부를 판단하여 조건 정은:나는 ‘한’씨 또는 ‘서’씨이다.
에 모순이 없는지 확인한다.
따라서 정은이 말이 참이며, 이때 정은이는 ‘한’씨이며, 승원이
1 만약 신발을 샀다면, 양말도 샀을 것이고 ㈎, 청바지는 사지 않
는 ‘사공’씨라야 한다.
았으며 ㈐, 모자도 사지 않았고 ㈑, 셔츠도 사지 않았다 ㈒. 그
따라서 사공승원, 서용현, 한정은이다.
러면 모자와 셔츠를 사지 않았으므로 ㈏에 어긋난다.
2 만약 양말을 샀다면, 청바지는 사지 않았으며 ㈐, 모자도 사지 다른 풀이
않았고㈑, 셔츠도 사지 않았다 ㈒. 그러면 모자와 셔츠를 사지
세 사람 중 한 사람의 말이 참인 경우에 따라 표를 만들면, 세 사
않았으므로 ㈏에 어긋난다.
람에게 가능한 성씨는 다음과 같다.
3 만약 청바지를 샀다면, 모자를 샀으며 ㈑, 양말을 사지 않았을 가능한 성씨
참인 사람 승원 용현 정은 판정
것이고 ㈐, 신발도 사지 않았을 것이고 ㈎, 셔츠도 사지 않았
승원 서 서 사공 _
다 ㈒. 따라서 청바지와 모자를 샀다.
용현 한, 사공 한, 사공 사공 _
4 만약 모자를 샀다면, 청바지를 샀으며 ㈑, 양말을 사지 않았을
정은 한, 사공 서 한, 서 ◯
것이고 ㈐, 신발도 사지 않았을 것이고 ㈎, 셔츠도 사지 않았다
㈒. 따라서 모자와 청바지를 샀다. 따라서 정은이 말이 참이며, 사공승원, 서용현, 한정은이다.

5 만약 셔츠를 샀다면, 신발과 모자를 샀으며 ㈒, 양말을 샀고㈎,


청바지를 사지 않았을 것이다 ㈐. 그런데 청바지를 사지 않으
면 모자를 살 수 없으므로 ㈑에 어긋난다.
1~5에서 청바지와 모자를 샀다.
STEP 3 1등급 뛰어넘기 p. 27~29

다른 풀이 01 ⑤ 02 ⑤ 03 ③ 04 ⑤
◯:샀음, _:안 샀음
05 ⑴ -;3$;<m<4 ⑵ m>0 06 ② 07 ③
확인
샀다면 신발 양말 청바지 모자 셔츠 모순점
08 ③ 09 ③
신발 ◯ ◯ _ _ _ ㈏
양말 ◯ _ _ _ ㈏ 01 ^⑤
청바지 _ _ ◯ ◯ _ GUIDE
모자 _ _ ◯ ◯ _ 조건 p를 참이 되게 하는 진리집합의 원소를 a라 할 때 a=2q+r이고,

셔츠 ◯ ◯ _ ◯ ◯ ㈑ q=3m+1이다. (단, q, m은 음이 아닌 정수이고, r는 0 또는 1)

16 정답과 풀이
조건 p의 진리집합을 P라 하고 그 원소를 a라 하자. 다른 풀이

a=2q+r, q=3m+1이므로 두 근의 합 k가 자연수이고, 두 근의 부호가 다르므로


(단, q, m은 음이 아닌 정수, r는 0 또는 1) 130=1_130=2_65=5_26=10_13에서
a=2(3m+1)+r=6m+2+r 가능한 k값은 130+(-1)=129, 65+(-2)=63,
 a=6m+2 또는 a=6m+3 ( r는 0 또는 1) 26+(-5)=21, 13+(-10)=3
1 a=6m+2일 때 따라서 n(P)=4
a=2(3m+1)이므로 a는 2의 배수이다.
또 a=6m+2=3_2m+2이므로 3으로 나눈 나머지가 2인
수이다. 즉 a가 속한 집합은 CDD이고,
D=EC이므로 CDD=CDEC 03 ^③

2 a=6m+3일 때 GUIDE
a-b=x, b-c=y라 하면 a-c=x+y이므로
a=2(3m+1)+1이므로 a는 2로 나눈 나머지가 1이다.
주어진 식은 |x|+|y|>|x+y|와 같다.
또 a=6m+3=3(2m+1)이므로 3으로 나누어 떨어진다.
즉 a가 속한 집합은 ADE이고, 모든 실수 x, y에 대하여 |x|+|y|>|x+y|이므로
C
E=D 이므로 ADE=ADD C
|x|+|y|>|x+y|의 부정은 |x|+|y|=|x+y| yy ㉠
1, 2에서 진리집합 P=(ADD )C(CDE )C C
㉠의 양변은 모두 양수이므로 양변을 제곱하면
(|x|+|y|)€=|x+y|€, |xy|=xy  xy>0
따라서 주어진 조건의 부정은 (a-b)(b-c)>0
③에서 (a-b)€+(b-c)€<(a-c)€을 정리하면
02 ^⑤ (b-a)(b-c)<0  (a-b)(b-c)>0
GUIDE
➊ x€-kx-130=0 ⇨ kx=x€-130 ⇨ k=x-
130
( x+0) 참고
x
|x|+|y|>|x+y|가 항상 성립하므로 |x|+|y|>|x+y|의 부정에
➋ k가 자연수이므로 x는 130의 약수여야 한다.
서 |x|+|y|<|x+y|인 경우는 생각하지 않는다.

130
k=x- 에서 다른 풀이
x
x가 정수일 때, k가 자연수이므로 x는 130의 약수여야 한다. ③에서 a-b=x, b-c=y라 하면 a-c=x+y이므로
130=2_5_13의 약수는 1, 2, 5, 10, 13, 26, x€+y€<(x+y)€=x€+y€+2xy에서 0<xy
65, 130 즉 (a-b)(b-c)>0이다.
130
또 k=x- >0이어야 하므로
x
1 x>0일 때
x€-130>0, (x-'ß130 )(x+'ß130 )>0에서 x>12
04 ^⑤
이 조건에 맞는 130의 약수는 13, 26, 65, 130 GUIDE
이때 k값은 차례로 3, 21, 63, 129 명제 ‘어떤 x에 대하여 p이다.’는 어떤 x가 조건 p를 참이 되도록 하므로
2 x<0일 때 ‘조건 p의 진리집합 P의 원소가 존재한다.’와 같은 말이다.

x€-130<0, (x-'ß130 )(x+'ß130 )<0에서 -11<x<0


명제 ‘어떤 x에 대하여 p이다.’가 거짓이므로 P는 공집합이다.
이 조건에 맞는 130의 약수는 -1, -2, -5, -10
① P=이고, C=U이므로 ‘모든 x에 대하여 ~p이다.’는 참
이때 k값은 차례로 129, 63, 21, 3
이다.
1, 2에서 P={3, 21, 63, 129}이므로 n(P)=4
② P=이므로 ‘~p의 진리집합 PC=U이다.’는 참이다.
③ ~p의 진리집합 PC=U이고, 주어진 조건에 따라 U는 공집
참고
합이 아니므로 참이다.
이차방정식 x€-kx-130=0의 두 정수 근 a, b에 대해서
130 130 ④ P=이고, 은 모든 집합의 부분집합이므로 참이다.
ab=-130이고 a+b=k이므로 k=a- =b-
a b ⑤ ‘모든 x에 대하여 p 또는 q이다.’가 참이 되기 위해서는
이때 a>0, b<0이라 하면 양의 정수 a값이 정해질 때 음의 정수 b값도
PCQ=U이어야 한다. P=이므로 Q=U이면 명제는 참
130
결정되므로 k=x- >0에서 x가 양의 정수인 경우만 따져도 된다.
x 이 될 수 있다. 따라서 거짓이다.

2. 명제 17
05 ^ ⑴ -;3$;<m<4 ⑵ m>0 ① A=;3!;x+1, B=|x-2|라 하면 A=-B의 진리집합이 공
GUIDE
집합이므로 p는 q이기 위한 필요충분조건이다.
⑴ 대우인 ‘x€+x-2>0이면 (x€-mx+m)(x€+x-2)>0’인 m값
A=B y A=-B
의 범위를 구한다. y
⑵ 최솟값이 양수가 되게 하는 m값을 찾는다. y=;3!;x+1 y=;3!;x+1
y=|x-2|
O x
⑴ 주어진 명제의 대우는 y=-|x-2|
O x
‘x€+x-2>0이면 (x€-mx+m)(x€+x-2)>0’
즉 ‘x<-2 또는 x>1이면 x€-mx+m>0’ ② A=;3!;x, B=|x-2|-1이라 하면 A=-B의 진리집합이
m
f(x)=x€-mx+m이라 하면 축 x= 에 대하여 공집합이 아니고 그 원소가 A=B의 진리집합에 포함되지 않
2
m 으므로 p는 q이기 위한 충분조건이다.
1 -2< <1, 즉 -4<m<2일 때
2 y A=B y A=-B
y=|x-2|-1
f(-2)=4+3m>0이고 f(1)>0에서 -;3$;<m<2 O
x
O x y=;3!;x
m m y=;3!;x
2 <-2 또는 >1, 즉 m<-4 또는 m>2일 때 y=-|x-2|+1
2 2
D=(-m)€-4m=m(m-4)<0이므로 2<m<4 ③ ‘b>a이면 a>-c’는 ‘b>a이면 a>-c 이고, b>-c’와 같

1, 2에서 m값의 범위는 -;3$;<m<4 고 ‘b<a 이면 b>-c’는 ‘b<a이면 b>-c이고, a>-c’와


같다.
⑵ 주어진 명제의 역은
즉 a, b의 대소에 관계없이 조건 p는 a+c>0이고 b+c>0
‘x€+x-2<0이면 (x€-mx+m)(x€+x-2)<0’
이므로 q:a+c>0 이고 b+c>0와 같다.
즉 ‘-2<x<1이면 x€-mx+m>0’
따라서 p는 q이기 위한 필요충분조건이다.
m
f(x)=x€-mx+m이라 하면 축 x= 에 대하여 ④ z를 곱하면 실수가 되는 복소수를 a라 하면
2
k
m za=k (단, k는 실수) %%,a= ( z+0)
1 -2< <1, 즉 -4<m<2일 때 z
2
a k
D=(-m)€-4m=m(m-4)<0이므로 0<m<2 양변을 z” 로 나누면 = 이고, k와 zz” 가 실수이므로
z” zz”
m
2 <-2일 때 f(-2)>0에서 공통 범위는 없다. a
2 는 실수이다. 즉 p는 q이기 위한 충분조건이다.
z”
m
또 >1일 때 f(1)>0이 항상 성립하므로 m>2 또한 z”로 나누면 실수가 되는 복소수를 b라 하면
2
b
1, 2에서 m값의 범위는 m>0 =m (m은 실수) %%,b=mz”
z”
양변에 z를 곱하면 bz=mzz” 이고, m과 zz” 가 실수이므로
참고
bz는 실수이다. 즉 q는 p이기 위한 충분조건이다.
m
⑴ <-2일 때 -2<x<1에서 f(x)>0 y y=f(x)
2 따라서 p와 q는 필요충분조건이다.
인 것을 오른쪽 그림처럼 생각할 수 있다.
'ßx-2(x-3)(x-5)
m m
;;2;;
⑤ >0은
즉 <-2와 f(-2)>0의 공통 범위를 (x-5)‹
2 x
-2 O
구해야 한다. 이때 f(-2)>0이면 f(1)>0 'ßx-2(x-3)(x-5) 'ßx-2(x-3)(x-5)
=0 또는 >0
(x-5)‹ (x-5)‹
이 당연히 성립하므로 따로 생각하지 않아도 된다.
m 'ßx-2(x-3)(x-5)
※ <-2, 즉 m<-4와 f(-2)=4+3m>0의 공통 범위는 없다. =0일 때 x=5일 수 없으므로
2 (x-5)‹
x=2 또는 x=3, x+5 yy ㉠
'ßx-2(x-3)(x-5)
또한 >0일 때
(x-5)‹
06 ^② 양변에 (x-5)›을 곱하면 'ßx-2 (x-3)(x-5)€>0
GUIDE
이때 x+5에서 (x-5)€>0이므로 x>3 yy ㉡
➊ 조건 p, q의 진리집합을 구하여 포함 관계를 파악한다.
㉠, ㉡에서 x=2 또는 3<x<5 또는 x>5이므로 p는 q이기
➋ 조건 A=-B의 진리집합이 이거나 조건 A=B의 진리집합의 부
분집합이면 A=B와 A€=B€은 필요충분조건 관계이다. 위한 필요충분조건이다.

18 정답과 풀이
다른 풀이 09 ^③

④에서 z=a+bi라 하면 z= GUIDE


” a-bi이고 (단, a, b는 실수)
커피를 좋아하는 사람은 ‘커피’로, 좋아하지 않는 사람은 ‘커피÷ ’로 나타내
z를 곱하면 실수가 되는 복소수를 a, z”로 나누면 실수가 되는 복
고, 우유를 좋아하는 사람과 좋아하지 않는 사람도 ‘우유’와 ‘우유÷ ’로 나
소수를 b라 하자. 타내어 그림을 그려 본다.
b bz
az와 = 가 모두 실수일 때 bz도 실수이므로 두 복소수
z” z”z ㈎ 에서 커피를 좋아하는 사람은 모두 창가에 있으므로 커피를 좋
a와 b의 조건은 같다. 아하는 사람과 좋아하지 않는 사람은 각각 2명이다.
따라서 p와 q는 필요충분조건 관계이다. ㈏ 에서 우유를 좋아하는 사람은 서로 옆자리에 있고, ㈐ 와 ㈓ 에
서 우유를 좋아하지 않는 사람도 있으므로 우유를 좋아하는 사람
과 좋아하지 않는 사람 역시 각각 2명이다.
㈎ 와 ㈏ 에서 다음과 같은 경우가 가능하다.
07 ^③
커피 커피?
GUIDE 우유 우유
조건 p, q, r의 진리집합을 구하여 포함 관계를 파악한다. 창
커피 커피?
우유? 우유?
세 조건 p, q, r의 진리집합을 각각 P, Q, R라 하면
P={(1, 1)}, Q={(a, 1)|a는 실수}, R={(1, 1)}이므로 ㈐, ㈑ 에서 국어 선생님은 다음과 같은 자리에 위치할 수 있다.
P=RAQ, P+Q, R+Q 커피(국어) 커피?
따라서 p는 q이기 위한 충분조건이고, q는 r이기 위한 필요조건 우유 우유

이며, r는 p이기 위한 필요충분조건이다. 커피 커피?
우유? 우유?

㈒ 에서 수학 선생님과 영어 선생님은 서로 대각선에 있으며,


㈓ 에서 수학 선생님은 우유를 좋아하지 않으므로 다음 경우가 가
08 ^③
능하다.
GUIDE
커피(국어) 커피?(영어)
0 이상의 정수 a, b에 대하여 k=2a_3b로 표현할 수 있는 k 중에서
우유 우유
a+b가 짝수이면 p(k)가 참임을 알아낸다. 창
커피(수학) 커피?(과학)
p(1)이 참이므로 p(2)와 p(3)은 거짓이다. 우유? 우유?

p(2)와 p(3)이 거짓이므로 p(2€), p(2_3), p(3€)은 참이다. 그러므로 커피와 우유를 모두 좋아하지 않는 사람은 과학 선생님
위의 규칙을 보면 0 이상의 정수 a, b에 대하여 k=2a_3b로 표 이고, 커피와 우유를 모두 좋아하는 사람은 국어 선생님이다.
현할 수 있는 수 중 a+b가 짝수이면 p(k)가 참임을 알 수 있다.  A-과학, B-국어
먼저, 3b<100이어야 하므로 0b4이다.
1 b=0일 때 2a<100인 짝수 a는 0, 2, 4, 6 참고

100 국어 선생님 자리를 창가 아래쪽이라 생각하면 다음과 같음을 알 수 있다.


2 b=1일 때 2a< 인 홀수 a는 1, 3, 5
3
커피(수학) 커피?(과학)
100
a 우유? 우유?
3 b=2일 때 2 < 인 짝수 a는 0, 2
9 창
커피(국어) 커피?(영어)
100 우유 우유
4 b=3일 때 2a< 인 홀수 a는 1
27
100
5 b=4일 때 2a< 인 짝수 a는 0
81
따라서 진리집합에는 최소한 11개 원소가 포함되어야 한다.

참고

p(2€), p(2_3), p(3€)이 참이므로 p(2‹), p(2€_3), p(2_3€), p(3‹)


은 거짓이다.
p(2‹), p(2€_3), p(2_3€), p(3‹)이 거짓이므로
p(2›), p(2‹_3), p(2€_3€), p(2_3‹), p(3›)은 참이다.

2. 명제 19
03 절대부등식 03
GUIDE
^②

ㄱ. 제곱한 것끼리 빼서 부호를 확인한다.


STEP 1 1등급 준비하기 p. 32 ~34
ㄴ. 차를 이용한다.
01 ④ 02 ④ 03 ② ㄷ. 차에서 식의 변형을 이용한다.
04 ⑴ 12 ⑵ '3 05 2'2 06 ③
07 ⑤ 08 ② 09 ⑴ 14 ⑵ -12 ㄱ. 'a+'b >0, 'ßa+b>0이므로
10 20 ('a+'b )€-('ßa+b )€=(a+b+2'ßab )-(a+b)
=2'ßab >0
01 ^④
따라서 'a+'b >'ßa+b ( ◯ )
GUIDE
4 a+b 4
➊ 부등식의 양변을 음수로 나누면 부등호의 방향이 바뀐다. ㄴ. {;a!;+;b!;}- = -
a+b ab a+b
➋ |b-a|=|a-b|
(a+b)€-4ab
=
ab(a+b)
ㄱ. a<b의 양변을 음수인 c로 나누면 ;cA;>;cB; ( _ )
(a-b)€
= >0
ㄴ. a<b<c이므로 c>b이다. 양변에서 a를 빼면 ab(a+b)

c-a>b-a이고, 이 부등식의 양변이 모두 양수이므로 4


따라서 ;a!;+;b!;> (◯)
a+b
|c-a|>|b-a|=|a-b| ( ◯ )
ㄷ. a+b+c-('ßab+'ßbc+'ßca )
5a+2b a+2b 6a€
ㄷ. - = >0 =('a )€+('b )€+('c )€-('ßab+'ßbc+'ßca )
4a+b 2a+b (4a+b)(2a+b)
a+2b 5a+2b =;2!;{('a-'b )€+('b-'c )€+('c-'a )€}>0
 < (◯)
2a+b 4a+b
따라서 a+b+c>'ßab+'ßbc+'ßca ( _ )

02 ^④ 다른 풀이
GUIDE a+b 2ab
ㄱ. x€+y€+z€-(xy+yz+zx)
ㄴ. (산술평균)>(조화평균), 즉 > 의 양변에
2 a+b
=;2!;{(x-y)€+(y-z)€+(z-x)€}>0 2 a+b 4 4
를 곱하면 > , 즉 ;a!;+;b!;>
ab ab a+b a+b
ㄴ. 2개씩 묶어서 (산술평균)>(기하평균)을 이용한다.

ㄱ. 대우를 확인해 보자. 주어진 명제의 대우는


‘xy+yz+zx=1이면 x€+y€+z€>1이다.’이다. 04 ^ ⑴ 12 ⑵ '3
x€+y€+z€-(xy+yz+zx) GUIDE
➊ 주어진 직선의 x절편, y절편이 각각 양수 a, b임을 이용하여 넓이를
=;2!;{(x-y)€+(y-z)€+(z-x)€}>0
구한다.
이고, xy+yz+zx=1이므로 x€+y€+z€-1>0 ➋ (산술평균)>(기하평균)을 이용하여 a€+b€의 최솟값을 구한다.

따라서 x€+y€+z€>1 ( ◯ )
⑴ 직선과 x축 및 y축으로 둘러싸인 부분의 넓이가 3이므로
b+c c+a a+b
ㄴ. + + =;aB;+;aC;+;bC;+;bA;+;cA;+;cB;
a b c ;2!;ab=3  ab=6

=;aB;+;bA;+;bC;+;cB;+;aC;+;cA; 이때 a€+b€>2"ƒa€b€=2ab=12이므로
a€+b€의 최솟값은 12
이때 ;aB;+;bA;>2, ;cB;+;bC;>2, ;aC;+;cA;>2이므로
⑵ 원점에서 직선 ;aX;+;bY;=1, 즉
b+c c+a a+b
+ + >6 ( ◯ ) bx+ay-ab=0까지 거리를 d 라 하면
a b c
ㄷ. ('a-'b )€=a+b-2'ßab, ('ßa-b )€=a-b이고 |-ab| ab 6
d= = =
"ƒa€+b€ "ƒa€+b€ "ƒa€+b€
a+b-2'ßab-(a-b)=2b-2'ßab 에서
이때 a€+b€>12이므로
a>b>0이므로 2b-2'ßab =2'b ('b-'a )<0 ( ◯ )
6 6
ㄹ. [반례] x=4, y=;2#;이면 xy>x+y>4이지만 d= < ='3
"ƒa€+b€ 'ß 1 2
x>2, y<2이다. ( _ ) 따라서 최댓값은 '3

20 정답과 풀이
05 ^ 2'2 이때 PH’:HA’=BH’:PH’, 즉 PH’:a=b:PH’ 이므로
GUIDE PH’ €= ㈐ ab  PH’="ƒ ㈐ ab
➊ 주어진 직선의 x절편, y절편을 이용하여 삼각형의 넓이를 구한다,
이때 선분 PH의 길이는 반지름 OC의 길이보다 짧거나 같다.
➋ (산술평균)>(기하평균)에서 등호가 성립하는 경우를 생각한다.
(두 점 P, C가 일치할 때 같다.)
a+b
a, b가 양수이고 x절편이 -;aB;, y절편이 b이므로 즉 OC’>PH’ 이므로 ㈎ >"ƒ ㈐ ab
2

;2!;_;aB;_b=4  b€=8a 참고

a+b
OA’+OB’=;aB;+b>2æ;aB;_b=4'2 AB’=a+b이므로 OA’=OB’=OC’=
2

이때 ;aB;=b=2'2에서 OA’+OB’가 최솟값을 가지므로

a=1, b=2'2  ab=2'2


08 ^②
GUIDE
➊ AB’, AC’의 길이는 각각 점 B, C의 x좌표와 같다.
06 ^③ ➋ 곡선 y=x€이 아래로 볼록하므로 AB’<AC’임을 이용한다.
GUIDE
함수 f(x)=x€의 그래프 위에 두 점 P('a, a), Q('b, b)를 잡자.
1 (산술평균)>(기하평균)을 이용하여 x+;x!; 의 범위를 구한다.
이때 D(0, a), E(0, b)의 중점인 y y=x€
2 t=x+;x!; 로 치환하여 식의 최댓값을 구한다. 점 A를 지나고 y축에 수직인 직선이 E Q
PQ’와 만나는 점을 B, 곡선 y=x€과 B
A C
x>0일 때, x+;x!;>2æx_;x!; =2 만나는 점을 C라 하면 AB’의 길이는
D P
점 P, Q의 x좌표의 평균이므로
{등호는 x=;x!;, 즉 x=1일 때 성립} O 'a 'b x
'a+'b
AB’= 이고,
2
t=x+;x!; 이라 하면 t>2이고
a+b
또 점 C의 x좌표를 t라 하면 C {t, }에서
2
y=-t€-2t+11=-(t+1)€+12
a+b a+b
따라서 함수 y는 t=2일 때 최댓값 3 t€=  t=AC’=æ
2 2

'a+'b a+b
이때 AB’<AC’이므로 ㈎ <æ
2 2
07 ^⑤
양변에 2를 곱하면 임의의 실수 a, b (a<b)에 대하여
GUIDE
다음 그림에서 P ㈏ 'a+'b <"ƒ2(a+b) yy ㉠
BAPBPHPAH
㉠에서 a=6, b=12일 때, '6+2'3 ㈐ < 6이다.
 AH’_BH’=PH’ €
A B
H
참고

C a+b
점 A는 DE’의 중점이므로 A{0, } y
2 Q
P E
또 AB’=AF’+FB’
A F
'b 'a B
AB’ = +
2 2
A B D
H O 'a+'b
O
AB’ =
2 'a 'b x
그림처럼 중심이 O인 반원 위의 한 점 P에서 지름 AB에 내린
수선의 발을 H라 하고, AH’=a, BH’=b라 하자.
a+b
이때 반지름인 선분 OC의 길이는 OC’= ㈎
2 09 ^ ⑴ 14 ⑵ -12
삼각형 PAH와 삼각형 BPH에서 PHA=BHP=90^ GUIDE
또 APB=90^이므로 PAH=BPH ⑴ 코시-슈바르츠 부등식, 즉 ( )( )>('a+2'b+3'c )€에서
괄호 안에 들어갈 식은 계수 부분과 문자 부분 각각을 제곱한 것이다.
 PAH ㈏ BPH

3. 절대부등식 21
⑴ ('a+2'b+3'c )€<(1€+2€+3€)(a+b+c)=14€ ㄴ. a€+b€+c€+(ab+bc+ca)
 'a+2'b+3'c <14
=;2!;{(a+b)€+(b+c)€+(c+a)€}>0
'a 'b 'c
{단, 등호는 = = 일 때 성립}
1 2 3  a€+b€+c€+ab+bc+ca>0 ( ◯ )
따라서 'a+2'b+3'c 의 최댓값은 14 16c€+32c+25 16(c+1)€ 9
ㄷ. = +
⑵ (a-2b+3c)€={a+'2 (-'2 b)+'3 ('3 c)}€이므로 18(c+1) 18(c+1) 18(c+1)

코시-슈바르츠 부등식에서 16(c+1) 9


>2Ƙ _
18 18(c+1)
{a+'2 (-'2 b)+'3 ('3 c)}€
<{1€+('2 )€+('3 )€}{a€+(-'2 b)€+('3 c)€} =;3$; ( ◯ )
=6(a€+2b€+3c€)<144
16(c+1) 9
{단, 등호는 = , 즉 c=-;4!;일 때 성립}
 -12<a-2b+3c<12 18 18(c+1)
(단, 등호는 a=-b=c일 때 성립)
다른 풀이
따라서 구하려는 최솟값은 -12
16c€+32c+25
ㄷ. 의 분모와 분자를 c+1로 나누면
18(c+1)

10 ^ 20 ;1¡8; [16(c+1)+
9
c+1
]
GUIDE
(a€+b€+c€)(x€+y€+z€)>(ax+by+cz)€ 꼴 코시-슈바르츠 부등 9
>;1¡8;_2æ16(c+1)_ =;3$;
c+1
식의 등호 조건이 ;aX;=;bY;=;cZ; 임을 이용한다.

02 ^ 4개
코시-슈바르츠 부등식에서
GUIDE
(2a+3b+6c)€<(2€+3€+6€)(a€+b€+c€)=49_4 ㅁ. x>y>z이고, x+y+z=0이면 x>0, z<0임을 이용한다.

 -14<2a+3b+6c<14 {단, 등호는 ;2A;=;3B;=;6C; 일 때 성립}


a b a-b
ㄱ. - = >0 ( a>b>0)
a+1 b+1 (a+1)(b+1)
따라서 최댓값 a=14
a b
  > (◯)
이때 ;2A;=;3B;=;6C;=k라 하면 a=2k, b=3k, c=6k에서 a+1 b+1
ㄴ. a>b>0이면 'b_'b<'a_'b<'a_'a
c€ 36k€
b= = =6  a+b=14+6=20   b<'ßab<a ( ◯ )
ab 6k€

ㄷ. [반례] a=-;2!;, b=;3!;이면 -1<a<b<1이지만

1 1
4= < =9 ( _ )
a€ b€
ㄹ. 0<a<b이면

STEP 2 1등급 굳히기 p. 35~40 b+;a!;-{a+;b!;}=(b-a)+{;a!;-;b!;}

01 ⑤ 02 4개 03 ② 04 ② b-a
=(b-a)+ >0
'6-2 ab
05 ② 06 ⑤ 07 08 0
2
09 36 10 ④ 11 6 12 ④
 a+;b!;<b+;a!; ( ◯ )
13 ② 14 ⑴ 1+2+3 ⑵ x+y+z ㅁ. x>0, y>0, z>0이면 x+y+z>0이다.
15 -3 16 ③ 17 ⑴ ;3!; ⑵ 1 따라서 z<0이어야 한다.
x<0, y<0, z<0이면 x+y+z<0이다.
18 ⑴ p+q+r ⑵ S ⑶ L 19 ⑴ ;1¡3; ⑵ 12 ⑶ ;;™2y;;
따라서 x>0이어야 한다.

01 ^⑤ x+y+z=0에서 y=-x-z이므로
GUIDE ax+by+cz=(a-b)x+(c-b)z
ㄷ. 식을 변형하여 (산술평균)>(기하평균)을 이용한다. 이때 a-b>0, c-b<0이므로
ax+by+cz=(a-b)x+(c-b)z>0 ( ◯ )
ㄱ. a€-ab+b€={a-;2B;}€+;4#; b€>0 ( ◯ )
따라서 참인 것은 ㄱ, ㄴ, ㄹ, ㅁ으로 4개

22 정답과 풀이
03 ^② 05 ^②
GUIDE GUIDE
세 식 b+c=x, c+a=y, a+b=z를 변끼리 더하면
;aB;+;bA; 와 ;bC;+;cB; 끼리 묶어서 (산술평균)>(기하평균)을 적용한다.
2(a+b+c)=x+y+z

1 ;aB;>0일 때 ;aB;+;bA;>2이고, ;bC;>0이라 하면 ;bC;+;cB;>2 b+c=x, c+a=y, a+b=z라 하면

a+b+c={㈎ ;2!;}(x+y+z)에서
이때 ;aB;+;bA;+;bC;+;cB;>4이므로 ;aB;+;bA;+;bC;+;cB;+2
a=;2!;(y+z-x), b=;2!;(z+x-y), c=;2!;(x+y-z)
즉 ;aB;>0이면 ;bC;<0에서 ;aB;_;bC;<0, ;aC;<0
이므로
 ac<0 a b c
+ +
b+c c+a a+b
2 ;aB;<0일 때 ;aB;+;bA;<-2이고, ;bC;<0이라 하면 ;bC;+;cB;<-2
y+z-x z+x-y x+y-z
=;2!; +;2!; +;2!;
x y z
이때 ;aB;+;bA;+;bC;+;cB;<-4이므로 ;aB;+;bA;+;bC;+;cB;+2
y z z x x y
=;2!; { + + + + + }+{㈏ -;2#;}
x x y y z z
즉 ;aB;<0이면 ;bC;>0에서 ;aB;_;bC;<0, ;aC;<0
y x z y x z
>;2!; {2æ _ +2æ _ +2æ _ }+{㈏ -;2#;}
 ac<0 x y y z z x
1, 2에서 ac<0 y x z y x z
=(㈐ 1) {æ _ +æ _ +æ _ }+{㈏ -;2#;}
x y y z z x

다른 풀이 =;2#; (단, 등호는 a=b=c일 때 성립한다.)


주어진 식의 양변에 abc를 곱하면 b€c+ac€+ab€+a€c=2abc
따라서 세 양수 a, b, c에 대하여 주어진 부등식이 성립한다.
(a-b)€
a(b€+c€)=-c(a-b)€에서 -;cA;= >0 ( a+b)
b€+c€ 다른 풀이

따라서 ;cA;<0이고, ac<0 a b c a b c


+ + >3 ‹Æ˜ _ _
b+c a+c a+b b+c a+c a+b

LECTURE
a b c
등호는 = = =k일 때 성립한다.
b+c a+c a+b
(산술평균)>(기하평균)을 두 수 a, b가 음수일 때도 쓸 수 있다.
a=(b+c)k, b=(c+a)k, c=(a+b)k
(-a)+(-b)>2"ƒ(-a)(-b)=2'ßab  a+b<-2'ßab
※부등호 방향에 주의한다. 세 등식을 변변 더하면 a+b+c=2(a+b+c)k  k=;2!;

a b c a b c
즉 + + >3 ‹Æ˜ _ _
b+c a+c a+b b+c a+c a+b
04 ^②
=3 ‹"|k‹=3k=;2#;
GUIDE
3x+y y+3z
y=;2Y;+;2Y; 이므로 ;2#;x+y+;2#;z= + 로 고치면
06
2 2
^⑤
주어진 식을 조금 더 간단하게 할 수 있다.
GUIDE
➊ 곱하면 상수가 되는 짝을 이루도록 식을 정리한다.
3x+y y+3z 1 1
{ + }{ + } ➋ 등호가 성립하는 조건을 확인한다.
2 2 3x+y y+3z
3x+y y+3z
=;2!;+ + +;2!; b€+c€ c€+a€ a€+b€
2(y+3z) 2(3x+y) + +
a€ b€ c€
>1+2æ;4!;=2 b€ c€ c€ a€ a€ b€
= + + + + +
a€ a€ b€ b€ c€ c€

참고 b€ c€ a€ c€ a€ b€
={ + + }+{ + + }
a€ b€ c€ a€ b€ c€
3x+y y+3z
등호는 = , 즉 (3x+y)€=(y+3z)€일 때 성립
2(y+3z) 2(3x+y) b€ c€ a€ c€ a€ b€
>3 ‹Æ˜ _ _ +3 ‹Æ˜ _ _ =3+3=6
한다. 정리하면 3(x-z)(3x+2y+3z)=0에서 a€ b€ c€ a€ b€ c€
x=z 또는 3x+2y+3z=0 b€ c€ a€
{등호는 = = , 즉 a€=b€=c€일 때 성립}
그런데 x>0, y>0, z>0이므로 등호가 성립할 조건은 x=z a€ b€ c€

3. 절대부등식 23
1등급 NOTE 10 ^④

서로 짝이 되는 문자끼리 2개씩 묶거나 이 문제처럼 3개씩 묶어서 곱하 GUIDE


면 상수가 되도록 하는 것이 스킬이다. ab와 cd의 값을 알고 있으므로 abcd의 값을 이용할 수 있도록 부등식을
세운다. 즉 ab+cd>2"ƒabcd

ab=4, cd=9이므로
07 ^
'6-2
2
(a+c)(b+d)=(ab+cd)+(ad+bc)
GUIDE
=13+(ad+bc)
(x€+2x+4)-(x€+4)
"ƒx€+2x+4-"ƒx€+4=
"ƒx€+2x+4+"ƒx€+4 >13+2"ƒabcd
2x =13+2'ß4_9
=
"ƒx€+2x+4+"ƒx€+4
=25 (단, 등호는 ad=bc=6일 때 성립)
주어진 식을 정리하면 따라서 가로 길이가 a+c, 세로 길이가 b+d 인 직사각형 넓이의
'x ("ƒx€+2x+4-"ƒx€+4 ) 'x 최솟값은 25
=
2x "ƒx€+2x+4+"ƒx€+4
1
=
æx+;x$;+2+æx+;x$;
11 ^6
1 '6-2 GUIDE
< =
'6+'4 2 S¡S™를 a, b에 대한 식으로 나타내고 (산술평균)>(기하평균)을 이용한다.

S¡=;2!;_(5-1)_b=2b,
08 ^0
GUIDE
S™=;2!;_(5-2)_a=;2#;a
x, y의 대소는 정해져 있지 않지만 조건에서 x>z, y>z임을 알 수 있다.
따라서 x>y인 경우와 x<y인 경우로 나누어 주어진 식을 더 간단한 부 이때 4=a+2b>2'ßa_2b=2'2'ßab에서 ab<2
등식으로 바꾼다.
(단, 등호는 a=2b=2, 즉 a=2, b=1일 때 성립)

1 x>y일 때 x>y>z이므로 따라서 S¡S™=2b_;2#;a=3ab<6이므로 S¡S™의 최댓값은 6


z-1 x-1 y-1 z-1 x-1 y-1
+ + > + +
x+1 y+1 z+1 x+1 x+1 x+1
x+y+z-3
= >0
12
x+1
^④
( x+y+z>3 ‹'ßxyz =3) GUIDE
2 x<y일 때 y>x>z이므로 밑면의 가로 길이와 세로 길이를 문자로 나타내고 (산술평균)>(기하평균)

z-1 x-1 y-1 z-1 x-1 y-1 을 이용한다.


+ + > + +
x+1 y+1 z+1 y+1 y+1 y+1
그림처럼 책꽂이의 가로 길이를 a cm, 세로 길이를 b cm라 하면
x+y+z-3
= >0
y+1 ab=400
1, 2에서 구하려는 최솟값은 0

옆면 칸막이 칸막이 옆면
20 cm
09 ^ 36 b cm
GUIDE a cm
직육면체의 전개도에서 긴 쪽의 길이가 12를 넘지 않으면 된다.
이때 총 비용은
직육면체 밑면의 한 모서리의 길이를 a, 높이를 b라 하면, 전개도 (20_a+20_b_4)_20+400_40(원)이므로
에서 짧은 쪽은 a+2b만큼, 긴 쪽은 2a+2b만큼의 길이가 필요 (20_a+20_b_4)_20+400_40
하다. 이때 2a+2b<12에서 a+b<6 =400a+1600b+16000>2'ß400a_1600b+16000
직육면체 네 옆면 넓이의 합은 4ab이고, 변의 길이는 양수이므로 =1600'ßab+16000
6>a+b>2'ßab (단, 등호는 a=b=3일 때 성립) =48000
 4ab<36 (단, 등호는 400a=1600b=16000, 즉 a=40, b=10일 때 성립)
따라서 직육면체에서 네 옆면 넓이 합의 최댓값은 36 따라서 비용의 최솟값은 48000원이다.

24 정답과 풀이
13 ^② 주어진 부등식이 항상 성립하려면 ;4#;+k<0  k<-;4#;
GUIDE
(산술평균)>(기하평균)에서
1
+
1
>
2
이므로 따라서 P=-;4#;이고, 4P=4_{-;4#;}=-3
PA’ AQ’ "ƒ PA’_AQ’
PA’_AQ’ 값의 범위를 구하는 방법을 생각한다.
다른 풀이

그림처럼 직선 OA와 원이 만나는 B a€+b€+c€>a+b+c+k, 즉 a€+b€+c€-a-b-c>k에서


두 점을 B, C라 하면 (a€+b€+c€-a-b-c의 최솟값)>k이면 된다.
PA’_AQ’=CA’_AB’ a€+b€+c€-a-b-c
O
이때 OA’=x (0<x<1)라 하면 x
A Q
={a-;2!;}€+{b-;2!;}€+{c-;2!;}€-;4#;>-;4#;
CA’_AB’=(1-x)(1+x)=1-x€ P
C
(산술평균)>(기하평균)에서 즉 a€+b€+c€-a-b-c의 최솟값이 -;4#;이므로 k<-;4#;

1 1 2 2 따라서 P=-;4#;이고, 4P=-3


+ > = >2
PA’ AQ’ "ƒ PA’_AQ’ "ƒ1-x€
(단, 등호는 x=0, 즉 점 A가 중심 O에 있을 때 성립)
따라서 최솟값은 2 16 ^③
GUIDE
➊ 점 P와 세 꼭짓점을 연결하여 만든 삼각형 3개의 넓이 합이 정삼각형
넓이와 같다.
14 ^ ⑴ 1+2+3 ⑵ x+y+z
➋ 절대부등식 (a+b+c)€>3(ab+bc+ca)를 이용한다.
GUIDE
a€ c€
+ >
(a+c)€
이므로 두 항씩 묶어 본다. 정삼각형 한 변의 길이를 l 이라 하면 A
b d b+d
ABC=PAB+PBC+PCA
1
1€ 2€ 3€ (1+2)€ 3€ ( ㈎ 1+2+3 )€ ;2!;l=;2!;lc+;2!;la+;2!;lb c
+ + > + > 이므로 b
x y z x+y z P
㈏ x+y+z
 a+b+c=1 a
B C
4 9 36 이때 (a+b+c)€>3(ab+bc+ca)
;x!;+ + > =4
y z 9
따라서 최솟값은 4 이므로 ab+bc+ca<;3!;

따라서 ab+bc+ca의 최댓값은 ;3!;이다.


다른 풀이

코시-슈바르츠 부등식에서 x, y, z가 양수이므로


LECTURE
1€ 2€ 3€ (a+b+c)€-3(ab+bc+ca)=a€+b€+c€-ab-bc-ca
(x+y+z){ + + }
x y z
=;2!;{(a-b)€+(b-c)€+(c-a)€}>0
1 2 3 €
>{'x_ +'y_ +'z_ } =36
'x 'y 'z  (a+b+c)€>3(ab+bc+ca) (단, 등호는 a=b=c일 때 성립)

즉 9 {;x!;+;y$;+;z(;}>36이므로 ;x!;+;y$;+;z(;의 최솟값은 4

17 ^ ⑴ ;3!; ⑵1

GUIDE
15 ^ -3 A값을 알고 있을 때 A-B의 최댓값을 구하면 B의 최솟값을 얻을 수
GUIDE 있다.
➊ 실수 조건에서 ( )€+( )€+( )€>0임을 이용한다.
➋ a€+b€+c€-a-b-c의 최솟값을 구하는 방법을 써도 된다. 1 1 9x
⑴ - =
3x+1 12x+1 (3x+1)(12x+1)
a€+b€+c€>a+b+c+k에서 a€+b€+c€-a-b-c>k 9x
=
36x€+15x+1
{a-;2!;}€+{b-;2!;}€+{c-;2!;}€>;4#;+k
9
= { ⇦ 36x+;x!;>12}
이때 a, b, c가 실수이므로 36x+;x!;+15

{a-;2!;}€>0, {b-;2!;}€>0, {c-;2!;}€>0 <;2ª7;=;3!;

3. 절대부등식 25
1 1 ⑴ (a€+b€){3€+(-2)€}>(3a-2b)€이므로
따라서 - <;3!; yy ㉠
3x+1 12x+1
a€+b€
>;1¡3;
{단, 등호는 36x=;x!;, 즉 x=;6!;일 때 성립} (3a-2b)€
121 11 € 169 13 €
⑵ ⑴과 같은 방법으로 ⑵ ={ }, ={ } 이므로
4a€ 2a 4b€ 2b
1 1 코시-슈바르츠 부등식에서
- <;3!; yy ㉡
3y+1 12y+1
11 € 13 € 11 13 €
1 1 (a€+b€)[{ } +{ } ]>{a_ +b_ } =12€
- <;3!; yy ㉢ 2a 2b 2a 2b
3z+1 12z+1
11 13
{단, 등호는 = 일 때 성립}
{단, 등호는 y=;6!;, z=;6!;일 때 성립} 2a€ 2b€
121 169 121 169
㉠+㉡+㉢을 하면 즉 + >12€이므로 æ + 의 최솟값은 12
4a€ 4b€ 4a€ 4b€
1 1 1 1 € 1 1 1
3x+1
+
3y+1
+
3z+1 ⑶ (1€+1€)_[{a+ } +{b+ }€]>{a+ +b+ }€
a b a b
1 1 1 1 € 1 1 €
-{
12x+1
+
12y+1
+
12z+1
}<1 2_[{a+ } +{b+ }€]>{1+ }
a b ab
1 1 1
2-{ + + }<1 이때 등호는 a+;a!;=b+;b!;일 때 성립한다.
12x+1 12y+1 12z+1
1 1 1 1
  + + >2-1=1 (a-b){1- }=0에서 a=b ( ab=1이면 a, b는 허수)
12x+1 12y+1 12z+1 ab

즉 a=b=;2!;일 때 성립한다.

18 ^ ⑴ p+q+r ⑵S ⑶L
또 1=a+b>2'ßab 에서
1
>4이므로
GUIDE ab
➊ A, B, C 중 가장 작은 수를 S라 하면 1 € 1 1 €
2_[{a+ } +{b+ }€]>{1+ } >25
A-S>0, B-S>0, C-S>0 a b ab
➋ A, B, C 중 가장 큰 수를 L이라 하면 (단, 등호는 a=b일 때 성립)
A-L<0, B-L<0, C-L<0
1 € 1 25
따라서 {a+ } +{b+ }€의 최솟값은
a b 2
a b c
=m, =n, =l이라 하고
p q r
m, n, l 중 가장 작은 값을 S, 가장 큰 값을 L이라 하면
a+b+c mp+nq+lr
=
p+q+r p+q+r
S( ⑴ p+q+r )+(m-S)p+(n-S)q+(l-S)r
= p+q+r
STEP 3 1등급 뛰어넘기 p. 41~42
(m-S)p+(n-S)q+(l-S)r
=S+ > ⑵ S
p+q+r
01 3 02 1 03 30대 04 'ß10
( m, n, l 중 S가 가장 작다.) 05 8 06 9 07 ⑴ 풀이 참조 ⑵ 풀이 참조
a+b+c mp+nq+lr 08 ⑴ 풀이 참조 ⑵ 풀이 참조
=
p+q+r p+q+r
L( ⑴ p+q+r )+(m-L)p+(n-L)q+(l-L)r 01 ^3
= p+q+r GUIDE

(m-L)p+(n-L)q+(l-L)r a€+b€+c€-ab-bc-ca=;2!;{(a-b)€+(b-c)€+(c-a)€}>0
=L+ < ⑶ L
p+q+r
을 이용한다.
( m, n, l 중 L이 가장 크다.)
(a+b+c)€-3(ab+bc+ca)

=;2!;{(a-b)€+(b-c)€+(c-a)€}>0
19 ^ ⑴ ;1¡3; ⑵ 12 ⑶ ;;™2y;;
즉 (a+b+c)€>3(ab+bc+ca)이므로
GUIDE
코시-슈바르츠 부등식을 이용한다.
(a+b+c)€
>3 ( ab+bc+ca>0)
ab+bc+ca

26 정답과 풀이
02 ^1 "ƒx›+2x‹+x€+1+"ƒx›-4x‹+5x€-2x+1
GUIDE x
a+b b+c c+a 1 2 1
➊{ -1}{ -1}{ -1} =æx€+2x+1+ +æx€-4x+5- +
c a b x€ x x€
a+b-c b+c-a c+a-b
={ }{ }{ } 1 1
c a b =æx€+2x+1+ +æ(x€-4x+4)+{ -;x@;+1}
➋ (a+b-c), (b+c-a), (c+a-b) 모두 양수이고, 어느 두 개를 합 x€ x€
해도 하나의 문자만 남는다. 1 €
=æ(x+1)€+ +Ƙ(x-2)€+{;x!;-1}
x€
a+b-c>0, b+c-a>0이므로 (기하평균)<(산술평균)에서
따라서 세 점 A(-1, 0), B(2, 1), P {x, ;x!;}이라 할 때,
(a+b-c)+(b+c-a)
"ƒ(a+b-c)(b+c-a)< =b
2 주어진 식은 AP’+BP’로 생각할 수 있으므로
마찬가지 방법으로 AP’+BP’>AB’='ß10
"ƒ(b+c-a)(c+a-b)<c, "ƒ(c+a-b)(a+b-c)<a -1+'ß13
{단, 등호는 점 P가 선분 AB 위에 있을 때, 즉 x= 일
이므로 세 부등식을 변변 곱하면 2

(a+b-c)(b+c-a)(c+a-b)<abc 때 성립한다.}

양변을 abc로 나누면


참고
a+b b+c c+a
{ -1}{ -1}{ -1}<1
c a b 등호가 성립하는 때는 C {x, ;x!;}가 직선 AB, 즉 y=;3!;(x+1) 위에 있을
따라서 주어진 부등식이 항상 성립하도록 하는 k의 최솟값은 1 -1+'ß13
때이므로 ;x!;=;3!;(x+1), x€+x-3=0, x>0이므로 x=
2

03 ^ 30대
GUIDE
➊ 차가 45초 동안 달린 거리에 차간거리가 늘어선다고 생각한다.
05 ^8
➋ (산술평균)>(기하평균)을 이용한다. GUIDE
x‹-3x€+8x-4 r
다음과 같이 생각할 수 있다. =(x-p)€+q(x-1)+ +s 꼴로 나타내고
x-1 x-1
45v (산술평균)>(기하평균)을 이용한다.
å y
f(v) f(v) f(v) f(v) f(v) f(v) x‹-3x€+8x-4=(x-1)(x€-2x+6)+2이므로

45초 동안 차들이 이동한 거리를 차간거리로 나누면 차는 모두 x‹-3x€+8x-4 2


=x€-2x+6+
x-1 x-1
45v
(대) 2
;2¡0;v€+;2!;v+5 =(x€-4x+4)+2(x-1)+ +4
x-1
45 2
분자와 분모를 v로 나누면 =(x-2)€+[2(x-1)+ ]+4
5 x-1
;2¡0;v+;2!;+
v
(x-2)€은 x=2일 때 최솟값이 0이고,
분모가 최소일 때 식의 값이 최대가 되므로
2 2
5 5 2(x-1)+ 도 2(x-1)= , (x-1)€=1, 즉
;2¡0;v+;2!;+ >2æ;2¡0;v_ +;2!;=;2#; x-1 x-1
v v
x=2일 때 최솟값이 4이므로
45v 45
따라서 < =30이므로 최대 30(대) x‹-3x€+8x-4
5 의 최솟값은 x=2일 때 0+4+4=8
;2¡0;v+;2!;+ ;2#; x-1
v

1등급 NOTE
주의
2개 이상의 최솟값(최댓값)을 통해 최솟값(최댓값)을 구할 때는 조건이
맨 끝에 있는 차는 통과하지 못한 것으로 생각한다.
일치하는지 확인해야 한다. 주어진 식을
2 2 19
x€-3x+(x-1)+ +7={x-;2#;}€+[(x-1)+ ]+
x-1 x-1 4
04 ^ 'ß10
로 고치고 최솟값을 0+2'2+
19
라고 하면 안 된다.
GUIDE 4
2
➊ 근호 안의 식을 x€으로 나누어 ( )€+( )€ 꼴로 만든다. {x-;2#;}€는 x=;2#;일 때 최소이고, [(x-1)+ ]는 x=1+'2일 때
x-1
➋ 두 점 사이의 거리 공식을 이용한다.
최소이다.

3. 절대부등식 27
06
a1 a a
^9 + 2 +y+ n
1€ 2€ n€
GUIDE 1 1 1
(p+q)+(q+r)+(r+q) >2 {;1!;+;2!;+y+;n!;}-{ + +y+ }
a¡ a™ an
p+q+r= 이다. 이때 ;aB;={Æ;aB; }€이라 생각
2
>2 {;1!;+;2!;+y+;n!;}-{;1!;+;2!;+y+;n!;}=;1!;+;2!;+y+;n!;
d c
하면 코시-슈바르츠 부등식에서 {;aB;+ } {;bA;+ }>(1+1)€
c d

2 2 2
(p+q+r){ + + }
p+q q+r r+p

={
p+q
+
q+r
+
r+p
}{
2
+
2
+
2
} 08 ^ ⑴ 풀이 참조 ⑵ 풀이 참조
2 2 2 p+q q+r r+p GUIDE
>(1+1+1)€=9 n-2
정 n각형의 한 내각 크기는 _180^
n
2 2 2
{단, 등호는 = = , 즉 p=q=r일 때 성립}
p+q q+r r+p
n-2
⑴ 정 n각형의 한 내각 크기는 _180^이므로
2 2 2 n
즉 (p+q+r){ + + }>9에서
p+q q+r r+p
2 2 2 9
+ + > 와
p+q q+r r+p p+q+r
x-2
2 2 2 k x _180^
+ + > 를 비교하면 z-2
p+q q+r r+p p+q+r z _180^
y-2
y _180^
k의 최댓값은 9

07
x-2 y-2 z-2
^ ⑴ 풀이 참조 ⑵ 풀이 참조 _180^+ _180^+ _180^=360^
x y z
GUIDE
서로 다른 n개의 자연수에서 1<a¡<a™<a£<y<an이면 3-2 {;x!;+;y!;+;z!;}=2
1 1 1 1 1
<1, <;2!;, <;3!;, y, < 이다.
a¡ a™ a£ an n
  ;x!;+;y!;+;z!;=;2!;
ak 1
⑴ >0, >0이므로 (산술평균)>(기하평균)에서 ⑵ x=y=z=6이면 ;6!;+;6!;+;6!;=;2!;
k€ ak
ak 1 a 1 2
+ >2æ k _ = x<6, y<6, z<6이면 ;x!;+;y!;+;z!;>;2!;이므로
k€ ak k€ ak k
⑵ ⑴의 결과에서 z>6이어야 하고
a a a 1 1 1
{ 1 + 2 +y+ n }+{ + +y+ } x>6, y>6, z>6이면 ;x!;+;y!;+;z!;<;2!;이므로
1€ 2€ n€ a1 a2 an
1 x<6이어야 한다.
>2{;1!;+;2!;+y+ }
n
따라서 x<6<z
또 서로 같지 않은 자연수이므로
1<a¡<a™<a£<y<an이라 하면 참고

1 1 1 1 1 ⑵에서 ;x!;+;y!;+;z!;>;2!;이고, ;6!;+;6!;+;6!;=;2!; 이므로 x, y, z 중


<1, <;2!;, <;3!;, y, < 이므로
a1 a2 a3 an n
a1 a a 가장 큰 수인 z가 6 이상이 아니면 ;x!;+;y!;+;z!;>;2!;일 수밖에 없다.
+ 2 +y+ n
1€ 2€ n€
마찬가지로 가장 작은 수인 x가 6 이하가 아니면 ;x!;+;y!;+;z!;<;2!;일 수
1 1 1 1
>2{;1!;+;2!;+y+ }-{ + +y+ }
n a1 a2 an 밖에 없다.

1
>;1!;+;2!;+y+
n 채점 기준 배점
1
⑴ ;x!;+;y!;+ =;2!;임을 보이기 30%
z
참고
⑵ x<6<z임을 보이기 70%
a1 a a 1 1 1
{ + 2 +y+ n }+{ + +y+ }
1€ 2€ n€ a¡ a™ an

>2 {;1!;+;2!;+y+;n!;}에서

28 정답과 풀이
04 함수 03
GUIDE
^ 12

f 가 항등함수임을 이용해 f(3), g(2), h(1)의 값을 구한다.


STEP 1 1등급 준비하기 p. 46 ~47

01 ⑤ 02 ③ 03 12 04 ④ f 는 항등함수이므로 f(1)=1, f(2)=2, f(3)=3, f(4)=4


05 ③ 06 ② 07 13 08 ③ ㈏ 에서 f(3)=3이므로 g(2)=h(1)=3
09 ① 10 5개 이때 g는 상수함수이므로 g(1)=g(3)=g(4)=3
㈐ f(2)=2이므로 h(4)<2<h(3)에서 h(4)=1이고
01 ^⑤
또 ㈏ 에서 h(1)=3이므로 h(3)=4, h(2)=2
GUIDE
함수 f(x)의 치역을 구해 a값을 먼저 구한다.
 f(1)_g(2)_h(3)=1_3_4=12

f(1)=2, f(2)=3, f(3)=6이므로 a=6이다.


g(1)=2+b, g(2)=5+b, g(3)=8+b이므로 b=-2이다.
04 ^④
따라서 a+b=4
GUIDE
일대일함수와 일대일대응의 개수를 구할 때는 한 줄로 나열하는 경우를
참고
생각한다.
2+b, 5+b, 8+b는 차례로 3씩 커지는 수이고,
집합 Y의 원소 중 차례로 3씩 커지는 수는 0, 3, 6이므로 ⑴ 정의역의 원소 1, 2, 3이 공역의 원소 1, 2, 3 중에 자신과 같은
2+b=0  b=-2 수에 대응하는 경우는 한 가지뿐이므로 a=1
⑵ 정의역의 모든 원소가 공역의 네 원소 중 한 원소에만 대응해
야 하므로 b=4
02 ^③ ⑶ 정의역의 원소가 서로 다른 공역의 원소에 대응해야 한다.
GUIDE 4가 공역의 다섯 원소 중 하나에 대응하면, 5는 4가 대응한 원
좌표평면 위에 정의역과 공역이 함께 보이도록 나타내고, 함수의 그래프
소를 제외한 나머지 네 원소 중 하나에 대응해야 하고, 6은 4,
를 어떻게 그려야 할지 따져 본다.
5가 대응한 원소를 제외한 원소에, 7은 4, 5, 6이 대응한 원소
실수 전체의 집합에서 함수 y 를 제외한 원소에 대응해야 하므로
13
f(x)=m(x+2)+1의 그래프는 c=5_4_3_2=120
항상 (-2, 1)을 지나므로 ⑷ ⑶과 마찬가지 방법으로 d=4_3_2_1=24
정의역 X={x|1x4}와 ⑸ 공역의 원소 중 세 개를 뽑아 크기 순서대로 정의역의 원소에
공역 Y={y|2y13}에서 함수 5_4_3
대응시키면 되므로 e= =10
가 되려면 그래프가 오른쪽 그림과 1_2_3
(-2, 1) 2 ∴ a+b+c+d+e=159
같아야 한다.
기울기 m의 최솟값은 (1, 2)를 O 1 4 x
참고
지날 때 ;3!;이고, 최댓값은 (4, 13)을 지날 때 2이다. ⑸와 같은 함수를 증가함수라 한다.
A에서 C로의 함수가 증가함수이려면 C의 원소 중에서 순서를 생각하지
따라서 ;3!;m2이므로 이 범위에 있는 정수는 2개 않고 3개를 뽑아 작은 것부터 f(1), f(2), f(3)으로 정의하면 된다. 즉 5
개 중 임의로 3개를 뽑는 경우의 수를 구하면 되므로 중학교에서 배운
‘대표 뽑기’를 이용해 계산한다.
1등급 NOTE

정의역에서 주어진 범위와 공역에서 주어진 범위를 이용해 만든 직사각


형에서 함수의 그래프는 직사각형의 가로변은 통과하지 않는다. 정의역

05
의 모든 원소가 공역의 원소에 대응해야 하기 때문이다.
^③
가로변을 통과하는 경우는 정의역에서 빠지는 부분이 있으므로 함수가
GUIDE
아니다.
af(a)에서 a값이 커질수록 f(a)값이 커질 때 최댓값, a값이 커질수
록 f(a)값이 작아질 때 최솟값이 된다.

최댓값 ⇨ 1_1+2_2+3_3+4_4+5_5=55
최솟값 ⇨ 1_5+2_4+3_3+4_2+5_1=35
(◯) (_) (_) (_) 따라서 최댓값과 최솟값의 차는 20

4. 함수 29
06 ^② y y=f(x)
y=x
GUIDE
y=f-1(x)
실수 전체 집합을 정의역으로 하는 함수가 일대일대응이 되려면 치역도 3
실수 전체 집합이어야 한다.

1
f(x)=m(x+2)+|x-1|+1
O 1 3 x
(m+1)x+2m (x>1)
=[
(m-1)x+2m+2 (x<1)
이때 두 직선의 기울기가 같은 부호여야 치역이 실수 전체 집합 즉 이차방정식 ;2!;x€-x+;2#;=x의 두 근이 교점의 x좌표이므로

이 되므로 (m+1)(m-1)>0  m<-1 또는 m>1 (x-1)(x-3)=0에서 x=1 또는 x=3


따라서 두 교점 (1, 1), (3, 3) 사이의 거리는 2'2
참고

주어진 함수 f(x)가 다음과 같은 꼴이면 일대일대응이 아니다. 1등급 NOTE


y y y y=f(x) ➊ 증가하는 함수 f 와 그 역함수 f -1의 그래프의 교점은 모두 직선
y=f(x)
y=x 위에 있고 교점은 보통 1개 또는 2개이다.
➋ 함수 f 가 감소하는 함수이면 f 와 f -1의 교점은 1개, 3개, 5개, ……
O x O x
O x 이고, 이 중에서 직선 y=x 위에 있는 교점은 1개이다.
y=f(x) 따라서 교점이 1개이면 그 교점
y= -x+1 y
은 y=x 위에 있고, 3개이면 직 A
선 y=x 밖에도 두 교점이 있다. 1 B

오른쪽 그림은 x>0에서 감소하


07 ^ 13
는 함수 f -1(x)=-x€+1과 그 1
C
x
O
GUIDE
역함수 f -1(x)='ß-x+1의 교
다항식 f(x)를 x-a로 나눈 나머지는 f(a)임을 이용한다.
점이 3개임을 보여 준다. 이때 교 y=-x€+1 (x>0)
점 2개는 직선 y=x 밖에 있다.
( f@f@f )(x)를 x+1로 나눈 나머지는 x=-1을 대입한
( f@f@f )(-1)=( f@ f )(1)=f(3)=13

10 ^ 5개

08 ^③ GUIDE
GUIDE 참인지 의심되는 경우이면 반례를 찾아본다.
함수 f(x)의 역함수가 존재하려면 일대일대응이어야 한다.
ㅁ. ( f@g-1)-1=g@ f -1 (_)
즉 함수 f(x)의 치역과 공역이 서로 같아야 한다.
ㅂ. [반례] f(1)=2, f(2)=1이면 역함수도 같은 두 점을 지나
f(a)=2a-1=3에서 a=2이고 므로 교점이 직선 y=x 위에 있지 않은 경우도 있다. (_)
f(5)=2_5-1=b에서 b=9이므로 ab=18 따라서 참인 것은 ㄱ, ㄴ, ㄷ, ㄹ, ㅅ으로 5개이다.

(5, b)
참고 참고
f(x)=2x-1의 역함수가 존재하는 경우를 ㅂ. f(x)=-x+k(k는 상수)이면 f=f -1 로 일치하므로 두 함수
y=f(x)
따질 때는 오른쪽 그림과 같이 직사각형을 그려 f, f -1의 그래프의 교점이 직선 y=x 밖에 있는 경우가 무수히 많다.
놓고 일대일대응이 되도록 해야 한다. (a, 3) ㅅ. 대응관계를 그려서 생각해 보면 참임을 알 수 있다.

09 ^①
GUIDE
y=f(x)가 주어진 범위에서 함숫값이 증가하는 함수이므로 y=f(x)의 STEP 2 1등급 굳히기 p. 48~52
그래프와 그 역함수 y=f -1(x)의 그래프가 만나는 점은 직선 y=x 위
01 ⑤ 02 ④ 03 ⑤ 04 ⑤
에 있다.
05 ④ 06 ⑤ 07 12개 08 ④
09 ⑴ 0 ⑵ 2 ⑶ -2 10 ③, ⑤ 11 ③
함수 f(x)=;2!;x€-x+;2#;=;2!;(x-1)€+1은 x>1인 범위에서
12 ⑤ 13 ③ 14 213 15 ②
증가하는 함수이므로 함수 y=f(x)와 그 역함수의 그래프의 교 16 ⑤ 17 ④ 18 ③ 19 ⑤
20 98 21 ④ 22 ②
점은 y=f(x)의 그래프와 직선 y=x의 교점과 같다.

30 정답과 풀이
01 ^⑤ f(2), f(4)의 값은 각각 1~5 중에서 하나이면 되므로 그 경우
GUIDE 의 수는 5_5
주어진 정의역에서 함수 f(x)의 최솟값과 최댓값을 구해 본다. f(1), f(3), f(5)의 값은 각각 2 또는 4 중에서 하나이면 되므로

f(x)=a(x-2)€+b-4a에서 그래프의 대칭축이 x=2이므로 그 경우의 수는 2_2_2

f(x)의 최댓값과 최솟값은 다음과 같이 생각할 수 있다. 따라서 조건에 맞는 함수 f 는 모두 5€_2‹=200(개)


(5, 7) (2, 7)

05 ^④
GUIDE
a>0 a<0 1 f(-x)=-f(x)이면 함수 f(x)의 그래프는 원점에 대하여 대칭이
다. 따라서 그래프가 원점을 지나고 f(0)=0이다.
2 치역과 공역이 같은 경우를 제외한다.
(5, -2)
(2, -2) 1 f(-x)값이 정해지면 f(x)의 값은 f(x)값과 부호가 반대인
1 a>0일 때 f(2)=b-4a=-2, f(5)=5a+b=7 값으로 정해진다.
 a=1, b=2 f(0)의 값은 0으로 정해져 있고, f(-3), f(-2), f(-1)의
2 a<0일 때 f(2)=b-4a=7, f(5)=5a+b=-2 값에 따라 f(1), f(2), f(3)의 값이 각각 결정되므로
 a=-1, b=3 f(-3), f(-2), f(-1)의 값을 정하는 경우만 생각하면 된
따라서 a+b의 값은 3 또는 2이므로 합은 5 다. 이때 각각 7가지씩 가능하므로 f(-x)=-f(x)가 되는
전체 경우의 수는 7‹=343
02 ^④ 2 치역과 공역이 같은 경우
GUIDE f(-1)의 값은 -3, -2, -1, 1, 2, 3 중 하나,
두 함수 f, g의 함숫값을 구해 비교한다.
f(-2)의 값은 f(0), f(1), f(-1)과 다른 네 값 중 하나,
두 함수가 같은 함수이므로 cGX일 때 f(c)=g(c)
f(-3)의 값은 f(0), f(1), f(-1), f(2), f(-2)와 다른
두 함수 f 와 g가 서로 같으므로 두 값 중 하나를 택하는 것이므로 경우의 수는 6_4_2=48
f(0)=g(0), f(1)=g(1), f(2)=g(2) 1, 2에서 조건에 맞는 함수 f 는 모두 343-48=295(개)
즉 f(0)=3, f(1)=1, f(2)=3
LECTURE
g(0)=a+b=3, g(1)=b=1, g(2)=a+b=3에서
➊ f(-x)=f(x)인 함수 (우함수)
a=2, b=1
f(x)=x€일 때, 함수 y=f(x)의 그래프는 y y=x€
따라서 2a-b=3
오른쪽 그림처럼 y축에 대하여 대칭이다.
이와 같이 어떤 함수 y=f(x)의 그래프가
a€
03 ^⑤ y축에 대하여 대칭일 때, 이 함수를 우함수
GUIDE 라 한다. 우함수 f 는 f(-x)=f(x)가 성 -a O a x
일대일대응은 공역과 치역이 같아야 하므로 함수의 그래프는 정의역에 립한다. f(x)=a 꼴 상수함수와
서 주어진 경계와 공역에서 주어진 경계가 만나는 점에서 시작한다. f(x)=ax€ 꼴 이차함수가 대표적인 우함수이다.
➋ f(-x)=-f(x)인 함수 (기함수)

f(x)=-{x-;2#;}€+:™4ª:에서 그래프의 축이 x=;2#;이다. f(x)=2x일 때, 함수 y=f(x)의 그래프는 y y=2x


오른쪽 그림처럼 원점에 대하여 대칭이다. 이 2a
또 오른쪽 그림과 같이 와 같이 어떤 함수 y=f(x)의 그래프가 원점
(a, 2a-1) -a
f(a)=2a-1이어야 하므로 에 대하여 대칭일 때, 이 함수를 기함수라 한 Oa x

-a€+3a+5=2a-1 다. 기함수 f 에서 f(-x)=-f(x)가 성립 -2a


한다. f(x)=ax 꼴 일차함수와 f(x)=ax‹
0=a€-a-6
y=-x2+3x+5 꼴 삼차함수가 대표적인 기함수이다.
(a+2)(a-3)=0 x=;2#;
 a=-2 또는 a=3
이때 a=-2인 경우에는 일대일대응이 아니므로 a=3 06 ^⑤
GUIDE
aGA일 때 3a-2-f(a)GB가 되는 경우를 따져 본다.
04 ^⑤
GUIDE x=1이면 1-f(1)GB이므로 f(1)의 값은 0 또는 1 중 하나이
a가 짝수이면 af(a)는 항상 짝수이고, a가 홀수이면 f(a)가 짝수이다.
다. (∵ 0<1-f(1)<5)

4. 함수 31
x=2이면 4-f(2)GB이므로 f(2)의 값은 0~4 중 하나이다. 10 ^ ③, ⑤
(∵ 0<4-f(2)<5) GUIDE
x=3이면 7-f(3)GB이므로 f(3)의 값은 2~5 중 하나이다. f( f(x))=f(ax+b)=a(ax+b)+b=a€x+ab+b

(∵ 0<7-f(3)<5)
f( f(x))=x에서 a€x+ab+b=x
x=4이면 10-f(4)GB이므로 f(4)=5
이때 a=1이면 b=0이고, a=-1이면 모든 경우에서 성립
따라서 조건에 맞는 함수 f 는 모두 2_5_4_1=40(개)
따라서 a=1, b=0 또는 a=-1

07
다른 풀이
^ 12개
GUIDE 역함수가 존재하는 함수 f 에서 f( f(x))=x가 성립하므로
‘x¡+x™이면 f(x¡)+f(x™)’인 함수 f 는 일대일함수이다. ( f -1@f@f )(x)= f -1(x)
 f(x)=f -1(x)
㈎는 일대일함수를 의미하고, ㈏에서 치역의 원소의 합이 8인 경
우를 생각하면 치역은 {1, 2, 5} 또는 {1, 3, 4}가 가능하다. 즉 ax+b=;a!;x-;aB;임을 이용한다.
이때 정의역의 세 원소 1, 2, 3이 치역의 세 원소에 각각 대응하
는 경우를 생각하면 조건에 맞는 함수 f 는 모두 1등급 NOTE
2_(3_2_1)=12(개) 일차함수 중 f(x)=f -1(x)인 경우는 f(x)=x 또는 f(x)=-x+c
꼴이다.
LECTURE 함수 f 가 일대일함수임을 나타내는 표현
➊ x¡+x™이면 f(x¡)+ f(x™)
➋ f(x¡)=f(x™)이면 x¡=x™ (➊의 대우)

11 ^③
GUIDE
08 ^④
좌표평면 위에 y=f(x)와 y=f(x)-1의 그래프를 나타내고, 이를 이용
GUIDE
해 y=( f@f )(x)의 그래프를 그린다.
치환을 이용해 f(x)를 구해도 된다.

f(x)=|x-1|-1의 그래프는 다음과 같다.


x-1
ㄱ. f { }=2x+1에 x=7을 대입하면 f(3)=15 ( ◯ )
2 y f(x)=|x-1|-1

x-1
ㄴ. t= 로 두면 x=2t+1이므로
2
1
f(t)=2(2t+1)+1=4t+3 f(x)=4x+3 ( _ ) O x
-1
ㄷ. f(x)=4x+3에 x 대신 2x+1을 대입하면
f(2x+1)=4(2x+1)+3=8x+7 ( ◯ )
y=( f@f )(x)=f(f(x))=| f(x)-1|-1이므로 y=f(x)의
참고 그래프와 y=f(x)-1의 그래프에서 다음과 같이
x-1 y=( f@f )(x)의 그래프를 그릴 수 있다.
치환하지 않고 f { }=2x+1에서 x 대신 2x+1을 대입해도 된다.
2
y y=f(x)-1

1 3
-1
09 O x
^⑴0 ⑵ 2 ⑶ -2
-1
GUIDE -2
f(xy)=f(x)+f(y)와 같은 함수방정식이 주어지면 x=y=1처럼 간
단한 수부터 대입하며 주어진 함숫값을 하나씩 찾는다. y y=f(f(x))=|f(x)-1|-1

1
f(xy)=f(x)+f(y)에 -1 2 3 4
⑴ x=y=1을 대입하면 f(1)=f(1)+f(1)  f(1)=0 -2 O 1 x
-1
⑵ x=y=2를 대입하면 f(4)=f(2)+f(2)=1+1=2

⑶ x=4, y=;4!;을 대입하면 f(1)=f(4)+f {;4!;}에서 또 y=( f@f @f )(x)=|f( f(x))-1|-1이므로 다음과 같이
y=f( f(x))-1의 그래프에서 y=( f@f @f )(x)의 그래프를
0=2+f {;4!;}  f {;4!;}=-2 그릴 수 있다.

32 정답과 풀이
y
y=f(f(x))-1 14 ^ 213
GUIDE
-1 1 3 5
-3 O x 반복되는 함숫값의 규칙성을 찾는다.

-2 h(abc)=f(g(abc))=f(cab)=bac이므로
h–(123)=h(123)=213
y
y=f(f(f(x)))=|f(f(x))-1|-1 h€(123)=h(h(123))=h(213)=123
1 h‹(123)=h(h€(123))=h(123)=213
-2 O1 4 5 6
-4 -1 2 3 x 따라서 h1(123)=h3(123)=h5(123)=y=h999(123)이므로
-1
h999(123)=h1(123)=213

참고

➊ y=f(x)-1의 그래프에서 y<0인 부분을 x축에 대하여 대칭이동한


것과 y>0인 부분을 함께 나타낸 것이 y=| f(x)-1|의 그래프이다.
15 ^②
GUIDE
이때 y=| f(x)-1|의 그래프를 y축 방향으로 -1만큼 평행이동한
f n(x)=f –(x)가 되는 n값을 찾는다.
것이 y=| f(x)-1|-1의 그래프이다.
➋ y=| f( f(x)-1|의 그래프도 ➊과 같은 방법으로 그린다. 주어진 함수의 정의에 따라 대응 관계를 나타내면 그림과 같다.
f f f f
A A A A A
1 1 1 1 1
12 ^⑤ 2 2 2 2 2
GUIDE 3 3 3 3 3
g( f(k))의 값을 찾을 때 x>0, x<0으로 나누지 말고 f(k)>0인 경우 4 4 4 4 4
와 f(k)<0인 경우로 나누어 생각한다.
f ‹(1)=4, f ‹(2)=1, f ‹(3)=2, f ‹(4)=3,
1 f(k)>0일 때 g( f(k))=-{ f(k)}€+5=4에서 f ›(1)=1, f ›(2)=2, f ›(3)=3, f ›(4)=4
{ f(k)}€=1  f(k)=1 즉 f ›(x)=x이므로
따라서 f(k)=|k|-5=1에서 k=6 또는 k=-6 f 999(2)=f 4_249+3(2)=f ‹(2)=1
2 f(k)<0일 때 g( f(k))={ f(k)}€-5=4에서 f 1000(3)=f 4_250(3)=f ›(3)=3
{ f(k)}€=9  f(k)=-3 따라서 f 999(2)+f 1000(3)=1+3=4
따라서 f(k)=|k|-5=-3에서 k=2 또는 k=-2
1, 2에서 조건에 맞는 모든 실수 k값들의 곱은

16
6_(-6)_2_(-2)=144
^⑤
GUIDE
f –(54), f €(54), f ‹(54), …의 값을 차례로 구해 f n(54)=f –(54)가 되

13 ^③
는 n값을 찾는다.

GUIDE
f –(54)=f(54)=18, f €(54)=f( f(54))=f(18)=6
x가 유리수이면 1-x는 유리수이고, x가 무리수이면 1-x는 무리수이
다. 즉 x가 유리수일 때와 무리수일 때로 경우를 나눈다. ……
f °(54)=f( f ‡(54))=f(24)=8
1 x가 유리수일 때 1-x도 유리수이므로 f ·(54)=f( f °(54))=f(8)=18
f(x)+f(1-x)+f( f(x))=x+(1-x)+f(x) 즉 f ·(54)=f –(54)이므로
=x+(1-x)+x f 1001(54)=f 8_125+1(54)=f 1(54)=18
=x+1
2 x가 무리수일 때 1-x도 무리수이므로

17
f(x)+f(1-x)+f( f(x))
^④
=1-x+1-(1-x)+f(1-x) GUIDE
=1-x+1-1+x+1-(1-x) |x-1|=1이 되는 x 값이 0 또는 2이므로 ( f@ f@ f )(x)=1에서
=x+1 ( f@ f )(x)=0 또는 2
( f@ f )(x)=0이면 f(x)=1, ( f@ f )(x)=2이면 f(x)=3임을 이용한다.
1, 2에서 f(x)+f(1-x)+( f@ f )(x)=x+1

4. 함수 33
( f@ f@ f )(x)=1에서 ( f@ f )(x)=0 또는 2 따라서 네 가지 경우 모두 0<{2f(x)-1}€<1이므로
이때 |x-1|=0이면 x=1이므로 y=( f@f )(x)=(2f(x)-1)€의 그래프는 [그림 3]과 같다.
( f@ f )(x)=0이면 f(x)=1에서 x=0 또는 2 이때 ( f@ f )(x)=x의 실근은 [그림 4]에서 y=( f@ f )(x)의 그
또 |x-1|=2에서 x=-1 또는 x=3이므로 래프와 직선 y=x가 만나는 점의 x좌표이다.
( f@ f )(x)=2이면 f(x)=3 ( f(x)>0)에서 y y
x=-2 또는 x=4
1 1
따라서 a=4, b=0+2+(-2)+4=4이므로 a+b=8

다른 풀이

y=( f@ f )(x), 즉 y=| f(x)-1|의 y=f(x) O a b 1 x O a b 1 x


;2!; ;2!;
그래프는 y=f(x)의 그래프를 y축 방 [그림 3] [그림 4]
향으로 -1만큼 평행이동하고 y<0인 따라서 방정식 f( f(x))=x의 실근은 4개
1 x
부분을 x축에 대하여 대칭이동한 것이다.

y=f(f(x))=|f(x)-1|

y=1
19 ^⑤
GUIDE
0 2 x 함수 y=f(x)는 증가하는 함수이므로 y=f(x)의 그래프와 y=x의 그
래프로 둘러싸인 부분의 넓이를 구하여 2배 한다.
y=( f@f@f )(x), 즉 y=| f( f(x))-1|의 그래프는
t=f( f(x))의 그래프를 y축 방향으로 -1만큼 평행이동하고 y=f(x)의 그래프와 직선 y=x의 y
y=f(x) y=x
y<0인 부분을 x축에 대하여 대칭이동한 것이다. 4
두 교점은 (0, 0), {;2%;, ;2%;}이고,
y=f(f(f(x)))=|f(f(x))-1| ;2%; y=f -1(x)
이 두 점을 이은 선분을 삼각형
y=1 1
의 밑변으로 생각하면 점 (2, 1)
-2 -1 0 1 2 3 4 x 과 직선 y=x, 즉 x-y=0 사 O 2 3 x
;2%;
이의 거리가 높이이므로 높이는
위 그림에서 ( f@f@f )(x)=1의 해 -2, 0, 2, 4를 찾을 수 있다.
|2-1| '2
=
"ƒ1€+(-1)€ 2
'2
18 ^③ 따라서 S=2_{;2!;_;2%;'2_
2
}=;2%; 이므로 100S=250
GUIDE
y=f( f(x)), 즉 y=(2f(x)-1)€의 그래프를 그려 본다. 1등급 NOTE

세 꼭짓점이 (0, 0), (a, b), (c, d)인 삼각형 넓이가 ;2!;|ad-bc|임을 이
다음과 같이 y=f(x)의 그래프 [그림 1]에서 y=2f(x)-1의 그
래프 [그림 2]를 그릴 수 있다. 용하면 세 꼭짓점이 (0, 0), {;2%;, ;2%;}, (2, 1)인 삼각형 넓이는

y y y=2f(x)-1 ;2!;|;2%;_1-;2%;_2|=;4%;
1
y=f(x)
1
;2!;
O a b 1 x
20 ^ 98
GUIDE
O 1 x
;2!; -1 두 함수 f, g의 그래프는 직선 y=x에 대해 대칭이므로 AB’ 길이의 최
솟값은 y=f(x)의 그래프와 직선 y=x 사이 최단 거리의 2배이다.
[그림 1] [그림 2]
2f(x)-1=0의 두 근을 a, b라 하면 y
y=f(x)
0<x<a일 때 0<2f(x)-1<1 y=x+k
y=x
a<x<;2!;일 때 -1<2f(x)-1<0 4
A y=f -1(x)
;2!;<x<b일 때 -1<2f(x)-1<0
-2 4 x
O B
b<x<1일 때 0<2f(x)-1<1

34 정답과 풀이
y=f(x)의 그래프에서 기울기가 1인 접선을 y=x+k라 하면 참고

f ‹=I, f(1)=3이면 f(3)=2, f(2)=1이어야 한다.


;2!;x€+2x+4=x+k, 즉 x€+2x+8-2k=0에서
f(3)=1 또는 f(3)=3이면 f ‹=I일 수가 없다. f ‹=I가 되는 경우는
다음과 같이 f(1)=3, f(2)=1, f(3)=2일 때이다.
;;4Î;;=1-8+2k=0  k=;2&;
f f f
X X X X
이때 x=-1, y=;2%; 이므로 A{-1, ;2%;}과 직선 y=x, 1 1 1 1
2 2 2 2
|-1-;2%;| 7'2
즉 x-y=0 사이의 거리는 = 3 3 3 3
"ƒ1€+(-1)€ 4

7'2 €
따라서 4_AB’ €의 최솟값은 4_{2_ } =98
4

21 ^④
STEP 3 1등급 뛰어넘기 p. 53~55
GUIDE
함수 2f(3x+1)-1을 더 간단한 꼴로 나타내 보자. 예를 들어 01 ㄷ, ㅁ 02 ③ 03 ⑴ 7 ⑵ 1
h(x)=3x+1, i(x)=2x-1이라 하면 04 4개 05 ⑤ 06 ② 07 ⑤
2f(3x+1)-1=i( f(h(x)))=(i@f@h)(x)이므로 08 ③ 09 ① 10 ③ 11 ⑤
[i@f@h(x)]-1=(h-1@f -1@i-1)(x)를 이용할 수 있다.
12 ⑴ 39 ⑵ 45 ⑶ ;;¡3º;;x<;;¡3¡;;

h(x)=3x+1, i(x)=2x-1이라 하면
2f(3x+1)-1=(i@f@h)(x) 01 ^ ㄷ, ㅁ
GUIDE
이때 (i@f@h)-1(x)=(h-1@f -1@i -1)(x)에서 f -1=g이고,
일대일함수와 일대일대응의 차이를 생각한다.

h-1(x)=;3!;x-;3!;, i -1(x)=;2!;x+;2!; 이므로


ㄱ. 함숫값도 모두 같아야 한다. ( _ )
(i@f@h)-1(x)=h-1(g(i -1(x)))=;3!; g {;2!;x+;2!;}-;3!; ㄴ. 일대일함수이다. ( _ )
ㄷ. f@g@h=f 의 양변에 g-1@f -1를 합성하면 h=g-1 ( ◯ )
ㄹ. [반례] X에서 Y로의 함수 f(x)=x와 Z에서 W로의 함수
g(x)=x에 대하여 X={1, 2, 3}, Y={1, 2, 3},
22 ^②
Z={1, 2, 3, 4}, W={1, 2, 3, 4}라 하면 g@f 의 치역은
GUIDE
역함수가 존재하는 함수 f 에서 f n=I (n은 자연수, I 는 항등함수)가 성
{1, 2, 3}이고 공역은 {1, 2, 3, 4}이므로 일대일대응이 아니
n
립할 때, f 의 역함수 g 에 대하여 g =I 이다. 다. ( _ )
ㅁ. X에서 Y로의 함수 f(x)=x와 Y에서 Z로의 함수
f 3=I이므로 f(1)=3이면 f(3)=2, f(2)=1이다.
x (x<3)
따라서 g(1)=2, g(2)=3, g(3)=1이다. g(x)=[ 에 대하여
3 (x=4)
f g X={1, 2, 3}, Y={1, 2, 3, 4}, Z={1, 2, 3}이라 하면
X X X X
g@f 는 일대일대응이다. ( ◯ )
1 1 1 1
ㅂ. f -1@f=f@f -1은 정의역과 공역이 같을 때만 가능하다.
2 2 2 2
3 3
(_)
3 3
참고

ㅂ. 함수 f : X → X가 일대일대응일 때 f -1@f=f@f -1=I (항등함수)


함수 f 의 역함수 g 에 대하여 g ‹=I이므로 g 10=g, g 11=g€
인 것처럼 정의역과 공역이 같아야 한다는 점을 주의한다.
g g
X X X
1 1 1
2 2 2 02 ^③
3 3 3 GUIDE
f(a)=g(a), f(b)=g(b), f(c)=g(c)에서 방정식 f(x)=g(x)의 세
근이 a, b, c임을 이용한다.
 g 10(2)+g 11(3)=g(2)+g €(3)=3+2=5

4. 함수 35
x‹+(1-a)x€-2bx+1-b=0의 세 근이 a, b, c이므로 -;2#; (t+2)+;2(;=-;2#;t+;2#; (0<t<1)
a+b+c=a-1, ab+bc+ca=-2b, abc=b-1
a€+b€+c€=(a+b+c)€-2(ab+bc+ca)
=(a-1)€+4b>3
[
f( f(t))= -;2#; {-;2(; t+;2(;}+;2(;=;4(;t-;4(; {1<t<;3&;}

{-;2#; t+;2(;}+2=-;2#;t+;;¡2£;; {;3&;<t<3}

a=1일 때, 최솟값 4b를 가지므로 a=1, b=;4#; 이므로


이때 a+b+c=a-1=0에서 f( f(t))=t가 되는 t의 값을 구하면 ([그림 1] 참조)
a‹+b‹+c‹=(a+b+c)(a€+b€+c€-ab-bc-ca)+3abc 1 -;2#;t+;2#;=t (0<t<1)에서 t=;5#;
=3abc=3(b-1)
2 ;4(;t-;4(;=t (1<t<;3&;)에서 t=;5(;
=-;4#;

3 -;2#;t+:¡2£:=t (;3&;<t<3)에서 t=:¡5£:

이때 [그림 2]와 같이 t=f(x)에서 f(x)=;5#;, ;5(;, ;;¡5;£ ; 이 되는 x값


03 ^⑴7 ⑵1
이 각각 1, 1, 2개이므로 f( f( f(x)))=f(x)가 되는 x값은 4개
GUIDE
⑴ g( f(4))+f(g(4))=g(5)+f(g(4))에서 y y=t y
3 y=f(x)
g가 일대일대응이므로 g(4)가 될 수 있는 값을 생각한다.
y=f(f(t)) f(x)=;;¡5£;;
⑵ (g@f@f )(k)=3의 양변에 g-1를 취한다. 2 f(x)=;5(;
;2#;
⑴ f(4)=5이므로 g( f(4))+f(g(4))=g(5)+f(g(4))
f(x)=;5#;
g 가 일대일대응이므로 g(4)의 값은 1, 2, 4 중 하나이다. O
1 3 t O x
;5#; ;5(; ;3&;
g(4) 1 2 4
;;¡5£;;
f(g(4)) 2 1 5
[그림 1] [그림 2]
g(5) 2 4 1 4 1 2

따라서 g(4)=4, g(5)=2일 때


g( f(4))+f(g(4))의 최댓값은 7
05 ^⑤
⑵ g(1)=3이므로 g-1@(g@ f@ f) (k)=g-1(3)에서 GUIDE
( f@ f )(k)=1이고 일대일대응인 함수는 역함수가 존재하므로 ( f@ f )(x)=x에서

( f -1@ f @ f )(k)= f -1(1)에서 f(k)=2  k=1 f(x)=f -1(x)가 성립한다.

ㄱ. 함수 f 가 일대일대응이므로 역함수가 존재한다.


다른 풀이
㈎에서 집합 X의 모든 원소 x에 대하여
g( f( f(k)))=3에서 g(1)=3이므로 f( f(k))=1
f(x)=f -1(x)이므로 f(3)=f -1(3) ( ◯ )
f(2)=1이므로 f(k)=2
이때 f(1)=2이므로 k=1 ㄴ. f(1)=3이면 ㈎에서 f( f(1))=f(3)=1이다.
따라서 ㈏에 의해 f(2)=4, f(4)=2이다. ( ◯ )
ㄷ. ㈏에서 f(1)=2와 f(2)=4 중 적어도 하나는 성립하므로
1 f(1)=2이고 f(2)+4일 때
04 ^ 4개
㈎에 따라 f( f(1))=f(2)=1
GUIDE
이때 f(3)=3, f(4)=4 또는 f(3)=4, f(4)=3
f(x)=t라 두고 f( f(t))=t가 되는 t값을 찾는다.
따라서 함수 f 는 2개
f( f( f(x)))=f(x)에서 y f f
3 X X X X
t=f(x)라 두면 f( f(t))=t이고, 1 1 1 1
2
y=f(t)의 그래프는 오른쪽 그림과 같다. 2 2 2 2
1
3 3 3 3
이때
O 1 3 t
f(t)+2 (0<f(t)<1) ;3&; 4 4 4 4
f( f(t))=[
-;2#; f(t)+;2(; (1<f(t)<3) 2 f(2)=4이고 f(1)+2일 때
에서 ㈎에 따라 f( f(2))=f(4)=2이고 이때

36 정답과 풀이
f(1)=1, f(3)=3 또는 f(1)=3, f(3)=1 08 ^③
따라서 함수 f 는 2개 GUIDE
f f f(n)은 n을 소인수분해 했을 때 소인수 2의 지수를 뜻한다.
X X X X
1 1 1 1
ㄱ. 모든 홀수는 소인수 2가 없다. ( ◯ )
2 2 2 2
ㄴ. [반례] 8<10이지만 f(8)=3, f(10)=1이므로
3 3 3 3
4 4 4 4 f(8)>f(10) ( _ )
ㄷ. f(2k)=f(k)+1이 성립하므로
3 f(1)=2이고 f(2)=4일 때
f(2)+f(4)+f(6)+y+f(200)
f( f(1))=f(2)=4이므로 ( f@ f )(1)=1이 성립하지 않
=f(1)+f(2)+f(3)+y+f(100)+100
는다.
이때 f(1)+f(2)+f(3)+y+f(100)은
1, 2, 3에서 가능한 함수 f 는 4개 ( ◯ )
(100 이하인 자연수 중 2의 배수 개수)
참고 +(100 이하인 자연수 중 4의 배수 개수)

역함수가 존재하는 함수 f 에서 +(100 이하인 자연수 중 8의 배수 개수)


( f@f )(x)=x는 f=f -1
, 즉 f(x)=f -1
(x)와 같은 표현이다. ⋮
+(100 이하인 자연수 중 64의 배수 개수)와 같으므로
100 100 100 100 100 100
06 ^②

2
‘+“
4
‘+“
8
‘+“
16
‘+“
32
‘+“
64

GUIDE +100
h–(x), h€(x), h‹(x), …에서 규칙성을 찾는다.
=50+25+12+6+3+1+100
3 (0<x<1) =197 ( ◯ )
h(x)=g([x])=[ 2 (1<x<2, x=3) 이므로 따라서 옳은 것은 ㄱ, ㄷ
1 (2<x<3)
참고
h–(x) h€(x) h‹(x) h›(x) h—(x) y h100(x)
0<x<1 3 2 1 2 1 y 2 ➊ 예를 들어 k=2€_3이면 f(k)=2, 이때 f(2k)=f(2‹_3)=3이다.
따라서 모든 자연수 k에 대하여 f(2k)=f(k)+1임을 알 수 있다.
1<x<2 2 1 2 1 2 y 1
➋ f(1)=f(3)=f(5)=…=f(99)=0
2<x<3 1 2 1 2 1 y 2
x=3 2 1 2 1 2 y 1
다른 풀이
따라서 h100(x)=h€(x)=x가 되는 x값은 1, 2로 2개
ㄷ. f(홀수)=0이므로 f(2)+f(4)+f(6)+y+f(200)은

참고
(200 이하인 자연수 중 2의 배수 개수)
+(200 이하인 자연수 중 4의 배수 개수)
1<x<2에서 h100(x)=1이므로 이때 x=1
2<x<3에서 h100(x)=2이므로 이때 x=2 +(200 이하인 자연수 중 8의 배수 개수)

+(200 이하인 자연수 중 128의 배수 개수)
07 ^⑤ 와 같다. 따라서
GUIDE
200 200 200 200 200 200
“ ‘+“ ‘+“ ‘+“ ‘+“ ‘+“ ‘
x=;3!; 과 "ƒ1-x€=;3!; 일 때 x값을 이용한다. 2 4 8 16 32 64
200
+“ ‘
128
2f(x)+3f("ƒ1-x€ )=x에 x=;3!; 을 대입하면
=100+50+25+12+6+3+1=197
2'2
2f {;3!;}+3f { }=;3!; …… ㉠
3
2'2
2f(x)+3f("ƒ1-x€ )=x에 x= 를 대입하면
3
09 ^①

2f {
2'2
}+3f {;3!;}=
2'2
…… ㉡ GUIDE
3 3
f¡(x)+f™(x)+f£(x)+y+fk(x)에서 k가 홀수일 때 그래프 개형과
-2+6'2 짝수일 때 그래프 개형을 따져 본다.
㉠, ㉡을 연립하여 풀면 f {;3!;}=
15

4. 함수 37
ㄱ. x=b일 때 y=|b-a|+0+|b-c|=c-a 참고

이므로 옳다. ( ◯ ) y=|x-a|+|x-b|+|x-c| 두 자연수 x, n과 0 이상의 정수 q에 대하여


ㄴ. b-a+c-b이면 좌우 대칭 x=nq+r (0<r<n)으로 나타내면
x r x r
이 아니므로 그래프의 대칭축 =q+ 이므로 “ ‘=“q+ ‘=q
n n n n
은 없다. ( _ ) x
따라서 x-n“ ‘=x-nq=r
n
ㄷ. k가 짝수인 경우에는 무수히 x=a x=b x=c

많다. 예를 들어
f¡(x)+f™(x)는 1x2일 때 최솟값을 갖는다. ( _ )

참고 y=|x-a|+|x-b| 12 ^ ⑴ 39 ⑵ 45 ⑶ ;;¡3º;;<x<;;¡3¡;;
k가 짝수인 경우 오른쪽 그림과
GUIDE
같이 그래프에서 x축과 평행한
➊ [x]=n이면 n이 정수이므로 nx<n+1이 성립한다.
부분이 있다.
a b x ➋ [x]는 정수이므로 [x[x]]+[x]=[x[x]+[x]]가 성립한다.

⑴ f(x)=[3x]+3=[3x+3]=[3(x+1)]이고,
[x]=3이면 3x<4, 즉 4x+1<5이므로

10 ^③ 123(x+1)<15
GUIDE 따라서 가능한 f(x)의 값은 12, 13, 14이므로 구하려는 값은
음이 아닌 정수 N에 대하여 f(N)은 N의 모든 자리 수의 합을 나타낸다. 12+13+14=39
⑵ f(x)=[x[x]]+[x]=[x[x]+[x]]=[(x+1)[x]]이고,
음이 아닌 정수 m에 대하여
[x]=n이면 nx<n+1, 즉 n+1x+1<n+2이므로
n=am_10m+y+a™_10€+a¡_10+aº이라 하면
n€+n(x+1)[x]<n€+2n
f(am_10m+y+a™_10€+a¡_10+aº)
따라서 가능한 f(x)의 값은
=f(am_10m-1+y+a™_10+a¡)+aº
n€+n, n€+n+1, n€+n+2, y, n€+2n-1
=f(am_10m-2+y+a™)+a¡+aº
즉 [x]=n일 때 가능한 f(x)의 값이 모두

(n€+2n-1)-(n€+n)+1=n개이므로 g(n)=n
=am+y+a™+a¡+aº
 g(1)+g(2)+y+g(9)=1+2+y+9=45
 f(n)=am+y+a™+a¡+aº
⑶ n€+n(x+1)[x]<n€+2n에서
ㄱ. f(100)=1 ( ◯ )
n=1일 때, 2(x+1)[x]<3
ㄴ. ( f@ f )(999)=f(27)=9 ( ◯ )
n=2일 때, 6(x+1)[x]<8
ㄷ. [반례] n=15일 때, f(n)=6이지만 n은 6의 배수가 아니다.
n=3일 때, 12(x+1)[x]<15
(_)
n=4일 때, 20(x+1)[x]<24

이므로 f(x)=13인 경우는 [x]=3에서 찾을 수 있다.

11 ^⑤ f(x)=[x[x]]+[x]=[3x]+3=13에서 [3x]=10,
GUIDE
즉 103x<11이므로 ;;¡3º;;x<;;¡3¡;;
예를 들어 f7(32)=32-7_“:£7™:‘=32-7_4=4,

즉 32=7_4+4에서 32를 7로 나눈 나머지가 4이다.


따라서 fn(x)는 x를 n으로 나눈 나머지를 뜻한다.

ㄱ. x를 10으로 나눈 나머지는 x의 일의 자리 수와 같다. ( ◯ )


ㄴ. f7(x)=1은 x를 7로 나눈 나머지가 1인 수이다.
x=7k+1 (k는 0 이상인 정수)이므로 모두 15개이다. ( ◯ )
ㄷ. 5!부터는 모두 120의 배수이므로 일의 자리 수가 모두 0이다.
1!+2!+3!+4!=1+2+6+24=33이므로
f10(1!+2!+3!+y+999!)=3 ( ◯ )

38 정답과 풀이
05 유리함수와 무리함수 04
GUIDE
^①

➊ 직사각형 ROQP의 가로 길이와 세로 길이를 한 문자로 나타내어 넓


STEP 1 1등급 준비하기 p. 58 ~59
이를 구한다.
01 6 02 ① 03 ;3!; 04 ① ➋ (산술평균)>(기하평균)을 이용한다.

05 -;9!; 06 ① 07 ⑴ 1 ⑵ '2 ⑶ '3-'2


x>0에서 함수 y=;x@;의 그래프를 y
08 0<a<1 09 -4 10 ④
x축 방향으로 1만큼, y축 방향으로

01 ^6 2만큼 평행이동한 y=
2
x-1
+2의 R P 2
y= x-1 +2
GUIDE
y 1
그래프는 오른쪽 그림과 같으므로
[x, y]= =;x!;- 을 이용한다.
x(x+y) x+y 2
그래프 위의 점 P {t, +2} O Q x
t-1
[a, 1]+[a+1, 2]+[a+3, 3]
(단, t>1)에 대하여 직사각형 ROQP의 넓이 S는
1 1 1 1 1
=;a!;- + - + - 2 2
a+1 a+1 a+3 a+3 a+6 S=t { +2}=(t-1+1){ +2}
t-1 t-1
1
=;a!;- =[a, 6]  b=6 2
a+6 =4+2(t-1)+
t-1
2
4+2Ƙ2(t-1){ }=8
t-1

02 ^①
(단, 등호는 t=2일 때 성립한다.)
GUIDE 따라서 직사각형 ROQP 넓이의 최솟값은 8이다.
분수함수의 그래프는 두 점근선의 교점을 지나는 기울기가 1 또는 -1인
직선에 대하여 대칭임을 이용한다.

05 ^ -;9!;
-3x+3 3
y= = -;2#; 의 그래프는 {;2!;, -;2#;}을 지나 GUIDE
2x-1
4{x-;2!;}
x-1
f(x)= 로 놓고 f {;3@;}, f { f {;3@;}}, f { f { f {;3@;}}}, …의 값을
6x-1
면서 기울기가 1인 직선에 대하여 대칭이다.
차례로 구해 보며 규칙성을 찾는다.
즉 y= {x-;2!;}-;2#;에서 y=x-2 또는 y=-x-1이므로
x-1
f(x)= 이라 하면
a+b의 값은 -1 또는 -2 6x-1
따라서 합은 (-1)+(-2)=-3
f {;3@;}=-;9!;, f { f {;3@;}}=;3@;, f { f { f {;3@;}}}=-;9!;, y

따라서 짝수 번 합성하면 ;3@;, 홀수 번 합성하면 -;9!;이므로

03 ^ ;3!; 1001번 합성하면 -;9!;


GUIDE
x+1
함수 y= (2x3)의 그래프를 그리고,
06
x-1
^①
두 직선 y=ax, y=bx+1의 위치를 생각해 본다.
GUIDE
➊ y=f(x)를 원점에 대하여 대칭이동한 것은 -y=f(-x)이다.
오른쪽 그림에서 2x3일 때 y
y=bx+1 ax+b -dx+b
➋ y= 의 역함수는 y=
x+1 cx+d cx-a
ax 이려면 y=ax
x-1 3
2 3(x-1)+2 2
a;3@;이므로 a의 최댓값은 ;3@; 1 f(x)= = +3이고, y=f(x)를 원점에 대하
x-1 x-1
-1
x+1 O 1 2 3 x 2
또한 bx+1이려면 여 대칭이동한 그래프의 식은 y= -3이다.
x-1 x+1

b1이므로 b의 최솟값은 1 또 y=f(x)의 역함수는 y=


x-1
=
2
+1이므로
x-3 x-3
따라서 b-a의 최솟값은 1-;3@;=;3!; 이것을 x축으로 -4만큼, y축 방향으로 -4만큼 평행이동하면

5. 유리함수와 무리함수 39
y=
2
x+1
-3의 그래프와 겹쳐진다. 09 ^ -4
GUIDE
따라서 a=-4, b=-4이므로 a+b=-8 역함수의 치역은 원래 함수의 정의역과 같으므로 원래 함수에서 정의역
을 구한다.

07 ^⑴1 ⑵ '2 ⑶ '3-'2 무리함수 y=-'ß-2x+16+3의 치역이 {y|-3y1}일 때,


GUIDE 1=-'ß-2x+16+3에서 x=6
⑴ 분모 유리화를 이용한다. -3=-'ß-2x+16+3에서 x=-10이므로
1 정의역은 {x|-10x6}
⑵ 2+ 을 간단히 해본다.
'2+1
⑶ {(a+b)+c}{(a+b)-c}를 이용하여 분모를 간단한 꼴로 바꾼다.
따라서 역함수의 치역 {y|-10y6}에서
a=-10, b=6  a+b=-4
1 1 1 1
⑴ + + +
1+'3 '3+'5 '5+'7 '7+'9

=;2!;{('3-1)+('5-'3 )+('7-'5 )+('9-'7 )} 10 ^④


GUIDE
=;2!;('9-1)=1 증가하는 함수의 그래프와 그 역함수의 그래프가 만나는 점은 그 함수의
그래프와 직선 y=x의 교점과 같다.
1
⑵ 2+ ='2+1이므로
'2+1 무리함수 y=f(x)와 그 역함수 y=f -1(x)의 그래프가 서로 다
1 1 른 두 점에서 만나려면 다음 그림과 같이 f(x)='ßx-1+k의
2+ =2+ ='2+1
1 '2+1
2+
'2+1 그래프가 직선 y=x와 서로 다른 두 점에서 만나야 한다.
y
1 y=x
 1+ ='2
'2+1
1-'2+'3 1-'2+'3 y= x-1+k
⑶ =
1+'2+'3 (1+'2 )+'3 1
(1, k)
(1-'2+'3 )(1+'2-'3 ) O 1 x
=
{(1+'2 )+'3 }{(1+'2 )-'3 }
1-('2-'3 )€ 그림에서 k>1일 때 y='ßx-1+k의 그래프와 직선 y=x는 한
=
(1+'2)€-3 점에서 만나므로, 두 점에서 만날 때 상수 k의 최댓값은 1이다.
-4+2'6
= ='3-'2
2'2 참고 두 점에서 만날 때 k의 범위
두 그래프가 접할 때는
'ßx-1+k=x,
08 ^ 0<a<1 x€-(2k+1)x+k€+1=0
GUIDE D=(2k+1)€-4(k€+1)=0에서
판별식을 이용해 a값을 구하고, 그래프에서 조건에 맞는 값은 무엇인지 k=;4#;  ;4#;<k<1
따져 본다.

문제 내용을 그림으로 나타내면 다음과 같다.


y
a=1일 때

y= 2x-1
STEP 2 1등급 굳히기 p. 60~67

01 ③ 02 10 03 ⑤ 04 ③
O x
05 ② 06 ④ 07 -3 08 ;8*1@;
y=ax
09 6 10 ④ 11 5 12 ④
y='ß2x-1 과 y=ax가 접할 때 a값을 구하면 13 36 14 2'ß17p 15 2 16 x1
a€x€=2x-1, a€x€-2x+1=0에서 17 ④ 18 ③ 19 ⑤ 20 3
21 8 22 3 23 26개 24 36
;;4Î;;=1-a€=0, 즉 a=1 또는 a=-1이다. 25 ② 26 10 27 ④ 28 ①
29 -1 30 ③
'ß2x-1 >0이므로 a+-1  0<a<1

40 정답과 풀이
01 ^③ a
4a-b-c
=
b
4b-a-c
=
c
4c-a-b
=k에서
GUIDE
c c k€ a b c
= {;a!;-;b!;} 을 이용하여 f(x)f(x+1)= 을 = = =;5!;=k
a_b b-a x(x+1) 4a+a 4b+b 4c+c
변형한다.
따라서 모든 k값의 곱은 ;2!;_;5!;=;1¡0;
f(1)f(2)+f(2)f(3)+y+f(80)f(81)
k€ k€ k€ LECTURE 가비의 리
= + +y+
1_2 2_3 80_81 a+c+e
;bA;=;dC;=;fE;=k일 때 k= (단, b+d+f+0)
b+d+f
=k€[{;1!;-;2!;}+{;2!;-;3!;}+y+{;8¡0;-;8¡1;}]
b+d+f=0인 경우에는 이를 이용해 ;bA;=;dC;=;fE;=k에서

80k€ 분자 또는 분모를 변형해 k값을 찾는다.


={1-;8¡1;}k€= =320
81
따라서 k=18 (∵ k>0)
04 ^③
GUIDE
k
y= +c가 분수함수이면 a+0이고 k+0임을 이용한다.
02 ^ 10 ax+b

GUIDE
bc bc
1 f( f( f(s)+r)+q)를 번분수 꼴로 나타낸다. ;aC;(ax+b)- +d - +d
cx+d a a
y= = = +;aC;
2 ;1$9#;=2+(번분수) 꼴로 나타낸다. ax+b ax+b ax+b
cx+d
이므로 y= 가 분수함수이면 a+0이고
ax+b
1
f( f( f(s)+r)+q)+p=p+ =;1$9#;이고
1 bc
q+ - +d+0에서 ad+bc여야 한다.
r+;s!; a
따라서 ㄱ, ㄹ
1 1 1
;1$9#;=2+;1y9;=2+ =2+ =2+
:¡5ª: 3+;5$; 3+ 1
;4%; 05 ^②

1 GUIDE
=2+
3+ 1 y=
2x-1
=
-3
+2이므로 정수 x에 대하여
-3
이 정수이면
x+1 x+1 x+1
1+;4!;
y도 정수가 된다.
이므로 p+q+r+s=2+3+1+4=10
2x-1 3
y= x+1 =- x+1 +2이므로 y가 정수이려면
1등급 NOTE
1 x+1이 3의 약수가 되어야 하므로 x=0, 2, -2, -4로 4개
번분수 p+ 를 변형하여 q, r, s를 구하기가 까다로우므로
1
q+

06
r+;s!;
^④
시도하지 않도록 한다. GUIDE
mx-3m-15 k
y= 를 y= +q 꼴로 바꾸어 좌표평면 위에서
x-3 x-p
생각한다.
03 ^⑤
GUIDE mx-3m-15 -15
y= = +m이므로 그래프의 점근선은
세 분수의 분모를 모두 더한 2a+2b+2c가 0이 아닐 때와 0일 때로 나 x-3 x-3
누어 생각한다. x=3, y=m이다.
y
1 2a+2b+2c+0일 때
a+b+c m+5
(4a-b-c)+(4b-a-c)+(4c-a-b)
O x
y=m
a+b+c
= =;2!;=k
2(a+b+c)
2 2a+2b+2c=0일 때
a=-b-c, b=-a-c, c=-a-b이므로 x=3

5. 유리함수와 무리함수 41
이때 그림처럼 제3사분면을 지나기 위해서는 m<0 09 ^6
또 y절편이 양수일 때 제2사분면을 지나므로 GUIDE
m+5>0  -5<m<0 1 점 P의 좌표를 {a, ;a!;}이라 한다.

따라서 정수 m은 -4, -3, -2, -1로 4개이다.


2 a+;a!;=k, OP’='å34에서 k값을 구한다.

1
P {a, ;a!;}이라 하면 a+;a!;=k이고 OP’=æa€+ ='å34 이므로
07 ^ -3 a€
GUIDE a€+
1 €
={a+;a!;} -2=k€-2=34, k€=36
함수 y=f(x)에 대하여 x¡+x™=2p, f(x¡)+f(x™)=2q가 성립하면 a€
함수의 그래프가 점 (p, q)에 대하여 대칭임을 이용한다. 따라서 양수 k값은 6

x¡+x™=10인 서로 다른 두 실수 y
10 ^④
x¡, x™에 대하여 GUIDE
f(x¡)+f(x™)=10이 성립하면 f(x™) 두 식을 연립하여 근과 계수의 관계와 피타고라스 정리를 이용하자.
5
함수 y=f(x)의 그래프는
f(x¡) 3
점 (5, 5)에 대하여 대칭이다. y= +1과 y=2x+k의 교점을 P, Q라 하자.
x-1
따라서 y=f(x)의 그래프의 두 O x¡ 5 x™ x
3
점근선은 x=5, y=5이고, 이때 +1=2x+k에서 2x€+(k-3)x-k-2=0의 해를
x-1

f(x)=
bx+1
=
k
+5=
5ax-25a+k a, b (a<b)라 하면 각각은 두 점 P, Q의 x좌표이다. 이때
ax+1 a(x-5) ax-5a
"ƒ(k-3)€-4_2_(-k-2) k€+2k+25
에서 -5a=1, 5a=b이므로 b-a= =Ƙ
2 4
5a+2b=(-1)+(-2)=-3 따라서 y y=2x+k
PQ’='5_(b-a) Q
1등급 NOTE
5(k€+2k+25)
=Ƙ O
b{x+;a!;}+1-;aB 4
bx+1 ; x
f(x)= = =y P
ax+1 b-a
a{x+;a!;} =æ;4%;{(k+1)€+24}
3
y= x-1 +1
이렇게 변형하면 복잡하므로 대칭점의 좌표를 이용하는 것이 편리하다. 에서 k=-1일 때, 최솟값은 'ß30

다른 풀이 참고 Q

bx+1 기울기가 2인 직선 위의 두 점 사이의 거리는


y= 의 점근선이 x=-;a!;, y=;aB;이고 이것과 점근선 '5m
ax+1 오른쪽 그림처럼 |b-a|의 '5배이다. 2m

x=5, y=5를 비교하면 a=-;5!;, b=-1 P


m

11 ^5
GUIDE

08 ^ ;8*1@; x>0일 때 두 함수 y=|1-;x!;|과 y=ax+b의 그래프에서

GUIDE 세 양수 근의 비가 1:2:3인 경우를 그림으로 나타내어 본다.


직선의 방정식과 함수식을 연립하였을 때, 실근이 존재하지 않아야 한다.
y

(-3, 1)을 지나고 기울기가 m인 직선을 y=m(x+3)+1이라 1

하면 ;x!;=m(x+3)+1의 실근이 존재하지 않아야 하므로


O
mx€+(3m+1)x-1=0에서 k1 2k 3k x

D=(3m+1)€+4m<0, (m+1)(9m+1)<0
x>0일 때, y=|1-;x!;|의 그래프는 위 그림과 같다.
 -1<m<-;9!;
이때 세 양수 근을 k, 2k, 3k라 하면 0<k<1이고 2k는 k와 3k
따라서 a=-1, b=-;9!;이므로 a€+b€=1+;8¡1;=;8*1@;
의 중점이다. 세 교점이 한 직선 위에 있으므로

42 정답과 풀이
f(k)+f(3k) 삼각형 PAB의 밑변을 AB’라 할 때, 높이 d 가 최소일 때 넓이도
f(x)=|1-;x!;|라 하면 =f(2k)
2
최소이다.
1 1
{-1+;k!;}+{1- }=2 {1- } 4
3k 2k (산술평균)>(기하평균)에서 t+ >4이므로
t
5
즉 =2에서 k=;6%;이므로 k+2k+3k=5 삼각형 넓이의 최솟값은
3k
4
a=;2!;_'ß36+9_ =6  a€=36
'5
12 ^④
GUIDE 다른 풀이 접선 이용하기
3x+5
➊ 함수 y= 의 그래프를 그려 최댓값을 찾는다.
|x|+1 직선 AB와 기울기가 같은 직선이 곡선 y=;x@;(x>0)에 접할 때
➋ y=mx+3과 교점이 두 개가 되는 조건을 찾는다.
이 두 직선 사이의 거리가 삼각형 PAB의 높이의 최솟값이다.
3x+5 2
x>0일 때 y= = +3이고, 직선 AB의 기울기가 -;2!;이므로 접선을 y=-;2!;x+k라 하면
x+1 x+1
3x+5 -8
x<0일 때 y= = -3이므로 ;x@;=-;2!;x+k, x€-2kx+4=0
-x+1 x-1
그래프는 다음과 같다.
D=0에서 ;;4Î;;=k€-4=0
y y=mx+3
5
이때 y=-;2!;x+k의 y절편이 양수이므로 k=2
3
점 A(-2, 1)에서 접선 x+2y-4=0까지의 거리는
-1 O1 x |-2+2_1-4| 4
= 이고 AB’=3'5 이므로
"ƒ1€+2€ '5
-3
4
PAB=;2!;_3'5_ =6에서 a€=36
x=0일 때 최댓값은 5이고 y=mx+3과 교점이 2개이려면 '5

-8
y= -3과 접해야 한다.
x-1
-8
14
mx+3= -3, mx€+(6-m)x+2=0에서
x-1 ^ 2'ß17p

D=(m-6)€-8m=0, (m-2)(m-18)=0 GUIDE


1
조건에 맞는 것은 m=2 양수 t에 대하여 점 A의 좌표를 {t,
t
}이라 하고 삼각형 OAB의 넓이를

따라서 p=0, q=5, m=2이므로 p+q+m=7 t로 나타낸다. 이때 사각형 ABCD가 직사각형인지 따져 본다.

참고
y=;x!;의 그래프와 중심이 원점인 원은 모두 두 직선 y=x와
-8
m=18일 때는 직선이 곡선에서 그려지지 않은 부분, 즉 y= -3
x-1 y=-x에 대칭이므로 사각형 ABCD는 직사각형이다.
에서 x>0인 부분과 접한다. 1 1
이때 양수 t에 대하여 A{t, }이라 하면 B { , t}
t t
또 직사각형 ABCD의 넓이는 삼각형 OAB 넓이의 4배이고,
13 ^ 36
GUIDE 1
OAB의 넓이는 ;2!;|t€- |이므로
t€
선분 AB를 삼각형의 밑변으로 생각하고 점 P의 x좌표를 t라 할 때 점
P에서 직선 AB까지의 거리를 t로 나타내어 높이의 최솟값을 구한다. 1 1
직사각형의 넓이는 4_;2!;|t€- |, 즉 2|t€- |
t€ t€
두 점 A, B를 지나는 직선의 y 1 1
y=;x@; 2|t€- |=16에서 |t€- |=8
t€ t€
방정식은 x+2y=0이고,
P{t, ;t@;} 1
2 A(-2, 1) 원의 반지름 길이는 선분 OA의 길이이고, OA’=æt€+ 이므로
점 P {t, }에서 직선 t€
t d
O x 1
x+2y=0까지의 거리 d 는 (원의 넓이)=p_OA’ €=p_{t€+ }
t€
4 B(4, -2)
|t+ | 1 €
t =p_Ƙ{t€- } +4=2'ß17p
d= t€
'5

5. 유리함수와 무리함수 43
참고 16 ^ x1

AB’=Ƙ2{t-;t!;}€이고, 직선 AB의 방정식이 x+y-t-;t!;=0이므로 GUIDE


➊ y=f(x)의 역함수가 존재하려면 일대일대응이어야 한다.
|-t-;t!;| ➋ y=f(x)의 그래프에서 생각한다.
원점에서 직선 AB까지 거리, 즉 OAB의 높이는
'2
3x
∴ OAB=;2!;_
1
|t+;t!;|_Ƙ2{t-;t!;}€
x1일 때, f(x)= =3
1+x-1
'2
3x 3x -6
=;2!;|{t+;t!;}{t-;t!;}| x<1일 때, f(x)= = = -3
1-x+1 2-x x-2
따라서 y=f(x)의 그래프는 그림과 같다.
1등급 NOTE y
y=3
원점 O와 두 점 A(x¡, y¡), B(x™, y™)가 꼭짓점인 삼각형의 넓이는 3

;2!;|x¡y™-x™y¡|임을 공식처럼 이용한다. O


-6 1 2 x
y= x-2 -3

-3

즉 y=f(x)가 x<1인 범위에서 일대일대응이므로 x1일 때


15 ^2 역함수가 존재한다.
GUIDE
1 원래 함수와 그 역함수가 일치할 조건에서 a값을 찾는다.
17 ^④
2 분수함수의 그래프와 직선이 만나는 두 점 사이의 거리를 이용해 b값
GUIDE
을 찾는다. 분수함수의 합성을 반복하며 규칙성을 찾는다. 또 합성함수의 정의역은
이전 함수의 정의역을 기준으로 생각해야 한다.
ax+b -2x+b
f(x)= 와 역함수 f -1(x)= 가 서로 일치하
x+2 x-a
x x
ㄱ. f(x)= 일 때 f -1(x)= 이므로
므로 a=-2 1-x 1+x
y=f(x)와 y=2x+2의 두 교점을 A, B라 하면 두 점 사이의 x
1+x x
거리가 4'5 이므로 점 A, B의 x좌표의 차는 4이다. {f }€(x)=
-1
= ,
x 1+2x
-2x+b 1+
=2x+2에서 -2x+b=2x€+6x+4 1+x
x+2
x
즉, 2x€+8x+4-b=0의 해를 a, b라 할 때, 1+2x
-1 x x
(f )‹(x)= = , y, ( f -1)‡=
4-b x 1+3x 1+7x
|a-b|=4, a+b=-4, ab= 이므로 1+
1+2x
2
(a-b)€=(a+b)€-4ab=16-8+2b=4€, b=4 ∴ ( f -1)‡(1)=;8!; ( _ )
a+b=2
x
x 1-x x
1등급 NOTE
ㄴ. f(x)= , f €(x)= = ,
1-x x 1-2x
1-
1-x
ax+b
➊ f(x)= 의 두 점근선은 x=-2, y=a이다.
x+2 x
분수함수의 그래프는 두 점근선의 교점을 지나고 기울기가 1 또는 1-2x x
f ‹(x)= = , y,
x 1-3x
-1인 직선에 대해 대칭이다. 따라서 분수함수의 그래프와 역함수의 1-
1-2x
그래프가 일치할 때, 두 점근선의 교점이 직선 y=x 위에 있으면 된
x
다. 즉 교점 (-2, a)가 y=x 위에 있으려면 a=-2 f 10(x)= 이므로 a=1, b=0, c=-10이다. ( ◯ )
1-10x
➋ 기울기가 2인 직선 위의 두 점 사이의 거리가
ㄷ. 정의역에 포함되지 않는 실수를 찾으면
4'5이면 피타고라스 정리에서 m=4임을 구할
4'5 2m x x
수 있다. f(x)= 에서 1, f €(x)= 에서 1, ;2!;
1-x 1-2x
➌ 이차방정식 ax€+bx+c=0에서 두 실근
'ßD m x
a, b에 대하여 |a-b|= 를 이용해도 f ‹(x)= 에서 1, ;2!;, ;3!;, y
|a| 1-3x
된다. 즉 a<b라 할 때 x 1
f k(x)= 에서 1, ;2!;, ;3!;, y,
"ƒ64-8(4-b) 1-kx k
b-a= =4 ∴ b=4
2 따라서 정의역에 포함되지 않는 실수는 k개 ( ◯ )

44 정답과 풀이
참고 10(x-3)
따라서 병렬 연결에서 저항은
x+7
ㄷ에서 f@f, 즉 f €가 정의되려면 X¡ #X™로의 함수와 X™ # X£로의
10(x-3)
함수를 모두 생각해야 하므로 X¡에서 x+1이고, X™에서 x+;2!; 임을 알 수 이것과 51은 직렬 연결이므로 저항은 +5
x+7
있다. 10(x-3) -100
즉 총 저항 R= +5= +15이므로
x+7 x+7
20
5x<13에서 R<10
18 ^③ 3
GUIDE
두 점 사이의 거리와 점과 직선 사이의 거리를 이용해 x, y에 대한 식을
구하고 정리한다.
20 ^3
GUIDE
P(x, y)와 A(2, 2) 사이의 거리는 "ƒ(x-2)€+(y-2)€ a€+b€=(a+b)€-2ab임을 이용하여 2ab+1과 ab-1의 부호를 알아
본다.
|x+y|
P(x, y)와 y=-x 사이의 거리는
'2
a€+b€=1이므로 2ab+1=2ab+a€+b€=(a+b)€>0
|x+y|
이때 "ƒ(x-2)€+(y-2)€: ='2:1에서  "ƒ(2ab+1)€=2ab+1
'2
ab-1=ab-a€-b€=-(a€+b€-ab)0
|x+y|="ƒ(x-2)€+(y-2)€ y
 "ƒ(ab-1)€=-ab+1
의 양변을 제곱해 정리하면 2 A(2, 2)
P(x, y) 이때 "ƒ(2ab+1)€+2"ƒ(ab-1)€=2ab+1+2(-ab+1)=3
8 -2
y= -2이고 그래프는 O 2 x
x+2
-2
오른쪽과 같다. 21 ^8
① 그래프는 모든 사분면을 y=-x GUIDE
지난다. ( ◯ ) x2, x<2일 때로 나누어 그린 그래프에서 따져 본다.

② 그래프는 점근선의 교점 (-2, -2)를 지나면서 기울기가 1


x>2일 때 y='ßx-2+3 y
인 직선에 대해 대칭이므로 y=x, y=-x-4에 대하여 대칭
x<2일 때 y="ƒ-(x-2)+3 (1, 4)
이다. ( ◯ ) (6, 5)
즉 y="ƒ |x-2|+3의 그래프
③ x>3에서 x값이 증가하면 y값은 감소한다. ( _ ) (2, 3)
는 꼭짓점이 (2, 3)이고 오른쪽
8 과 같으므로 O x
④ 그래프 위의 점 P {a, -2}와 (-2, -2) 사이의 거리는
a+2
x=2일 때, 최솟값 3
64
æ(a+2)€+ 이고, x=6일 때, 최댓값 5
(a+2)€
64 따라서 최댓값과 최솟값의 합은 5+3=8
(a+2)€+ 2'ß64=16이므로 최단 거리는 4 ( ◯ )
(a+2)€
⑤ xy<0이면 제2사분면과 제4사분면에 존재하는 점이다.
22 ^3
제2사분면에 존재하려면 -2<x<0이므로 (-1, 6) GUIDE
제4사분면에 존재하려면 -2<y<0이므로 (6, -1) 두 점 A, B의 좌표를 문자를 이용해 나타내고, 기울기를 이용한다.
따라서 조건에 맞는 점은 2개 ( ◯ )
a+3 b+3
A{a, æ }, B{b, æ }라 하면 직선 AB의 기울기가
2 2
1등급 NOTE

분수함수 그래프 위의 점에서 최대*최소 문제는 (산술평균)>(기하평균) ;1¡8;이므로


을 생각한다.
b+3 a+3
æ -æ
2 2
b-a
19 ^⑤
{æ
b+3
-æ
a+3
} {æ
b+3
+æ
a+3
}
GUIDE 2 2 2 2
⇦ 분자 유리화
=
1 병렬 연결 부분을 찾아 저항을 x로 나타낸다. b+3 a+3
(b-a){Ƙ +Ƙ }
2 직렬 연결에서 전체 저항을 x로 나타낸다.
2 2
1
= =;1¡8;
1 1 1 R¡R™ b+3 a+3
= + 을 정리하면 R= 2{Ƙ +Ƙ 2 }
R R¡ R™ R¡+R™ 2

5. 유리함수와 무리함수 45
b+3 a+3 이때 점 B'(-c, -d )는 y=2'ßx-2+1의 그래프를 원점에 대
 Ƙ +Ƙ =9
2 2
하여 대칭이동한 y=-2'ß-x-2-1 (-27x-2)의 그래
이때 OAB의 무게중심 G의 좌표는
프 위에 있다.
a+3 b+3
æ +æ 1 기울기의 최댓값은 공통접선일 때이고, 이 접선은 원점을 지나
{ a+b
,
2 2 }
이므로 y좌표는 ;3(;=3
3 3 므로 y=mx와 y=2'ßx-2+1이 접할 때
mx=2'ßx-2+1, m€x€-2(m+2)x+9=0

;;4Î;;=(m+2)€-9m€=-8m€+4m+4=0
23 ^ 26개
2m€-m-1=0  m=1 ( m>0)
GUIDE
➊ |x-4|+2|y-3|=4를 좌표평면 위에 나타낸다. 2 최솟값은 두 점 (27, 11), (-27, -11)을 지나는 직선의 기
➋ y='ß2x-k의 그래프가 |x-4|+2|y-3|=4와 만나는 범위를 따
울기인 ;2!7!;
져 본다.
y
1
그래프는 다음과 같고, 점 A(0, 3), B(4, 1), C(8, 3), D(4, 5)
11
라 하자. 2
y
D -27
y= 2x-k O 27 x
C
A

B -11
O x

변 BC, 즉 y=;2!;x-1 (4x8)에 y='ß2x-k의 그래프가 접 1, 2에서 3p+81q=3_1+81_;2!7!;=36

할 때 k값이 가장 크다.

'ß2x-k=;2!;x-1에서 2x-k=;4!;x€-x+1

즉 x€-12x+4+4k=0이 중근을 가져야 하므로


25 ^②
;;4Î;;=6€-4-4k=0  k=8 GUIDE
[x]가 식에 포함되어 있으므로 정수 n에 대하여 nx<n+1마다 그래
또한 y='ß2x-k 의 그래프가 점 D(4, 5)를 지날 때
프를 그려 생각한다.
k값이 가장 작으므로 5='ß2_4-k  k=-17
따라서 -17k8에 속하는 정수는 -17, -16, …, 8로 모두 방정식의 실근의 개수는 두 함수 y="ƒx-[x], y=ax의 그래프
26개 에서 교점의 개수와 같다. y=ax가 원점을 지나므로
y
참고 y=x y=;2!;x

➊ |x|+2|y|=4의 그래프는 x+2y=4 (x>0, y>0)의 그래프와 이 1


그래프를 x축, y축, 원점에 대하여 대칭이동한 것을 함께 나타낸다. y= x-[x]
➋ |x-4|+2|y-3|=4의 그래프는 ➊에서 구한 그래프를 x축 방향
으로 4만큼, y축 방향으로 3만큼 평행이동한 것이다. O 1 2 x

y=ax가 점(1, 1)을 지날 때 a=1


y=ax가 y='ßx-1과 접할 때는
24 ^ 36
ax='ßx-1 (a>0)에서 a€x€-x+1=0
GUIDE
b+d b-(-d ) 이 방정식이 중근을 가질 조건 D=1-4a€=0에서 a=;2!;
➊ = 에서 두 점 A(a, b)와 점 B를 원점에 대하여 대
a+c a-(-c)
칭이동한 B'(-c, -d )를 지나는 직선의 기울기임을 안다.
이때 ;4!;x€-x+1=0에서 x=2이므로
➋ 기울기가 최대일 때와 최소일 때를 각각 따져 본다.

1<x<2에서 y='ßx-1은 y=;2!;x와 만나지 않는다.


b+d b-(-d )
= 이므로 두 점 A(a, b)와 B'(-c, -d)를
a+c a-(-c)
따라서 a값의 범위는 ;2!;a1이므로 a+b=;2#;
지나는 직선의 기울기를 나타낸다.

46 정답과 풀이
참고 y=2'ßt-a-2 (t>-1)와 y=t의 그래프는 다음과 같다.
a>1일 때는 'x=ax, a€x€-x=0에서 D=1>0이므로 교점이 2개이다. y y=t
y=2 t-a-2

y=2 t+2-2

26 ^ 10 -1
O t
GUIDE
-1
P(t€, t)로 놓고 CP’를 t에 대한 식으로 나타낸다. 2 -1-a-2
-2

y='x 그래프 위의 점 P(t €, t)에 y

대하여 P
y=(g@f )(x)와 y=f(x) 그래프가 서로 다른 4개의 실근을 갖
CP’="ƒ(t€-3)€+t€ Q 기 위해서는 t>-1에서 서로 다른 두 실근을 가져야 한다.
C
="ƒt›-5t€+9 O 3 x 1 t=-1일 때 2'ß-1-a-2<-1이어야 하므로
'ß11
=æ{t€-;2%;}€+:¡4¡:> -1-a<;4!;  -;4%;<a
2
따라서 CP’와 원의 교점을 Q라 하면 PQ’가 두 점 P, Q 사이 거 2 y=2'ßt-a-2 (t>-1)와 y=t가 접할 때 a값은
리의 최솟값이다. 이때 PQ’의 최솟값은 2'ßt-a=t+2, 4(t-a)=t €+4t+4
'ß11 t €+4+4a=0
CP’-CQ’= -1에서 a=11, b=-1이므로
2 D=-(4+4a)=0  a=-1
a+b=10 따라서 a<-1이어야 두 점에서 만난다.

다른 풀이 판별식 이용하기 1, 2에서 -;4%;<a<-1이므로

원 C의 반지름 길이를 r라 하면 원의 방정식은 (x-3)€+y€=r€


a=-;4%;, b=-1  a+b=-;4(;
원과 y='x의 교점의 x좌표는 (x-3)€+('x )€=r€의 근이다.
방정식 x€-5x+9-r€=0이 중근을 가지므로
1등급 NOTE
'ß11
D=25-36+4r€=0, r€=:¡4¡:, r= y=f(x)는 이차함수이고 f(x)-1이므로
2
k>-1인 k에 대하여 f(x)=k는 서로 다른 두 실근을 갖는다.

27 ^④
GUIDE
h(x)=(g@ f )(x)를 구하고 분모 유리화를 이용해 정리한다.
29 ^ -1
GUIDE
f(k)='ß2k-1+'ß2k+1이고
함수 f(x)의 역함수가 존재하려면 함수 f(x)가 일대일대응이어야 하므
4 로 x2일 때 치역을 구해 x<2일 때 함수의 모양을 생각한다.
h(k)= =2('ß2k+1-'ß2k-1 )이므로
'ß2k-1+'ß2k+1
h(1)+h(2)+h(3)+y+h(40) 함수 f(x)의 역함수가 존재하려면 실수 전체에서 일대일대응이
=2('3-'1 )+2('5-'3 )+y+2('ß81-'ß79 ) 어야 한다.
=2_(-'1+'ß81 )=16 x2일 때 f(x)='ßx-2+1이므로 f(x)>1
x<2일 때 f(x)<1인 일대일대응이 되어야 한다.
점근선이 x=2, y=1일 때, y
일대일대응이 되므로 y= x-2+1
28 ^①
k
GUIDE f(x)= +1 1
x-2
(g@f)(x)=f(x)에서 f(x)=t로 치환하여 g(t)=t에서 교점의 개수 O 2 x
를 살펴본다. 이때 t값의 범위에 주의한다.
x-2+k x
= =
x-2 ax+b

f(x)=(x+1)€-1-1이고 에서
(g@f )(x)=f(x)에서 f(x)=t (t>-1)로 치환하면 k=2, a=1, b=-2
g(t)=t이므로 2'ßt-a-2=t  a+b=-1

5. 유리함수와 무리함수 47
30 ^③ 02 ^④
GUIDE GUIDE
-1
y='ßx-1의 역함수는 y=x€+1이고, 이 함수의 그래프는 y=x€의 그 함수 f(x)의 역함수가 존재할 때, f( f(x))=x이면 f(x)=f (x)임을
래프와 포개어진다. 즉 두 곡선 바깥 부분에서 넓이가 서로 같은 것을 생 이용한다.
각할 수 있다.
-1
㈐에서 f=f 이므로 y=f(x) y
y y 의 그래프는 y=x에 대하여 대
(1, 2) (1, 2) 칭이다. (7, 7)
(-2, 4) (-2, 4) (1, 1)
(5, 2) ⇨ (5, 2)
㈎, ㈏에서 함수 y=f(x)의 그래
프는 두 점 (1, 1), (7, 7)을 지난 O x
O x O x
다. 이때 두 점 (1, 1), (7, 7)이
위 그림처럼 생각하면 삼각형 1개와 직사각형 1개로 생각할 수 y=x 위의 점이므로 대칭점은
있다. 따라서 넓이는 두 점을 이은 선분의 중점 (4, 4)이고 점근선의 교점과 같다.
;2!;_{5-(-2)}_(4-2)+{1-(-2)}_2=13 k
 f(x)= +4
x-4
이 그래프가 (1, 1)을 지나므로 k=9
9
즉 f(x)= +4이므로 f(5)=13
x-4

03 ^ 22
GUIDE
분수함수 y=f(x)의 그래프가 두 점근선의 교점 (m, n)에 대하여 대칭
STEP 3 1등급 뛰어넘기 p. 68~71
이므로 a+b=2m, f(a)+f(b)=2n을 이용한다.
01 ④ 02 ④ 03 22 04 ①
05 ④ 06 ⑤ 07 6개 08 ③
10x 105 21
f(x)= = +5이므로 y=f(x)는 점 { , 5}
09 2 10 3개 11 ;4#;<m<1 12 11 2x-21 2x-21 2

1 에 대칭이다.
13 ③ 14 ⑴ y=-t €x+t ‹- ⑵ 4개 ⑶ 2a€p
t
21
따라서 { , 5}에 대하여 대칭인 두 점 P(a, f(a)), Q(b, f(b))
2
01 ^④
GUIDE 에서 a+b=21, f(a)+f(b)=10이 성립하므로
두 점 P, Q의 좌표를 문자로 나타내고, 직사각형의 가로 길이와 세로 길 1+20=21에서 f(1)+f(20)=10
이를 이용해 넓이를 구한다. 2+19=21에서 f(2)+f(19)=10

y
yy
10+11=21에서 f(10)+f(11)=10
P{a, ;a$;}  f(1)+f(2)+y+f(20)=100
;a$;+;b(; y=;x$;
한편 f(21)=10, 5<f(22)<10이므로
O x
f(1)+f(2)+y+f(21)=110,
a+b
Q{-b, -;b(;} f(1)+f(2)+y+f(22)>115
따라서 m의 최솟값은 22
y=;x(;

참고 y
P{a, ;a$;}, Q {-b, -;b(;}라 하면 a>0, b>0이고
오른쪽 그림에서 ;;™2¡;;-a=b-;;™2¡;;, Q(b, f(b))
가로 길이는 a+b, 세로 길이는 ;a$;+;b(; 인 직사각형의 넓이는 {21
2 5{
,
5-f(a)=f(b)-5이므로
a+b=21, f(a)+f(b)=10임을
4b 9a O x
(a+b){;a$;+;b(;}=4+9+ + 13+2'ß4_9=25 P(a, f(a))
a b 알 수 있다.

이므로 최솟값은 25

48 정답과 풀이
04 ^① 06 ^⑤
GUIDE GUIDE
분수함수의 그래프가 점근선의 교점을 지나고 기울기가 -1인 직선에 대 xy-2=0과 xy+2x-2y-3=0을 연립한
하여 대칭임을 이용한다. k(xy-2)+xy+2x-2y-3=0에서 생각한다.

3x-12 상수 k에 대하여 k(xy-2)+xy+2x-2y-3=0에서


y= 와 y=-2x+23의 교점은 (7, 9)와 (9, 5)이다.
x-6
ㄱ. k=-1일 때, 2x-2y-1=0이고, 이것이 교점을 지나는 직
3x-12 6
y= = +3에서 대칭의 중심이 (6, 3)이고 선이다. y=x-;2!; ( ◯ )
x-6 x-6
(0, 2)와 (7, 9), (4, 0)과 (9, 5)는 각각 y=-(x-6)+3, 즉
ㄴ. xy=2이고, 2x-2y-1=0에서 x-y=;2!;이므로
y=-x+9에 대하여 대칭이다.
y=-2x+23 17
y x€+y€=(x-y)€+2xy=;4!;+4=
4
(7, 9) 17 17
따라서 x€+y€= 이므로 원의 넓이는 p(◯)
4 4
(9, 5)
3 A B ㄷ. k(xy-2)+xy+2x-2y-3=0에 x=-1, y=-1을 대
(0, 2) 입하면 -k-2=0, k=-2
O (4, 0) 6 x
따라서 -2(xy-2)+xy+2x-2y-3=0이므로

y=-x+9 2x+1
y= (◯)
x+2
따라서 사각형 넓이를 구하면 되므로
LECTURE
;2!;(2+8)_6-;2!;_4_2=26
f(x, y)=0, g(x, y)=0의 교점을 지나는 도형의 방정식은
f(x, y)+kg(x, y)=0이다. k=-1이면 두 도형의 교점을 지나는 직선
1등급 NOTE
의 방정식이 된다.
3x-12
판별식을 이용해 직선 y=-2x+23과 y= 가 서로 다른 두 점
x-6
에서 만나는지 확인하고 B 영역의 대칭이동을 생각한다.
07 ^ 6개
GUIDE
05 ^④
➊ 점 P가 정사각형의 중심이므로 B의 x좌표는 t-1이고, D의 x좌표는
GUIDE
t+1이다.
y=-'ß-x+a+a를 먼저 생각한다. y=|a-'ßa-x|-2의 그래프에
서 꼭짓점과 y절편이 어떤 위치에 있을 때 조건을 만족시키는지 확인한 ➋ 함수 y=;x@;+1의 그래프가 정사각형 ABCD와 만나는 경우는 점 B

다. 를 지날 때와 점 D를 지날 때 사이에서 생각할 수 있다.

y=|a-'ßa-x|-2의 그래프는 y=-"ƒ-(x-a)+a에 절댓 P(t, 't )일 때, B(t-1, 't -1), D(t+1, 't +1)
값을 씌우고 y축 방향으로 -2만큼 평행이동한 그래프이다.
1 점 D가 y=;x@;+1의 그래프 위에 있을 때
이 그래프가 모든 사분면을 지나려면 그림과 같이 a>0이어야
한다. 2
+1='t +1, 4=t(t+1)€
t+1
y
t ‹+2t €+t-4=0, (t-1)(t €+3t+4)=0 t=1
y=| -(x-a)+a| (a, a)
2 점 B가 y=;x@;+1의 그래프 위에 있을 때
(a, a-2)
O x 2 2t
+1='t -1, ='t
a-'a-2 t-1 t-1
y=| -(x-a)+a|-2 4t €=t(t-1)€, t(t €-6t+1)=0 t=0, t=3+2'2
( t=3-2'2 일 때, 정사각형과 만나지 않는다.)
(a, a-2)가 제1사분면에 있어야 하므로
1, 2에서 t=0, 1t3+2'2 이므로
a-2>0에서 a>2 yy ㉠
정수 t는 0, 1, 2, 3, 4, 5로 6개
(y절편)<0이므로 a-'a-2<0에서 'a=t로 치환하면
t €-t-2<0, (t-2)(t+1)<0 참고

즉 -1<'a<2이므로 0a<4 yy ㉡ 풀이 결과에서 t=0일 때는 점 B가 분수함수의 왼쪽 부분 위에 있으며,

㉠, ㉡에서 2<a<4이므로 정수 a값은 3 1<t<3+2'2일 때는 분수함수의 오른쪽 부분이 선분 BD와 만난다.

5. 유리함수와 무리함수 49
1등급 NOTE t-4 -8
y= = +1 (t4k) y
t+4 t+4
함수 y=;x@;+1의 그래프는 고정되어 있고 정사각형 ABCD는 중심이 t=4일 때 y=0이고

y='x를 따라 움직인다. 따라서 정사각형이 함수 y=;x@;+1의 그래프와 t>4일 때 0<y<1이므로


1 y=1

만나는 범위를 생각할 수 있는데, 이때 특이점(critical point)은 점 B와


y는 정수값을 가지지 않는다. -4 O 4 t

점 D이다. 즉 4k>4에서 자연수 k의


최솟값은 2

08 ^③
GUIDE
그래프의 개형이나 식을 연립하여 구한 이차방정식의 판별식을 이용한다. 10 ^ 3개
GUIDE
ㄱ. a=-1이면 y=2이다. 직선 PQ의 기울기를 n, k로 나타내고 문제에서 주어진 범위를 만족시키
는 자연수 n, k의 값을 n=1일 때부터 따져 본다.
따라서 k=0이면 그래프가 만나지 않는다. ( _ )
ㄴ. a=5일 때 'ß6x-2=kx, k€x€-6x+2=0 'ßn+k-'n
직선 PQ의 기울기는 이므로
3'2 k
;;4Î;;=9-2k€>0에서 0<k< 이므로
2 'ßn+k-'n
;3!; ;2!; y
정수 k는 1, 2이고, 합은 3 ( _ ) k y='x
ㄷ. y="ƒ(a+1)x+3-a의 그래프가 다음 두 가지 경우와 같은 1
;3!; ;2!; Q(n+k , n+k )
모양일 때 k값에 상관없이 하나의 실근을 가진다. 'ßn+k+'n
2'ßn+k+'n3 P(n, 'n)
y y
O x
2-'n'ßn+k3-'n
n=1일 때 0k3에서 자연수 k는 1, 2, 3으로 3개
a-3 O x O a-3 x
a+1 a+1 n2일 때 자연수 k가 존재하지 않는다.
[그림 1] [그림 2] 따라서 조건에 맞는 순서쌍은 (1, 1), (1, 2), (1, 3)으로 모두 3개
a-3
[그림 1]의 경우 a+1>0, <0 ⇨ -1<a<3
a+1

11
a-3
[그림 2]의 경우 a+1<0, >0 ⇨ a<-1 ^ ;4#;<m1
a+1
따라서 정수 a의 최댓값은 2이다. ( ◯ ) GUIDE
y=1-"ƒ-x€+2x 로 놓고 이 식을 변형해 그래프가 원의 아래쪽 반원
임을 이용한다.
1등급 NOTE

3'2 3'2 y=1-"ƒ-x€+2x 라 하면 y


ㄴ의 ;;4Î;;>0에서 - <k< 이므로 y
2 2 y= 6x-2
조건에 맞는 정수 k를 -2, -1, 0, 1, 2로 y-1=-"ƒ-x€+2x 에서 (1, 1) (2, 1)
1
구할 수 있지만 오른쪽 그림에서 생각하면 O 양변을 제곱하면
x
k>0일 때만 y=kx와 y='ß6x-2의 그래 y=kx (y-1)€=-x€+2x
O x
프가 두 점에서 만난다.
즉 (x-1)€+(y-1)€=1이므로 -1
등식의 좌변은 (1, 1)이 중심이
고 반지름 길이가 1이며 y1
09 ^2
GUIDE 인 반원을 나타낸다. 이때 우변은 (0, -1)을 지나는 직선이므로
4k€ 1 두 도형이 접할 경우는 y=mx-1과 (1, 1)의 거리가 1일 때
x+ -4
x€-4x+4k€ x
y= = 에서 치환할 부분을 찾는다. 이다. 즉
x€+4x+4k€ 4k€
x+ +4
x |m-1-1|
=1  m=;4#;
"ƒm€+1
4k€
x+
x
-4 2 두 점에서 만나는 경우에서 m의 최댓값은 y=mx-1이
4k€
y= 에서 t=x+ 으로 치환하면
4k€ x (2, 1)을 지날 때이다. 즉
x+ +4
x
2m-1=1 m=1
4k€ 4k€
x+ 2Ƙx_ =4k 1, 2에서 ;4#;<m<1
x x

50 정답과 풀이
12 ^ 11 1 y=-t €x+t ‹-
1
t
과 y=x의 교점은
GUIDE
y>0이므로 y='x+'ß18-x의 양변을 제곱하여 y€의 범위에서 생각한다. t ›-1 t €-1 1
(t €+1)x= 에서 x= =t- 이므로
t t t
y0이므로 1 1
원의 중심은 {t- , t- }이다.
t t
y€=18+2"ƒ-x€+18x=18+2"ƒ81-(x-9)€
(x-9)€=X, y€=Y라 하면 1 1 1
Y {t- , t- }과 원점 사이의 거리 a는 a='2 |t- |
t t t
Y=2"ƒ81-X+18 36
1
(0X81) 2 y=-t €x+t ‹- 과 y=-x의 교점은
(81, 18) t
이때 18<Y<36, O X t ›-1 1
(t €-1)x= 에서 x=t+ 이므로 원의 중심은
즉 18y€36에서 t t
3'2y6이므로 정수 y값은 5, 6  5+6=11 {t+
1 1 1
, -t- }이다. 이때 a='2 |t+ |
t t t
a>2'2 이므로 중심이 y=x와 y=-x 위에 있는 것 각각 2

13 ^③ 개씩, 모두 4개의 원을 구할 수 있다.


y
GUIDE y=x
A+B 값이 일정할 때 "|A+"|B의 최댓값, 최솟값을 구하려면
코시-슈바르츠 부등식을 이용한다.
a
A(1, 0), B(a, 'ß2a ), C(b, 'ßb-2 )이므로 a
O x
삼각형 ABC의 무게중심 G의 좌표는
1+a+b 0+'ß2a+'ßb-2 'ß2a+'ßb-2
{ , }, 즉 {2, }
3 3 3
X='a, Y='ßb-2 라 하면 X €+Y €=a+b-2=5-2=3
y=-x
'ß2a+'ßb-2='2 X+Y이므로 코시-슈바르츠 부등식에서
⑶ ⑵에서
(('2 )€+1€)(X €+Y €)('2 X+Y)€
1 1
 -3'2 X+Y3 1 의 경우 반지름 길이는 원의 중심 {t- , t- }과
t t
즉 'ß2a+'ßb-2의 최댓값은 3이므로 1 1
접접 {t, - } 사이의 거리이므로 æt€+
'ß2a+'ßb-2 t t€
m+n=2+ 2+;3#;=3
3 1 1
이때 a='2 |t- |이고, a€=2 {t €+ -2} 에서
t t€
1
t €+ =;2!;a€+2
t€
14 ^ ⑴ y=-t €x+t ‹-
1
t
⑵ 4개 ⑶ 2a€p
1
 (원의 넓이)={t €+ }p={;2!;a€+2}p
GUIDE t€
y=-;x!;의 그래프가 y=x, y=-x에 대칭이므로 y=-;x!;의 그래프와 1 1 1
2 의 경우 반지름 길이는 {t+ , -t- }과 {t, - }
t t t
두 점에서 접하는 원의 중심은 y=x, y=-x 위에 있다.
1
사이의 거리이므로 æt€+
t€
1
⑴ 함수 y=-;x!; 위의 점 {t, - }에서 그은 접선의 방정식을
t 1
이때 a='2 |t+ |이므로
t
1
y=m(x-t)- 이라 하면
t 1 1
a€=2 {t €+ +2}, t €+ =;2!;a€-2
t€ t€
1 1
-;x!;=m(x-t)- 에서 mx€-{mt+ }x+1=0
t t 1
 (원의 넓이)={t €+ }p={;2!;a€-2}p
t€
1 € 1 1
D={mt+ } -4m={mt- }€=0  m= a>2'2 일 때, 가능한 원은 1에서 2개, 2에서 2개 있으므로
t t t€
1 넓이의 총합은 2{;2!;a€+2}p+2{;2!;a€-2}p=2a€p
따라서 접선에 수직인 직선의 방정식은 y=-t €x+t ‹-
t
⑵ ⑴에서 구한 직선과 y=x, y=-x와의 교점이 원의 중심이   f(a)=2a€p
된다.

5. 유리함수와 무리함수 51
06 경우의 수와 순열 1등급 NOTE

‘교란수’를 이용하면 시간을 아낄 수 있다.

STEP 1 1등급 준비하기 p. 74 ~75 이 문제의 경우는 n=5일 때이므로


1 1 1 1 1
5!{1- + - + - }=60-20+5-1=44
01 40 02 12 03 44 04 ④ 1! 2! 3! 4! 5!
05 ④ 06 ④ 07 300 08 12
n 1 2 3 4 5 6
09 ⑤ 10 4명 11 3개
교란수 0 1 2 9 44 265

01 ^ 40
GUIDE
두 가지 이상의 사건이 함께 일어나는 경우에는 곱의 법칙을 이용하고, 04 ^④
함께 일어나지 않는 경우에는 합의 법칙을 이용한다. GUIDE
지불할 수 있는 금액의 수를 구할 때 작은 단위의 화폐 금액의 합이 큰 단
1 P # R:2 위보다 크면 작은 단위의 돈으로 바꾼 것을 생각한다.
2 P # Q # R:2_3=6
3 P # S # R:3_2=6 50원 3개, 100원 3개, 500원 3개이므로

4 P # Q # S # R:2_2_2=8 1 지불할 수 있는 경우의 수

5 P # S # Q # R:3_2_3=18 4_4_4-1=63  a=63

따라서 2+6+6+8+18=40 2 지불할 수 있는 금액의 수


50원이 2개면 100원이므로 100원은 50원으로 바꾸고, 50원,
100원을 다 모아도 500원이 안 되므로 500원은 그냥 둔다.
02 ^ 12 즉 50원 9개와 500원 3개가 있는 것과 같으므로
GUIDE 10_4-1=39  b=39
(1+x€+x‹)(1+x+x€+x—)을 먼저 계산하고, 전체에서 x10이 나오는
1, 2에서 a+b=63+39=102
경우를 따져 본다.

1등급 NOTE
(1+x+y+x10)(1+x€+x‹)(1+x+x€+x—)
(작은 단위의 화폐 금액의 합)>(큰 단위의 화폐)일 때
=(1+x+y+x10)(1+x+2x€+2x‹+2x›+2x—+x‡+x°)
큰 단위의 화폐를 작은 단위로 바꾼다고 생각한다.
이므로 x10의 계수는
① 50원 1개, 100원 3개, 500원 3개
1+1+2+2+2+2+1+1=12 (환전 불필요) ⇨ 2_4_4-1=31
② 50원 3개, 100원 3개, 500원 3개
(100원 환전):50원 9개, 500원 3개
03 ^ 44 ⇨ 10_4-1=39
GUIDE ③ 50원 5개, 100원 3개, 500원 3개
a¡+1, a™+2, a£+3, a¢+4, a+5가 되도록 하는 경우이다. 수형도를 (100원, 500원 환전):50원 41개 ⇨ 41
이용하여 가능한 경우를 모두 따져 본다.
※ 교란수를 이용한다.
05 ^④
ai+i여야 하므로 다음과 같이 a¡=2일 때 가능한 경우는 11가지 GUIDE
이다. 같은 색을 여러 번 쓸 수 있고, 이웃한 부분에는 다른 색을 칠할 때, 먼저
칠한 영역 중 서로 이웃하지 않는 영역의 색이 같은지 다른지에 따라 달
a¡ a™ a£  a¢  a
4 ! ! 5 ! ! 3 라진다.
1
5 ! ! 3 ! ! 4
1 ! ! 5 ! ! 4 E D E D
3 ! ! 4 ! ! 5 ! ! 1 A A
! ! ! ! C C
2
5 1 4 B B
1 ! ! 5 ! ! 3
4 1 ! ! 3
5
3 ! ! 1 색을 한 번씩만 사용하는 경우의 수
1 ! ! 3 ! ! 4
5 p=5_4_3_2_1=120 E D
1 ! ! 3
4 A
3 ! ! 1 같은 색을 여러 번 사용할 수 있을 때
C
B
a¡=3, 4, 5일 때도 마찬가지이므로 4_11=44 A에 칠하는 경우의 수:5

52 정답과 풀이
1 B, D에 같은 색을 칠하는 경우 08 ^ 12
B, D를 칠하는 경우의 수 4 GUIDE
C, E를 칠하는 경우의 수 3_3=9 1089!=108_(108-9)_(108-2_9)_y_(108-11_9)
=9_18_y_108
 4_9=36
2 B, D에 다른 색을 칠하는 경우 108ª!
B, D를 칠하는 경우의 수 4_3=12 36£!
C, E를 칠하는 경우의 수 2_2=4 =
108_(108-9)_(108-2_9)_y_(108-11_9)
36_(36-3)_(36-2_3)_y_(36-11_3)
  12_4=48
108_99_90_81_y_9
따라서 q=5(36+48)=420이므로 |p-q|=300 =
36_33_30_27_y_3
=3_3_3_y_3=312
다른 풀이
 a=12
q를 다음과 같이 구할 수 있다.

A를 칠하는 경우의 수 5, B를 칠하는 경우의 수 4


C를 칠하는 경우의 수 3, 이때 D를 칠하는 경우의 수는
09 ^⑤
① B와 같은 색을 칠하는 경우의 수 1
GUIDE
이때 E를 칠하는 경우의 수 3 홀수 번째 자리인 만의 자리, 백의 자리, 일의 자리에 홀수를 배열하는 경
② B와 다른 색을 칠하는 경우의 수 2 우의 수와 짝수 번째 자리인 천의 자리, 십의 자리에 짝수를 배열하는 경
이때 E를 칠하는 경우의 수 2 우의 수를 생각한다.

따라서 q=5_4_3_(1_3+2_2)=420
홀 짝 홀 짝 홀 로 배열해야 하므로
홀수 1, 3, 5, 7, 9 중에서 세 개를 세 자리에 배열하는 경우의 수는
P£=5_4_3=60
06 ^④
짝수 2, 4, 6, 8 중에서 두 개를 두 자리에 배열하는 경우의 수는
GUIDE
¢P™=4_3=12
맨 앞자리에 올 수 있는 것부터 따져 차례로 개수를 센다.
따라서 60_12=720
1 꼴:5_4_3=60, 2 꼴:5_4_3=60
3 꼴:5_4_3=60, 41 꼴:4_3=12
42 꼴:4_3=12, 431 꼴:3
10 ^ 4명
432 꼴:3, 435 꼴:3 GUIDE
60+60+60+12+12+3+3+3=213이고, 4361은 이웃하는 것들을 하나로 묶어 배열하는 경우의 수를 구하고, 그 묶음 안

436 꼴의 첫 번째 수이므로 214번째로 작은 자연수이다. 에서 자리를 바꾸는 경우의 수를 곱한다.

아이 4명을 하나로 생각하여 어른 n명과 배열하는 경우의 수는


(n+1)!_4!
07 ^ 300
이때 (n+1)!_4!=2880에서 (n+1)!=120이고,
GUIDE
120=5_4_3_2_1이므로 n+1=5  n=4(명)
1 1 1 1
P2=n(n-1)이므로 = = -
n
nP2 n(n-1) n-1 n 따라서 4명

1 1 1 1
{ + + +y+ }_m
™P™ £P™ ¢P™ ™ºP™

={
1
+
1
+
1
+y+
1
}_m 11 ^ 3개
2_1 3_2 4_3 20_19 GUIDE
➊ 전체 경우의 수에서 반대 경우인 ‘양쪽 끝에 모두 자음이 오는 경우’를
={;1!;-;2!;+;2!;-;3!;+;3!;-;4!;+y+;1¡9;-;2¡0;}_m
제외한다.
➋ 모음이 n개 있다고 하면 자음은 7-n개 있다.
={;1!;-;2¡0;}_m=;2!0(; m
※ 자음이 n개 있다고 놓고 풀어도 된다.
19
즉 m이 두 자리 자연수가 되도록 하는 자연수 m은 적어도 한쪽 끝에 모음이 오는 경우는 7개를 배열하는 전체 경우
20
20, 40, 60, 80, 100이므로 합은 300 에서 양 끝에 자음이 오는 경우를 뺀 것과 같다.

6. 경우의 수와 순열 53
즉 7!-7-nP™_5!=3600에서 (7-n)(6-n)=12를 정리하면 03 ^②
(n-3)(n-10)=0  n=3 ( n<7) GUIDE
따라서 3개 1 420을 소인수분해 한다.
2 각 소인수들을 어떻게 나누어야 하는지 생각한다.

420=2€_3_5_7이므로 {B 3, 5, 7}
420=a_b(a, b는 서로소)
2€ _
라 하면 소인수 2, 3, 5, 7은 a나 b
중 한 군데에만 들어가야 한다.
이때 2€이 들어간 수를 a라고 하면, 3이 들어갈 곳을 정하는 경우
는 a 또는 b 두 가지이고, 마찬가지로 5와 7이 들어갈 곳을 정하
STEP 2 1등급 굳히기 p. 76~81 는 경우도 각각 두 가지이므로 구하는 경우의 수는

01 ③ 02 ④ 03 ② 04 81 2_2_2=8
05 ⑤ 06 504 07 100 08 20
09 ② 10 1440 11 ④ 12 ② 참고

13 5140개 14 768개 15 ④ 16 ④ 서로소인 두 자연수의 곱으로 나타내는 것이므로 a=2€_3, b=5_7과


17 ③ 18 ⑤ 19 ② 20 40개 a=5_7, b=2€_3은 같은 경우이다. 따라서 2€을 기준으로 2€에 곱해
21 70개 22 546 23 ④ 24 318개 지는 소인수가 어떤 종류인지만 파악하면 된다.
25 ⑤ 3을 기준으로 생각해도 답은 같고, 5 또는 7을 기준으로 풀어도 마찬가지
이다.
01 ^③
GUIDE
어떤 두 원의 접점에서 다음 접점으로 가는 경로가 몇 가지인지 따져 본다.
04 ^ 81
GUIDE
A ➊ (홀)+(짝)=(홀), (홀)_(짝)=(짝)이다.
C
➋ a, b, c 중 짝수, 홀수가 몇 개씩 있을 때 a+b+c+abc가 홀수가 되
D
는지 따져 본다.
B

E F 1 a, b, c가 모두 짝수일 때
abc와 a+b+c가 짝수이므로 a+b+c+abc는 짝수

그림처럼 두 원의 접점을 C, D, E, F라 하자. 이때 2 a, b, c 중 홀수가 1개, 짝수가 2개일 때

A # C # D # E # F # B에서 각 단계마다 선택할 수 있는 abc는 짝수, a+b+c는 홀수이므로 a+b+c+abc는 홀수

경로는 2개씩 있고, B에서 A로 돌아올 때는 지나왔던 길과 다른 3 a, b, c 중 홀수가 2개, 짝수가 1개일 때

길로 와야 하므로 1가지 경로뿐이다. 따라서 abc와 a+b+c가 짝수이므로 a+b+c+abc는 짝수

2_2_2_2_2_1_1_1_1_1=2— 4 a, b, c가 모두 홀수일 때
abc와 a+b+c가 홀수이므로 a+b+c+abc는 짝수
즉 2일 때의 경우의 수를 구한다.

02 ^④ a가 홀수, b, c가 짝수일 때는 3_3_3=27(가지)


GUIDE b, c만 각각 홀수일 때도 마찬가지로 27가지이므로
1 2a_5b의 약수 개수를 a, b에 대한 식으로 나타낸다. 경우의 수는 27+27+27=81
2 가능한 (a, b)의 개수를 구한다.

n=2a_5b의 약수는 (a+1)(b+1)(개)이므로


(a+1)(b+1)=18에서 05 ^⑤
GUIDE
a+1 1 2 3 6 9 18 x+f(x)가 홀수이려면
b+1 18 9 6 3 2 1 1 x가 홀수일 때 f(x)는 짝수
2 x가 짝수일 때 f(x)는 홀수
즉 (a, b)는 (0, 17), (1, 8), (2, 5), (5, 2), (8, 1), (17, 0)으로
6개이다. 1 x가 1, 3, 5일 때
따라서 구하는 자연수 n은 6개 f(1), f(3), f(5)는 2, 4 중 하나여야 하므로 2_2_2=8

54 정답과 풀이
2 x가 2, 4일 때 참고

f(2), f(4)는 1, 3, 5 중 하나여야 하므로 3_3=9 원소 1, 2, 3, 4 각각이 들어가는 구역에 따라 집합 A, B, C는 유일하므


1, 2에서 8_9=72(개) 로 순서쌍 (A, B, C)의 개수도 100개이다.

참고

두 자연수를 더한 결과가 홀수인 경우는 홀수와 짝수를 더했을 때이다.


08 ^ 20
GUIDE
면을 넣고 파를 넣기 전에 들어가는 다른 재료의 개수가 0, 1, 2, 3인 경우
로 나누어 생각한다.

06 ^ 504
1 면과 파를 잇달아 넣는 경우
GUIDE
처음에 자동차 A에 탔던 운전자를 제외한 두 명을 p, q라 할 때 다음과 면 !# 파 !# b !# b !# h
같이 경우를 나누어 생각한다. b !# 면 !# 파 !# b !# h
1 두 사람 모두 처음에 두 사람이 앉은 자리를 제외한 자리에 앉을 때  b !# b !# 면 !# 파 !# h
2 p가 q의 자리에 앉을 때
b !# b !# h !# 면 !# 파
3 q가 p의 자리에 앉을 때
이때 b b를 채우는 경우의 수는 2이므로 8가지 경우가 있
처음에 A에 탔던 운전자를 제외한 두 명을 p, q라 할 때 다. (h는 계란을 나타낸다.)
1 두 사람 모두 두 사람이 앉은 자리를 제외한 자리에 앉을 때 2 면과 파 사이에 다른 재료를 1개 넣는 경우
p, q가 먼저 4자리 중 2자리에 앉고, B에 탔던 4명이 나머지 면 !# b !# 파 !# b !# h
4자리에 앉으면 되므로 ¢P™_4!=288 b !# 면 !# b !# 파 !# h
2 p가 q의 자리에 앉을 때 q와 B에 탔던 4명이 남은 자리에 앉 b !# b !# 면 !# h !# 파
으면 되므로 5!=120 마찬가지로 생각하면 6가지 경우이다.
3 q가 p의 자리에 앉을 때 2와 마찬가지로 120 3 면과 파 사이에 다른 재료를 2개 넣는 경우
이때 2, 3에서 p, q가 서로 자리를 바꿔 앉은 경우인 면 !# b !# b !# 파 !# h
4!=24(가지)가 중복되었으므로 b !# 면 !# b !# h !# 파
288+120+120-24=504 마찬가지로 생각하면 4가지 경우이다.
4 면과 파 사이에 다른 재료를 3개 넣는 경우
주의
면 !# b !# b !# h !# 파
경우의 수의 합을 구할 때 두 사건에서 중복된 사건은 두 번 더해졌으므 에서 2가지 경우이다.
로 한 번 제외해야 한다. 합집합의 원소의 개수를 구하는 방법과 같다.
따라서 8+6+4+2=20

다른 풀이

분말수프, 건더기수프, 계란이 들어갈 자리를 a, 면과 파가 들어


07 ^ 100
갈 자리를 b라 하면
GUIDE
세 집합 A, B, C에 대한 벤 다이어그램을 그리고, 각 원소가 선택할 수 5!
a, a, a, b, b를 일렬로 배열하는 경우의 수는 =10
3!_2!
있는 영역을 따져 본다.
이때 분말수프와 건더기수프는 계란보다 앞에 있어야 하므로
㈎에서 ACBCC=S이므로 A 분말수프 !# 건더기수프 !# 계란
원소 1, 2, 3, 4는 반드시 a
건더기수프 !# 분말수프 !# 계란
a, b, c, d, e, f, g 중 한 구역에 들어가야 b d 의 두 가지 경우가 있고, 파는 면보다 뒤에 있어야 하므로
c
한다. e f
g 면 !# 파
B C
㈏에서 ADB={1, 2}이므로 의 한 가지 경우가 있다.
1과 2가 각각 선택할 수 있는 구역은 b 또는 c 따라서 10_2_1=20(가지)
⇨ 2_2=4(가지)
LECTURE 같은 것이 있는 순열(확률과 통계 과정)
3과 4는 b와 c를 제외한 나머지 5구역 중 한 구역에 들어가면 되
n개 중에 서로 같은 것이 각각 p개, q개, y, r개씩 있을 때, 이 n개를 한
므로 5_5=25(가지)
n!
줄로 배열하는 경우의 수는 이다. (단, p+q+y+r=n)
 4_25=100(가지) p!q!yr!

6. 경우의 수와 순열 55
09 ^② 11 ^④
GUIDE GUIDE
다음과 같이 경우를 나누어 생각한다. 네 자리 자연수를 abcd 라 하면 a는 1, 2, 3, 4, 5, 6, 7 중 하나이고, d 는 1,
1 3번 공을 A¡에 넣을 때 3, 5, 7, 9 중 하나이다.
2 3번 공을 A¡에 넣지 않을 때 ➊ a가 홀수인 경우와 짝수인 경우로 나누어 생각한다.
➋ a # d # b # c 순으로 선택한다고 생각한다.
1 3번 공을 A¡에 넣을 때
2, 4, 5, 6번 공을 자기 번호가 적히지 않은 곳에 넣으면 된다. 조건에 맞는 네 자리 자연수를 abcd 라 하자.
즉 n=4일 때 교란수와 같으므로 9 1 a가 홀수, 즉 1, 3, 5, 7 중 하나이면 d 가 되는 경우의 수는 4
2 3번 공을 A¡에 넣지 않을 때 b, c가 되는 경우의 수는 8P2=56

3번 공이 A¡에 들어 가지 않는 경우이므로 A¡을 A£으로 생각 즉 4_4_56


하면 2, 3, 4, 5, 6번 공을 자기 번호가 적히지 않은 곳에 넣는 2 a가 짝수, 즉 2, 4, 6 중 하나이면 d 가 되는 경우의 수는 5

것과 같다. 이때 b, c가 되는 경우의 수는 마찬가지로 8P2=56


즉 n=5일 때 교란수와 같으므로 44 즉 3_5_56
1, 2에서 경우의 수는 9+44=53 1, 2에서 4_4_56+3_5_56=31_56=1736

12 ^②
GUIDE
10 ^ 1440 네 자리 자연수 abcd 를 1000a+100b+10c+d와 같이 각 자리의 수를
GUIDE 따로 생각하고, 다음을 이용한다.
1 4쌍의 동물을 (a, a'), (b, b'), (c, c'), (d, d ')이라 하자. 예 12+21=10+2+20+1=10+1+20+2=11+22
2 먼저 한 우리에 두 종류의 동물을 넣어 단순화한다. =(1+2)_11
3 우리에 먼저 넣은 동물과 같은 종류의 동물이 들어갈 우리를 결정한다.
1은 다음과 같이 나타난다.
4_3 1 꼴:3_2_1=6(개)
1 A에 넣을 동물 두 종류를 고르는 경우의 수는 =6
2
1 꼴:3_2_1=6(개)
2 1에서 고른 동물 종류에서 각각 한 마리씩 고르는 경우의 수는
1 꼴:3_2_1=6(개)
2_2=4
1 꼴:3_2_1=6(개)
3 이 두 동물을 a, b라 하면
따라서 1이 나타내는 모든 수의 합은 1111_6=6666
a'이 들어갈 우리를 고르는 경우의 수는 3
2, 3, 4에 대해서도 마찬가지이므로
① 이때 b'이 a'과 같은 우리에 들어가는 경우
모든 수의 합은
나머지 4마리의 동물을 2개의 우리에 넣는 경우의 수는 4
(1111+2222+3333+4444)_6
② b'이 a'과 다른 우리에 들어가는 경우
=(1+2+3+4)_1111_6
b'이 들어갈 우리를 결정하는 경우의 수는 2
=66660
비어 있는 우리에 넣을 2마리의 동물을 결정하는 경우의
수는 4
나머지 2개의 우리에 한 마리씩의 동물을 채우는 경우의
13 ^ 5140개
수는 2 GUIDE
1, 2, 3이 함께 일어나는 사건이므로 전체 경우의 수는 첫 두 자리 숫자가 서로 같은 경우와 다른 경우로 나누어 생각한다.
6_4_3_(4+2_4_2)=1440
1 첫 두 자리에 사용한 숫자가 서로 같은 경우
참고 첫 자리 숫자를 결정하는 경우의 수는 10
➊ 1에서 a, b, c, d 네 종류의 동물 중 두 종류를 선택한 결과에서 (a, b), 이 숫자를 a라 할 때,
(b, a)처럼 순서만 다른 것이 2개씩 중복되므로 2로 나누어야 한다. 마지막 세 자리 숫자가 모두 a로 같은 경우 1가지
➋ 3에서 ① 또는 ② 중 어느 하나가 일어나면 되므로 3의 경우의 수는
마지막 세 자리 숫자 중 a가 두 개인 경우의 수는 3_9
3_4+3_2_4_2=60
  10_(1+3_9)=280
2 첫 두 자리에 사용한 숫자가 서로 다른 경우
이 숫자를 결정하는 방법이 10_9가지

56 정답과 풀이
이 두 숫자를 a, b라 할 때, 여기까지 모두 60(=12+18+12+18)개이고, 60번째 홀수는
마지막 세 숫자가 a와 b로만 구성되는 경우의 수는 2_3 45321이므로 59번째 홀수는 45231
a, b 외에 다른 숫자가 존재하는 경우의 수는 8_3!
  10_9_(2_3+8_3!)=4860
1, 2에서 등록할 수 없는 비밀번호는 280+4860=5140(개)
16 ^④
GUIDE
맨 앞자리가 소수일 때와 소수가 아닐 때 경우의 수가 달라짐에 주의한다.
14 ^ 768개
GUIDE 1~9에서 소수는 2, 3, 5, 7이고 소수가 아닌 수는 1, 4, 6, 8, 9
다음 두 가지 경우이면 집합 X의 원소가 된다. 9 꼴은 빈자리에 소수를 선택해야 하므로 경우의 수는
➊ 10의 배수이다. (⇦ 일의 자리 수가 0)
4_3=12
➋ 십의 자리 수가 0이고 10의 배수가 아니다.
8 꼴도 마찬가지로 12
A={110, 120, 130, y, 190}, B={101, 102, 103, y, 109} 7 꼴은 다음과 같이 두 가지 경우로 생각한다.
라 하면 X=ACB 1 빈자리에 소수가 하나만 들어갈 때
이때 다음과 같은 경우에서 고른 원소를 a, b, c라 하면 소수가 들어갈 자리를 고르는 경우의 수 2
1 A에서 원소 세 개를 고를 때 ⇨ ªP£=9_8_7=504 소수를 고르는 경우의 수 3
2 A에서 a를 고르고 B에서 고른 두 원소에 b, c에 대하여 나머지 자리에 소수가 아닌 수를 선택하는 경우의 수 5
b+c=210이 되는 {b, c}는 다음과 같다.  2_3_5=30
{101, 109}, {102, 108}, {103, 107}, {104, 106} 2 빈자리 모두 소수를 선택하는 경우의 수 3_2=6
a를 고르는 경우의 수 9 즉 30+6=36
위 4개의 {b, c}에 대하여 a, b, c를 배열하는 경우의 수 3! 여기까지 12+12+36=60이므로 61번째로 큰 수는 675
 9_4_3!=216
3 B에서 a, b, c를 고를 때, a+b+c=310이 되는 {a, b, c}는
다음과 같다.
17 ^③
{101, 102, 107}, {101, 103, 106},
GUIDE
{101, 104, 105}, {102, 103, 105} 2n명이 원탁에 앉아 있을 때, 새로 온 2명이 2n명 사이 빈자리 2n곳 중
또 a+b+c=320이 되는 {a, b, c}는 다음과 같다. 두 곳을 선택하는 경우의 수는 2nP™=2n(2n-1)
{103, 108, 109}, {104, 107, 109},
f(n)=2n(2n-1)이므로
{105, 106, 109}, {105, 107, 108}
f(1)+f(2)+f(3)+f(4)+f(5)
위 각각에서 순서쌍은 3!=6(개)씩 생기므로
=2_1+4_3+6_5+8_7+10_9
경우의 수는 8_6=48
=190
따라서 순서쌍 (a, b, c)는 모두 504+216+48=768(개)

주의

➊ 원탁이므로 2n명이 앉으면 빈자리는 2n+2개가 아니라 2n개 있다.


15 ^④ ➋ n쌍의 부부가 서로 이웃하지 않도록 앉은 상태에서 생각한다.
GUIDE
홀수를 작은 것부터 더해 가다가 59개를 넘으면 거꾸로 찾아 간다.

1 홀 꼴은 ‘홀’에 3 또는 5가 들어가고
빈자리에 나머지 세 숫자가 들어가면 되므로 2_3!=12
18 ^⑤
GUIDE
마찬가지 방법으로 1 남학생 4명을 일렬로 세운다.
2 홀 꼴은 ‘홀’에 1, 3, 5 중 하나가 들어가고 빈자리 2 영희를 철수 옆에 세운다.

에 나머지 세 숫자가 들어가면 되므로 3_3!=18 3 남은 여학생 2명을 이웃하지 않도록 세운다.

3 홀 꼴은 ‘홀’에 1 또는 5가 들어가고 빈자리에 나머


1 남학생 4명을 일렬로 세우기 ⇨ 4!=24
지 세 숫자가 들어가면 되므로 2_3!=12
2 영희를 철수 옆에 세우기 ⇨ 2
4 홀 꼴은 ‘홀’에 1, 3, 5 중 하나가 들어가고 빈자리 ∨◯∨ 철수 영희 ◯∨◯∨
에 나머지 세 숫자가 들어가면 되므로 3_3!=18 여학생이 설 수 없는 자리

6. 경우의 수와 순열 57
3 남은 두 여학생이 설 자리 고르기 ⇨ 4_3=12 21 ^ 70개
1, 2, 3에서 24_2_12=576 GUIDE
f(x)=a¶x‡+a§x˜+y+a¡x+aº (ai=1 또는 -1, i=0, 1, 2, y, 7)
참고 로 놓고 f(-1)=aº-a¡+a™-a£+y-a¶=0을 이용한다.

➊ ◯는 남학생이 선 자리, ∨는 여학생이 설 수 있는 자리를 나타낸다.


㈐에서 f(-1)=aº-a¡+a™-a£+y-a¶=0이므로
➋ 남학생 4명을 일렬로 세웠을 때 생긴 5자리 중 한 자리에 영희가 섰으
므로 여학생끼리 이웃하지 않으려면 남은 두 여학생이 4자리 중 두 자 aº+a™+a¢+a§=a¡+a£+a+a¶
리에 서면 된다. 즉 (짝수차항 계수의 합)=(홀수차항 계수의 합)이다.
1 4=4, 즉 계수가 모두 1일 때, 1_1=1
2 2=2, 즉 계수 3개가 1일 때, 4_4=16
3 0=0, 즉 계수 2개가 1일 때, 6_6=36
19 ^②
4 -2=-2, 즉 계수 1개가 1일 때, 4_4=16
GUIDE
세 변을 이루는 성냥개비 개수를 각각 x, y, z (x<y<z)라 할 때, 서로 5 -4=-4, 즉 계수가 모두 -1일 때, 1_1=1
다른 삼각형의 개수는 x+y+z=15, x+y>z가 되는 자연수의 순서쌍 1~5에서 다항식 f(x)는 모두
(x, y, z) 개수와 같다. 1+16+36+16+1=70(개)

삼각형 세 변의 성냥개비 개수를 각각 x, y, z (x<y<z)라 하면 참고


3z>x+y+z=15에서 z>5 2에서 짝수차항 계수의 순서쌍 (aº, a™, a¢, a§)은
(1, 1, 1, -1), (1, 1, -1, 1), (1, -1, 1, 1), (-1, 1, 1, 1)로 4개이다.
또 (x+y)+z>z+z에서 15>2z, 즉 z<;;¡2;y ;이므로
나머지도 마찬가지로 생각한다.
z값은 5, 6, 7 중 하나이다. ※ 같은 것이 3개 있는 경우이므로
4!
로 구해도 된다.
3!
1 z=5일 때 x+y=10, x<y<5이므로
x+y+z=15의 해는 (5, 5, 5) ⇨ 1가지
2 z=6일 때 x+y=9, x<y<6이므로
22 ^ 546
x+y+z=15의 해는 (3, 6, 6), (4, 5, 6) ⇨ 2가지
GUIDE
3 z=7일 때 x+y=8, x<y<7이므로 (a-b)(b-c)(c-d)=0이면 a=b 또는 b=c 또는 c=d 이므로
x+y+z=15의 해는 전체 경우에서 a+b, b+c, c+d 인 경우를 제외한다.
(1, 7, 7), (2, 6, 7), (3, 5, 7), (4, 4, 7) ⇨ 4가지
주사위를 네 번 던질 때 모든 경우의 수는 64
1, 2, 3에서 삼각형은 모두 1+2+4=7(개)
a+b, b+c, c+d 인 경우는
a는 6가지, b는 a와 다른 5가지,
c는 b와 다른 5가지, d 는 c와 다른 5가지
20 ^ 40개 이므로 a+b, b+c, c+d 가 되는 경우의 수는 6_5_5_5
GUIDE 따라서 (a-b)(b-c)(c-d )=0이 되는 경우의 수는
x+y+z의 값이 3, 4, 5일 때로 나누어 순서쌍 개수를 구한다.
6›-6_5_5_5=6_(216-125)=546

x, y, z가 자연수이므로 3<x+y+z<5에서
1 x+y+z=3일 때
(x, y, z)는 (1, 1, 1)로 1가지 23 ^④
GUIDE
2 x+y+z=4일 때
➊ 서로 다르며, 어느 두 숫자의 차도 1보다 크다.
(x, y, z)는 (1, 1, 2), (1, 2, 1), (2, 1, 1)로 3가지 ➋ 가장 작은 수가 1인 경우, 2인 경우, 3인 경우, … 차례로 따져 본다.
3 x+y+z=5일 때
(x, y, z)는 (1, 2, 2), (2, 1, 2), (2, 2, 1), (1, 1, 3), a¡, a™, a£에서 가장 작은 수를 a, 가운데 수를 b, 가장 큰 수를 c

(1, 3, 1), (3, 1, 1)로 6가지 라 하면

1, 2, 3에서 T=2·_3‹이므로 약수는 모두 1 a=1인 경우

(9+1)(3+1)=40(개) b=3일 때 c는 5, 6, 7, 8, 9, b=4일 때 c는 6, 7, 8, 9


b=5일 때 c는 7, 8, 9, b=6일 때 c는 8, 9, b=7일 때 c=9
참고
따라서 a=1일 때 조건에 맞는 수는 모두
소인수 2는 2 에서 3개, 3 에서 6개이고 소인수 3은 3 에서 3개이다.
5+4+3+2+1=15(개)

58 정답과 풀이
마찬가지 방법으로
STEP 3 1등급 뛰어넘기 p. 82~84
2 a=2인 경우 4+3+2+1=10(개)
01 672 02 ② 03 ① 04 ④
3 a=3인 경우 3+2+1=6(개)
05 ② 06 ⑤ 07 24 08 19개
4 a=4인 경우 2+1=3(개) 09 ④ 10 126
5 a=5인 경우 b=7, c=9이므로 1개
위 각각의 경우에서 세 숫자를 나열하는 경우의 수는 3!=6 01 ^ 672
GUIDE
 6_(15+10+6+3+1)=210(개)
개수가 적은 색깔의 공부터 선택한다. 즉 파란 공 ⇨ 빨간 공 ⇨ 검은 공
⇨ 흰 공 순서로 하나씩 고르는 경우의 수를 구한다.

24 ^ 318개 파란 공을 고르는 경우 ⇨ 4가지


GUIDE 빨간 공 5개 중에서 먼저 뽑은 파란 공에 적힌 문자를 제외한 것
5의 배수가 아닌 수 중에서 5가 이웃하는 수를 제외한다. 을 고르는 경우 ⇨ 4가지
검은 공 8개 중에서 먼저 선택한 문자 2개를 제외한 것을 고르는
만의 자리에는 0이 올 수 없고, 5의 배수가 아니므로 일의 자리에
경우 ⇨ 6가지
는 0, 5가 올 수 없다. 따라서 ㈎에 맞는 다섯 자리 자연수는
흰 공 10개 중에서 먼저 선택한 문자 3개를 제외한 것을 고르는
3_4_4_4_2=384(개)
경우 ⇨ 7가지
이 중에서 5가 이웃하는 경우를 다음과 같이 생각할 수 있다.
따라서 색깔도 다르고 적힌 문자도 다른 공 4개를 선택하는 모든
1 5가 2개 이웃할 때
경우의 수는 4_4_6_7=672
5 5 꼴:3_4_2=24
5 5 꼴:2_3_2=12
1등급 NOTE
5 5 꼴:3_3_2=18
개수가 많은 색깔부터 선택하면, 개수가 적은 색의 공에 적힌 문자가 선
  24+12+18=54(개) 택되었는지 아닌지 따져야 하기 때문에 풀이가 복잡해진다.
2 5가 3개 이웃할 때 a b c d
5 5 5 꼴:3_2=6
a b c d e
5 5 5 꼴:2_2=4
a b c d e f g h
  6+4=10(개)
a b c d e f g h i j
3 5가 4개 이웃할 때
5 5 5 5 꼴:2개
따라서 구하는 자연수는 모두 384-(54+10+2)=318(개)

25 ^⑤ 02 ^②

GUIDE GUIDE
1부터 999까지의 자연수 중에서 각 자리에 3, 6, 9가 들어 있지 않은 수 주어진 삼각형을 포함할 수 있도록 x축과 y축에서 사각형의 변을 잡고,

의 개수를 뺀다. 나머지 한 점을 택한다.

1부터 999까지 모두 999개의 자연수에서 3, 6, 9가 하나도 들어 A(0, 4), B(0, 8)이라 하면

있지 않은 수는 각 자리에서 0, 1, 2, 4, 5, 7, 8 중 하나를 택한 것 1 OA’를 한 변으로 하는 사각형이 (4, 0) 또는 (8, 0)을 한 꼭

으로 생각할 수 있다. 이 조건에 맞는 수는 모두 짓점으로 하고 나머지 한 꼭짓점으로 (4, 4), (4, 8), (8, 4),

7_7_7-1=342(개) (8, 8) 중 하나를 택하는 경우 ⇨ 8가지

이므로 3, 6, 9를 적어도 하나 포함하는 수는 999-342=657(개) 2 OB’를 한 변으로 하는 사각형이 (4, 0) 또는 (8, 0)을 한 꼭짓

따라서 박수는 657번 친다. 점으로 하고 나머지 한 꼭짓점으로 (4, 4), (4, 8), (8, 4), (8, 8)
중 하나를 택하는 경우 ⇨ 8가지
참고 그중 (8, 0)이 한 꼭짓점일 때 (4, 4)를 꼭짓점으로 택하면 삼
➊ 0~9를 이용해 세 자리 수를 만들 때, 008이면 8로, 065이면 65로 생 각형이 되므로 이 경우를 제외하면 7가지이다.
각하면 된다. 1, 2에서 전체 사각형 수는 8+7=15
➋ 3, 6, 9를 제외한 숫자로 만들 수 있는 수 중에서 0이 있으므로 이 경
우를 제외한다.

6. 경우의 수와 순열 59
03 ^① 2 b a b로 세운 경우
GUIDE 2반 학생끼리 자리를 바꾸는 경우의 수 ⇨ 2
➊ 세로 방향을 기준으로 생각하지 말고, 가로 방향을 기준으로 생각한 3반 학생 세 명이 빈 네 자리를 차지하는 경우의 수
다. 즉 1층, 2층, 3층에 쌓는 블록 개수를 생각한다.
⇨ ¢P£=4_3_2=24
➋ 층의 왼쪽부터 채우면 높이가 같거나 감소하고, 오른쪽부터 쌓으면 높
  2_24=48
이가 같거나 증가한다고 생각한다.
3 b b a로 세운 경우는 1과 같으므로 경우의 수는 36
세 수 중 첫 번째 수를 1층에 쌓는 블록 개수, 가운데 수를 2층에 따라서 모든 경우의 수는 36+48+36=120
쌓는 블록 개수, 세 번째 수를 3층에 쌓는 블록 개수라 하자. 예를
들어 문제에 있는 [그림1]은 5+3+2이다.
㈎ 에 따라 10을 세 자연수의 합으로 나타내어 보자.
06 ^⑤
이때 큰 수부터 점점 작거나 같아지도록 나열하면
GUIDE
10=8+1+1=7+2+1=6+3+1=6+2+2 호텔의 모든 객실을 표로 나타내어 생각한다.
=5+4+1=5+3+2=4+4+2=4+3+3
1층의 모든 방과 각 층의 8호실을 제외한 객실에 대하여 다음과
과 같이 8가지가 있다. yy ㉠
같이 표를 만든다.
→ 방향으로 쌓는 것이 점점 낮아지는 경우라면 ← 방향으로 쌓
는 것은 점점 높아지는 경우이므로 ㉠의 8가지 각각에는 2가지 1호 2호 y 7호 B
방법이 있다. 10층
따라서 구하려는 경우의 수는 2_8=16 ⋮
3층
2층

04 ^④
A

GUIDE 투숙객이 있는 곳에는 1, 없는 곳에는 0을 쓰고, A행의 각 열에


| f(x¡)|+| f(x™)|<2에서 다음과 같이 두 가지 경우를 생각할 수 있다.
는 세로 합이 짝수가 되도록 0 또는 1을, B열의 각 행에는 가로
1 치역의 원소가 -1, 0, 1인 경우
2 치역의 원소가 0, 2 또는 0, -2이고, 정의역의 원소 중 하나만 2나
합이 짝수가 되도록 0 또는 1을 적어 넣는다.
-2에 대응하는 경우 따라서 2~10층의 1~7호실에 따라 나머지 방에 투숙객이 있는
지 결정되므로 경우의 수는 2·_7=263
1의 경우 정의역의 모든 원소가 -1, 0, 1 중 하나에 대응하면
되므로 이런 함수는 3—=243(개) 참고

2의 경우 치역의 원소가 0, 2일 때, 정의역의 원소 중 하나만 2 색선으로 둘러싸인 각 방에는 0, 1 중 어떤 값이 오더라도 상관없다.


에 대응하고 나머지는 모두 0에 대응하는 함수는 모두 5개
1등급 NOTE
치역의 원소가 0, -2일 때도 마찬가지로 5개
오른쪽 그림과 같이 각 영역에 있는 수의 합을 A, B, C,
따라서 조건에 맞는 함수는 243+5+5=253(개) D B
D라 하면 D+A와 D+B가 모두 짝수이다. 따라서 A
가 짝수이면 B도 짝수, A가 홀수이면 B도 홀수가 되 A C
어 C값이 하나로 결정된다.
05 ^②
GUIDE
1반 학생 한 명과 2반 학생 두 명을 먼저 세운 다음 3반 학생 세 명을 세
우는 방법을 생각해 본다. 07 ^ 24
GUIDE
1반 학생을 a, 2반 학생을 b, 3반 학생을 c로 나타내자. 이때 1반 ➊ 꼭짓점에 있는 수는 2번 선택된다.
과 2반 학생을 다음 세 가지 순서로 먼저 세울 수 있다. ➋ 꼭짓점 원에 들어가는 수를 각각 a, b, c라 할 때, a+b+c의 조건을
1 a b b로 세운 경우 (b와 b 사이에 반드시 c가 서야 한다.) 찾는다.

2반 학생끼리 자리를 바꾸는 경우의 수 ⇨ 2 ➌ 꼭짓점 원에 들어가는 수가 정해지면 변에 들어가는 수는 정해진다.

3반 학생 한 명이 2반 학생 사이에 서는 경우의 수 ⇨ 3
꼭짓점 원에 들어가는 수를 각각 a, b, c라 할 때, 각 변에 있는 세
◯ a ◯ b c b ◯ 에서 나머지 두 명이 자리를 선택하는 경우의
수의 합이 모두 같아야 하므로
수 ⇨ 3_2=6
1+2+3+4+5+6+a+b+c의 값이 3의 배수여야 한다.
  2_3_6=36
즉 a+b+c의 값도 3의 배수여야 한다.

60 정답과 풀이
이 중에서 가능한 집합 {a, b, c}는 {1, 2, 3}, {1, 3, 5}, {2, 4, 6}, 12+0+0, 11+1+0, 10+2+0, 10+1+1,
{4, 5, 6}이고, 이 네 집합 각각에서 세 원소를 배열하는 경우의 9+3+0, 9+2+1, 8+4+0, 8+3+1,
수는 3!=6이므로 4_6=24 8+2+2, 7+5+0, 7+4+1, 7+3+2,
6+6+0, 6+5+1, 6+4+2, 6+3+3,
주의
5+5+2, 5+4+3
{a, b, c}가 {1, 2, 6} 또는 {2, 3, 4} 또는 {3, 4, 5}이면 각 변에 있는 세
으로 18가지가 있고, 각 경우마다 a, b, c가 어떤 값을 가져도 a,
원 안에 적힌 수의 합이 같은 경우가 없다.
b의 값은 하나로 결정되므로 순서쌍 (a, b)는 18개이다.

08 ^ 19개
GUIDE
10 ^ 126
구간의 크기가 ;10!0; 이므로 점과 점 사이의 간격을 ;10!0; 과 비교해 본다. GUIDE
(A의 점수, B의 점수)로 나타내고, 다음과 같이 생각한다.

1 1 1 ➊ A가 4점일 때 다음과 같은 네 가지 경우만 있다.


먼저 - < 이 되는 양의 정수 k를 생각해 보면
k k+1 100 (4, 0), (4, 1), (4, 2), (4, 3)

100<k(k+1)이므로 k는 10 이상이다. ➋ ➊의 각각은 다음 과정으로 승부가 진행된다.


(3, 0) ⇨ (4, 0) ⇨ (5, 5) ⇨ (5, 6)
1
k가 10 이상이면 인접한 두 빨간색 점의 차이가 보다 작다. (3, 1) ⇨ (4, 1) ⇨ (5, 5) ⇨ (5, 6)
100
(3, 2) ⇨ (4, 2) ⇨ (5, 5) ⇨ (5, 6)
10 1 9 10
예를 들어 k=10이면 ;1¡0;= 에 찍히고, 은 와 (3, 3) ⇨ (4, 3) ⇨ (5, 5) ⇨ (5, 6)
100 11 100 100
➌ a가 p개이고, b가 q개일 때 이 (p+q)개를 나열하는 경우의 수는
10
사이에 찍힌다. 이렇게 생각하면 0부터 까지 10개의 구간은 (p+q)!
100 (같은 것이 있는 순열)
p!q!
모두 빨간색 점을 1개 이상 포함한다.
1 A가 1점을 따는 것을 a, B가 1점을 따는 것을 b로 나타낸다고
또 k가 9 이하일 때는 점들 사이의 거리가 보다 크므로 모두
100 하자. (※ (3, ) ⇨ (4, )와 (5, 5) ⇨ (5, 6)은 경우의 수가
서로 다른 구간에 포함되게 된다. 이때 빨간색 점을 포함하는 구 1이므로 따로 계산하지 않는다.)
간은 9개이다. 1 (3, 0) ⇨ (4, 0) ⇨ (5, 5)일 때
따라서 빨간색 점을 포함하는 구간의 개수는 19개 (3, 0)까지는 a 세 개를 나열하면 되므로 방법은 1가지이고,
(5, 5)까지는 a 한 개와 b 5개를 일렬로 나열하면 되므로 그
참고
경우의 수는 6
➊ 수직선에서 왼쪽부터 1~100까지 구간 번호를 붙이면 1~10구간까지
2 (3, 1) ⇨ (4, 1) ⇨ (5, 5)일 때
각 구간은 ;10!0; 부터 ;1¡0; 까지 빨간색 점을 적어도 하나 이상 포함한다.
a 3개, b 1개를 나열하는 경우의 수는 4이고, 그 다음에 a 1개,
➋ 11번째 구간부터는 ;9!;, ;8!;, y, ;2!;, ;1!; 이 각각 서로 다른 구간에 들어간다. b 4개를 나열하는 경우의 수는 5이므로 4_5=20
3 (3, 2) ⇨ (4, 2) ⇨ (5, 5)일 때
5!
a 3개, b 2개를 나열하는 경우의 수는 이고,
3!2!
09 ^④ 그 다음 a 1개, b 3개를 나열하는 경우의 수는 4이므로
GUIDE
5!
➊ 방정식 -x‹+12x€+ax+b=0에서 세 근의 합을 이용해 세 근의 형 _4=40
3!2!
태를 생각한다.
4 (3, 3) ⇨ (4, 3) ⇨ (5, 5)일 때
➋ a=b=0일 때 f(0)=0이므로 0은 방정식 f(x)=0의 한 근이 될 수
있다.
6!
_3=60
3!3!

방정식 -x‹+12x€+ax+b=0, 즉 x‹-12x€-ax-b=0이 따라서 6+20+40+60=126


적어도 2개의 서로 다른 정수 근을 가지고, 세 근의 합이 12이므
로 세 근 모두 정수이다.
또 x<0일 때 f(x)>0이므로 f(x)=0의 근은 모두 0 이상이다.
세 근을 a, b, c라 하면 모두 0 이상의 정수이고,
a+b+c=12인 경우를 따져 보면

6. 경우의 수와 순열 61
07 조합 04
GUIDE
^ 30

세 수가 모두 홀수일 때와 세 수 중 하나만 홀수일 때로 나누어 생각한다.


STEP 1 1등급 준비하기 p. 88 ~89

01 ① 02 ④ 03 ⑤ 04 30 홀수 1, 3, 5, 7, 9 중 3의 배수는 3, 9이고, 짝수 2, 4, 6, 8 중 3의
05 ⑤ 06 ③ 07 ③ 08 ④ 배수는 6이다.
09 ① 10 ① 1 a, b, c가 모두 홀수일 때 a_b_c가 3의 배수이려면 홀수 5
개 중 3개를 뽑는 경우에서 3의 배수가 하나도 없는 경우를 제
01 ^①
외한다.
GUIDE
 5C3-3C3=9
n!
Cr= , C+ C = C
n
r!(n-r)! n-1 r n-1 r-1 n r 2 a, b, c 중 하나만 홀수일 때 a_b_c가 3의 배수이려면 홀수
5개 중 1개를 뽑고, 짝수 4개 중 2개를 뽑는 경우에서 3의 배
C3+5C2=6C3이므로 n=6
5
수가 하나도 없는 경우를 제외한다.
7_9! 9_8!
79Cr=98Cr-1에서 =   5C1_4C2-3C1_3C2=30-9=21
r!(9-r)! (r-1)!(9-r)!
7 1, 2에서 9+21=30
간단히 하면 =1에서 r=7
r
따라서 n+r=13

05 ^⑤
GUIDE
a™<a£인 경우와 a™=a£인 경우로 나누어 생각한다.

02 ^④ 1 a¡<a™<a£<a¢일 때 1~6 중 4개를 뽑아 크기 순으로 a¡,


GUIDE a™, a£, a¢로 정하면 되므로 6C4=15
7일 중 5일을 정한다. ⇨ ¶C
2 a¡<a™=a£<a¢일 때 1~6 중 3개를 뽑아 크기 순으로 a¡,
남은 2일은 각각 두 가지씩 선택할 수 있다.
a™(=a£), a¢로 정하면 되므로 6C3=20
㈎에서 7일 중 5일을 택하는 경우의 수는 7C5=7C2=21 1, 2에서 15+20=35
㈏에서 2_2_2=8
 21_8=168
06 ^③
참고 GUIDE
‘적어도 ~’인 문제는 전체 경우에서 조건과 반대 경우를 제외한다.
n Cr을 계산할 때 r>;2N;이면 nCn-r로 바꿔 계산하는 것이 좀 더 편하다.

사탕 개수를 n이라 할 때, 15개 중에서 적어도 초콜릿 한 개를 고


르는 경우의 수는 전체 경우의 수에서 3개 모두 사탕을 고르는 경
우의 수를 뺀 것과 같다.

03 ^⑤ 즉 15C3-nC3=445에서 15C3=455이므로 nC3=10


GUIDE 이때 n(n-1)(n-2)=60=5_4_3이므로 n=5
6번째 경기까지 A, B 두 팀이 각각 3승을 해야 한다.

7번째 경기가 벌어지려면 A, B 두 팀이 각각 3승을 해야 한다.


1~6번째 경기 중 A가 승리하는 세 경기를 선택하면 나머지 세
07 ^③
GUIDE
경기는 B가 승리를 하는 것이므로 이때 경우의 수는 6C3=20
변 AB 위의 세 점과 변 CD 위의 세 점을 차례로 연결하면 된다.
7번째 경기에서 A가 승리해야 하므로 경우의 수는 1가지
따라서 20_1=20 변 AB 위의 네 점 중 세 점을 뽑고, 변 CD 위의 여섯 점 중 세 점
을 뽑아 위쪽 점끼리, 가운데 점끼리, 아래쪽 점끼리 연결하면 서
다른 풀이 로 만나지 않으므로 4C3_6C3=4_20=80
‘◯◯◯___’를 나열하는 경우의 수와 같다. 즉 같은 것이 3개
참고
6!
씩 있는 순열이므로 =20
3!3! 순서가 정해진 경우이므로 조합의 수를 이용한다.

62 정답과 풀이
08 ^④
STEP 2 1등급 굳히기 p. 90~93
GUIDE
네 개의 가로줄 중 어느 두 개를 택해 만들어지는 직사각형을 생각한다. 01 ⑤ 02 ① 03 1800 04 ⑤
05 ⑤ 06 71 07 ⑤ 08 ④
④ 09 ④ 10 38 11 114 12 ①
③ 13 ① 14 ③ 15 ② 16 ③
17 ⑤ 18 ①

① 01 ^⑤

위 그림에서 네 가로줄 ①, ②, ③, ④ 중 두 개를 택해 만들어지는 GUIDE


(8-n)+1=9-n이므로 n-1Cr+n-1Cr-1=nCr을 이용한다.
직사각형을 각각 따져 보면
1 ①, ②일 때 세로줄이 7개이므로 7C2=21 C8-n+8C9-n=9C9-n이고, 9C2n-3=9C12-2n이므로
8
2 ①, ③일 때 세로줄이 5개이므로 5C2=10 1 9-n=2n-3일 때 3n=12  n=4
3 ①, ④일 때 세로줄이 3개이므로 3C2=3 2 9-n=12-2n일 때 n=3
4 ②, ③일 때 세로줄이 5개이므로 5C2=10 따라서 4+3=7
5 ②, ④일 때 세로줄이 3개이므로 3C2=3
6 ③, ④일 때 세로줄이 3개이므로 3C2=3 주의
따라서 21+10+3+10+3+3=50 n Cr=nCn-r이므로 nCr만 생각하지 말고 nCn-r도 생각한다.

다른 풀이

C8-n+8C9-n=8Cn+8Cn-1=9Cn=9C2n-3
09
8
^①
으로 놓고 풀어도 된다.
GUIDE
‘집합 X의 임의의 두 원소 x¡, x™에 대하여 x¡<x™이면 f(x¡)<f(x™)’
라는 조건이 있으면 집합 X에서 뽑은 원소들과 집합 Y에서 뽑은 원소들
을 크기순으로 대응시키면 된다. 02 ^①
GUIDE
㈏에서 f(4)=5이므로 ㈎가 성립하려면 집합 X의 원소 1, 2, 3 ➊ 다섯 사람 중 다시 자기 의자에 앉을 두 사람을 선택한다.
이 집합 Y의 원소 1, 2, 3, 4 중 세 원소에 크기순으로 대응하면 ➋ 남은 세 사람은 처음과 다른 의자에 앉는다.
된다.  4C3=4
다섯 사람 중 자기 의자에 앉는 두 사람을 택하는 경우의 수는
또 집합 X의 원소 5는 집합 Y의 원소 6, 7, 8 중 하나에 대응하
C2=10
5
면 되므로 3C1=3
남은 세 사람이 처음과 다른 자리에 앉는 경우의 수는 2
따라서 4_3=12
따라서 10_2=20

참고

10 ^① 예를 들어 a, b 두 사람이 원래 자리에 앉는 경우일 때 남은 세 사람이 앉


GUIDE 는 자리를 수형도로 나타내 보면 2가지만 있음을 알 수 있다.
민경이와 현준이가 2명 조에 있을 때와 3명 조에 있을 때로 나누어 생각 ※ 세 사람이 서로 자리를 바꿔 앉는 경우의 수는 n=3일 때 교란수(완
한다. 전순열)이므로 2

1 민경이와 현준이가 2명 조에 있을 때 나머지 5명을 2명, 3명


으로 나누면 되므로 5C2_3C3=10
03 ^ 1800
2 민경이와 현준이가 3명 조에 있을 때 나머지 5명 중 민경이와 GUIDE
현준이 조에 들어갈 1명을 뽑고 4명은 2명, 2명으로 나누면 모든 회차에 여자 가수들이 적어도 한 명은 있어야 하므로 여자 가수들을
먼저 정한다.
1
되므로 5C1_4C2_2C2_ =15
2!
1회, 2회, 3회 공연에 먼저 여자 가수를 정한다.
1, 2에서 10+15=25
여자 가수가 2명인 공연을 선택:3C1=3
주의 여자 가수 4명 중 여자 가수가 2명인 공연에 참가할 사람을 뽑고,
1에는 같은 수가 없지만 2에는 같은 수가 두 개 있으므로 2!로 나눈다. 나머지 두 명은 다른 공연에 배열:4C2_2_1=12

7. 조합 63
 (여자 가수를 정하는 경우의 수)=3_12=36 05 ^⑤
이때 여자 가수가 2명인 공연에 함께 할 남자 가수를 다음과 같이 GUIDE
생각할 수 있다. ak는 1부터 k-1까지 자연수 중 2개를 뽑고, k+1부터 100까지 자연수
중 1개를 뽑는 것이다.
1 2명인 경우 4명 중 2명을 뽑고 남은 두 명은 다른 두 공연에
배열하면 된다. 이때 남은 2명이 공연 2개 중 하나를 정하는 ㄱ. a¢는 1~3에서 2개를 뽑고, 5~100에서 1개를 뽑는 것이므
방법 2€에서 두 명 모두 같은 공연을 정하는 경우 2가지를 빼 로 a¢=3C2_96C1 ( ◯ )
면 4C2_(2€-2)=12 9_8_90
ㄴ. a10=9C2_90C1=
2 1명인 경우 1과 마찬가지 방법으로 4C1_(2‹-2)=24 2
3 0명인 경우 1과 마찬가지 방법으로 4C0_(2›-2)=14 a90=89C2_10C1=
89_88_10
2
 (남자 가수를 정하는 경우의 수)=12+24+14=50
 a10<a90 ( ◯ )
따라서 36_50=1800
ㄷ. a£+a¢+a+y+a99는 1부터 100까지 자연수 중 4개를 뽑
참고 는 모든 경우를 구한 것과 같으므로
남자 가수를 정하는 경우 1에서 여자 가수 2명 a£+a¢+a+y+a99=100C4 ( ◯ )
a 1
이 2회 공연에 참여하고, 남자 가수 a, b, c, d
b
중 a, b가 2회 공연에 참여한다고 할 때 나머 2
c
지 남자 가수 2명이 공연에 참가하는 경우의 3
d
수는 오른쪽과 같이 생각할 수 있다.
06 ^ 71
즉 c가 선택할 수 있는 경우의 수:2
GUIDE
d 가 선택할 수 있는 경우의 수:2
➊ ㈎ 에서 주어진 조건에 맞도록 자연수 1~9 중 5개를 뽑고 ㈏ 에 주어
이때 c, d 모두 같은 공연을 선택하는 경우의 수:2
진 크기 순서를 따져 본다.
➋ 크기 순서가 정해진 경우이면 조합의 수를 이용한다.
다른 풀이 남자 가수가 참여하는 공연 정하기

세 공연 중 여자 가수가 2명인 공연을 1, 나머지를 2, 3이라 하면 ㈏ 에 따라 5자리 자연수를 pqrst로 나타내면

1 4명이 1, 2, 3 중 하나를 정하는 경우의 수 ⇨ 3› ㈎ 에 따라 t=5, 즉 r<s<5, p>q>r이므로

2 4명이 3을 제외한 1, 2 중 하나만 정하는 경우의 수 ⇨ 2› r이 1, 2, 3인 경우로 나누어 생각하면

3 4명이 2를 제외한 1, 3 중 하나만 정하는 경우의 수 ⇨ 2› 1 r=1일 때 s는 2, 3, 4 중 하나이고, p, q는 2~9에서 r, s, t

1에서 2, 3을 빼면 문제의 조건에 맞는 공연이 된다. 를 제외한 것 중 2개를 뽑는 경우의 수이므로 6C2=15

이때 4명이 모두 공연 1을 정한 경우(1›)는 두 번 빠지게 되므로  3_15=45

3›-2›-2›+1›=50 2 r=2일 때 s는 3, 4 중 하나이고, p, q는 3~9에서 r, s, t를


제외한 것 중 2개를 뽑는 경우의 수이므로 5C2=10
 2_10=20
3 r=3일 때 s=4이고, p, q는 6~9 중 2개를 뽑는 경우의 수이

04 ^⑤ 므로 4C2=6  1_6=6
GUIDE 따라서 45+20+6=71
정수 k에 대하여 자연수를 3k+1, 3k+2, 3k 꼴로 나누어 생각한다.

정수 k에 대하여
3k+1 꼴은 1, 4, 7, 10, 13, 16, 19
07 ^⑤
3k+2 꼴은 2, 5, 8, 11, 14, 17, 20 GUIDE
3k 꼴은 3, 6, 9, 12, 15, 18 1 a¡값을 정한다.

이때 세 수의 합이 3의 배수가 되는 경우는 2 a™, a¢, a, ao을 정하는 경우의 수를 구한다.


3 a£, a§, a¶을 정하는 경우의 수를 구한다.
1 세 수 모두 3k+1 꼴일 때 ⇨ 7C3=35
2 세 수 모두 3k+2 꼴일 때 ⇨ 7C3=35 ㈏에서 a¡>a™>a¢>ao, a£>a§이고,
3 세 수 모두 3k 꼴일 때 ⇨ 6C3=20 ㈐에서 a¡>a£>a¶, a™>a이므로 a1=8
4 3k+1, 3k+2, 3k 꼴이 각각 하나씩일 때 1 1~7 중 네 수를 뽑아 a™, a¢, a, ao을 정할 때 가장 큰 수를 a™
⇨ 7C1_7C1_6C1=294 라 한다. 또 남은 세 수 중 두 수를 뽑아 a¢>ao이 되도록 a¢,
따라서 35+35+20+294=384 ao을 정하고 남은 수를 a라 하면 a™>a¢>ao, a™>a가 된다.

64 정답과 풀이
  7C4_3C2=105 ② 여학생만 3명 뽑는 경우 (10가지)
2 1에서 남은 세 수 중 가장 큰 수를 a£이라 하고 남은 두 수를 ③ 안경을 쓴 학생만 3명 뽑는 경우 (4가지)
a§, a¶로 정하면 a£>a§, a£>a¶이므로 이때 경우의 수는 2 ④ 안경을 쓰지 않은 학생만 3명 뽑는 경우 (18가지)
1, 2에서 105_2=210

참고

1에서 a™>a¢>a>ao로 순서가 1가지로 정해졌다면 경우의 수는 7C4


10 ^ 38
이지만, 이것 외에도 a™>a>a¢>ao, a™>a¢>ao>a도 가능하므로 경 GUIDE
우의 수는 7C4_3=105가 된다. 정사면체의 모서리는 모두 6개이므로 3개, 4개, 5개, 6개를 칠하는 경우
로 나누어 생각한다.

정사면체를 한 꼭짓점 위에서 바라보면 B


08 ^④ 모든 모서리가 다 보이므로 오른쪽 그림
GUIDE 에서 생각한다.
벤 다이어그램에서 각 영역에 포함될 원소의 개수를 생각한다. A
먼저 모서리 3개를 칠하는 경우는 A를
C D
n(ADB)=3이므로 6개 정수 중 U 기준으로 다음과 같이 나눌 수 있다.
A B ㉣ 1 A에서 세 꼭짓점에 연결하는 경우 ⇨ 1가지
벤 다이어그램에서 ㉠에 들어갈 정수
3개를 뽑는 경우의 수는 6C3=20 ㉡ ㉠ ㉢ 2 A에서 두 꼭짓점에 연결하는 경우
남은 세 정수는 ㉡, ㉢, ㉣ 중 한 곳에 세 꼭짓점 B, C, D 중 두 개를 뽑고, 뽑은 두 꼭짓점 중 하나
들어갈 수 있고, 이때 n(A)>4, 와 남은 하나를 연결하는 경우의 수는 3C2_2=6
n(B)>4이므로 ㉡, ㉢에는 반드시 하나 이상 들어가야 한다. 3 A에서 한 꼭짓점에 연결하는 경우
1 ㉡에 2개, ㉢에 1개 들어갈 때 세 꼭짓점 B, C, D 중 하나를 뽑고, 세 모서리 BC, CD, DB
㉡에 들어갈 2개를 뽑으면 되므로 3C2=3 중 두 개를 뽑으면 되므로 이때 경우의 수는 3C1_3C2=9
2 ㉡에 1개, ㉢에 2개 들어갈 때 즉 1+6+9=16
㉢에 들어갈 2개를 뽑으면 되므로 3C2=3 모서리를 4개, 5개, 6개 칠하는 경우는 어떻게 칠하여도 모든 꼭
3 ㉡, ㉢, ㉣에 1개씩 들어갈 때 3!=6 짓점이 연결되므로 6C4+6C5+6C6=22
따라서 20(3+3+6)=240 따라서 16+22=38

참고

[그림 1]은 A와 연결할 B, C를 뽑고 (모서리 AB, AC를 색칠)


09 ^④ 이 중 B, D를 연결한 모서리를 색칠한 경우이다.
GUIDE [그림 2]는 A와 연결할 B를 뽑고, BC, BD를 색칠한 경우와 BC, CD를
표에서 분류된 학생 수를 나타내는 네 칸 중 구분 A B 색칠한 경우이다.
몇 칸을 선택해 뽑는 것을 생각한다. 남학생 2 3 B B B
여학생 2 3
A A A
1 네 칸 중 세 칸을 선택해 한 명씩 뽑을 때 C D C D C D

2 C1_3C1_3C1+2C1_3C1_2C1 [그림 1] [그림 2]


+2C1_2C1_3C1+3C1_2C1_3C1
=(2_2_3+2_3_3)_2=60
2 대각선 방향으로 두 칸을 선택할 때 (한 칸에서는 2명 선택)
(2C1_3C2+2C2_3C1)_2=18 11 ^ 114
GUIDE
따라서 60+18=78
세 점이 한 직선 위에 있는 경우는 삼각형을 만들 수 없으므로 이 경우를
제외한다.
다른 풀이

‘적어도~’ 조건에서 반대 경우를 이용해도 된다. 즉 전체 경우의 세 점이 한 직선 위에 있는 경우는 한 모서리 위에 세 점이 있을


수 10C3=120에서 다음 네 경우의 수를 뺀다. 때이므로 6가지
① 남학생만 3명 뽑는 경우 (10가지)  10C3-6=114

7. 조합 65
12 ^① 참고

GUIDE 2는 오른쪽과 같이 생각할 수 있다. 가로 방향 세로 방향


15개 점으로 정해질 수 있는 모든 직선 개수에서 세 점이 한 직선 위에 ➊ 가로 방향에서 선분 2개를 선택하는 (6C2-4C2)_(2C2+4C2)
있는 경우와 네 점이 한 직선 위에 있는 경우를 제외한다. 경우의 수 ➊ ➋ ➌ ➍

➋ A에서 선분 2개를 선택하는 경우의 수


가능한 직선 개수:15C2=105 ➌ 세로 방향 1번, 2번에서 선분 2개를 선택하는 경우의 수
점 3개가 한 직선 위에 있는 경우:11_(3C2-1)=22 ➍ 세로 방향 5번, 6번, 7번, 8번에서 선분 2개를 선택하는 경우의 수
점 4개가 한 직선 위에 있는 경우:x_(4C2-1)=5x ※ ‘적어도~’를 이용하지 않는다.

따라서 105-22-5x=58이므로 x=5

15 ^②
GUIDE
13 ^① f(1)=7, f(2)=9, f(3)=3, f(4)=1, f(5)=7, …이므로 모든 자연수

GUIDE x에 대하여 치역은 {7, 9, 3, 1}이다.


직각삼각형이 되려면 정팔각형의 외접원의 지름이 한
f(x)=7인 x값은 1, 5, 9, f(x)=9인 x값은 2, 6, 10
변이 되어야 한다. 즉 그림처럼 정팔각형의 대각선 중
가장 긴 것이 한 변이 되어야 한다. f(x)=3인 x값은 3, 7, f(x)=1인 x값은 4, 8
이때 세 함숫값 p, q, r에 대해 p+q+r<10이려면 세 함숫값이
삼각형이 한 변만 공유하는 경우를 오른 A H 1 7, 1, 1일 때 p, q, r 중 7인 것을 정하면 되므로 3C1=3
쪽 그림과 같이 생각하면 변 AB에 대 2 3, 3, 3일 때 f(a)=p, f(b)=q, f(c)=r에서 a, b, c는 서로
B G
해 가능한 꼭짓점이 4개 있으므로 다른 자연수이므로 가능하지 않다.
8_4=32  a=32 C F 3 3, 3, 1일 때 p, q, r 중 1인 것을 정하면 되므로 3C1=3
직각삼각형의 개수는 가장 긴 대각선 4 3, 1, 1일 때 p, q, r 중 3인 것을 정하면 되므로 3C1=3
D E
하나에 대해 다른 꼭짓점을 선택하는 5 1, 1, 1일 때 f(a)=p, f(b)=q, f(c)=r에서 a, b, c는 서로
경우가 6가지 있고, 이런 대각선이 4개 있으므로 다른 자연수이므로 가능하지 않다.
6_4=24  b=24 따라서 조건에 맞는 순서쌍 개수는 3+3+3=9
따라서 a+b=56
참고

만약 (a, b, c)의 순서쌍 개수를 구한다면


1 3C1_3!=18 3 2C1_3!=12 4 2C1_3!=12
조건에 맞는 순서쌍 개수는 18+12+12=42
14 ^③
GUIDE
가로 방향 선분 2개와 세로 방향 선분 2개로 직사각형이 결정된다.

가로 방향으로 위에서부터 1, 2, 5, 6번째에 있는 선분의 집합을 16 ^③


A라 하고, 3, 4번째에 있는 선분의 집합을 B라 하자. GUIDE
또 세로 방향으로 왼쪽에서부터 1, 2, 5, 6, 7, 8번째에 있는 선분 ➊ 두 명이 같은 선물을 받게 되는 경우와 모두 다른 선물을 받게 되는 경
우로 나누어 생각한다.
의 집합을 C라 하고, 3, 4번째에 있는 선분의 집합을 D라 하자.
➋ 가능한 선물 묶음은 모두 4C2=6(가지) 있다.
1 A에서 2개, C에서 2개 선택하는 경우 4C2_6C2=90
2 A에서 1개와 B에서 1개를 선택하거나 B에서 2개를 선택하 1 두 명이 같은 종류의 선물을 받는 경우
는 경우이면 C에서 1, 2번째에 있는 선분을 고르거나 5, 6, 7, 두 명을 선택하는 경우의 수 4C2=6
8번째에 있는 선분 중 2개를 골라야 하므로 이 두 명이 받을 선물을 선택하는 경우의 수 4C2=6
(6C2-4C2)_(2C2+4C2)=63  6_6=36
3 C에서 1개와 D에서 1개를 선택하거나 D에서 2개를 선택하 2 같은 종류의 선물을 받는 사람이 없는 경우
는 경우이면 A에서 1, 2번째에 있는 선분을 고르거나 5, 6번 책과 함께 줄 선물을 선택하는 경우의 수 3C2=3
째에 있는 선분을 골라야 하므로 이때 나머지 선물 조합도 결정되므로 서로 다른 4가지 선물
(8C2-6C2)_2=26 조합을 친구 4명에게 나눠주는 경우의 수는 3_4!=72
따라서 90+63+26=179 1, 2에 36+72=108

66 정답과 풀이
참고 1등급 NOTE
다음은 책과 함께 줄 선물을 선택하는 경우(3가지) 중 한 예이다. 2는 4명을 대칭점이 3개인 대진표에
배정하는 경우의 수와 같으므로
책 책 피규어 이어폰
4!
피규어 이어폰 마우스 마우스 =3
2‹

17 ^⑤
GUIDE
기약분수는 분자와 분모를 각각 소인수분해 했을 때 서로 공통인 소인수
가 없다. STEP 3 1등급 뛰어넘기 p. 94~95

12!=210_3—_5€_7_11이므로 12!의 소인수는 2, 3, 5, 7, 01 ④ 02 15 03 56 04 80


05 32 06 31 07 432
11로 5개이다.
08 ⑴ 11 ⑵ 200 ⑶ 90
이때 같은 소인수를 분자와 분모가 모두 가지면 기약분수가 아니
므로 이 소인수들은 분자나 분모 중 어느 한 쪽에만 속해야 한다. 01 ^④

따라서 분자와 분모의 곱이 12!인 기약분수는 모두 2—=32(개) GUIDE


6명 중 3명을 뽑아 만든 모든 조합 중 세 가지를 골라서 나열하는 것을
이 중에서 0과 1 사이의 수는 전체의 반이므로 조건에 맞는 기약
생각한다.
32
분수는 모두 =16(개)
2
6명 중 초대할 3명을 택하는 경우의 수는 6C3=20
참고 3일 동안 서로 다른 구성으로 초대해야 하므로
➊ 12!에 포함된 소인수 2의 개수는 20 P3=20_19_18=6840
12 12
“:¡2™:‘+“ ‘+“ ‘=6+3+1=10
2€ 2‹
➋ 오른쪽과 같이 생각하면 전체 경우의 수는
2—=32 분자 분모 02 ^ 15
➌ 예를 들어 분모와 분자의 곱이 2‹_3인 경우 GUIDE
{2, 3, 5, 7, 11} ➊ 6개의 수를 2개씩 3개 조로 나눈다.
조건과 같이 (분자 / 분모)로 나타내면
(3 / 2‹), (2‹ / 3), (2‹_3 / 1), (1 / 2‹_3)으로 4개이고, 이 중 0과 1 ➋ 각 조에서 큰 수가 집합 B의 어떤 원소에 대응해야 할지 생각한다.
사이에 있는 것은 절반인 2개이다.
1에서 6까지 6개 수를 2개씩 3개의 조로 나누는 경우의 수는
1
C2_4C2_2C2_
6 =15
18
3!
^①
GUIDE 이때 6이 포함된 조에 있는 숫자들은 3에 대응시키고, 나머지 두
3위인 선수를 한 자리에 고정시키고 이 선수가 결승에 올라가기 위한 경 조에서 더 큰 수가 포함된 조에 있는 숫자들은 2에 대응시키고,
우를 생각한다. 마지막 조의 숫자들을 1에 대응시키면 배열 방법은 유일하다.

결승
참고

(1, 6) 1 (1, 6) 1
(2, 5) 2 (2, 5) 2
3위 A (3, 4) 3 (3, 4) 3
3위인 선수가 한 자리를 잡았을 때, 남은 선수를
[그림 1] [그림 2]
1위, 2위, a, b, c, d, e라 하면
[그림 1]은 조건에 맞지만 [그림 2]는 ㈏에 어긋난다.
1 a, b, c, d, e 중 3위와 1차전에서 만날 선수를 뽑는 경우의 수
는 5C1=5
남은 네 명 중 A에 두 명을 정하는 경우의 수는 4C2=6
03 ^ 56
2 남은 두 명과 1위, 2위를 두 명씩 두 조로 정하는 경우의 수는 GUIDE
1 1 영역을 붉은색으로 칠하는 경우와 파란색으로 칠하는 경우로 나누어
4 C2_2C2_ =3
2!
색깔을 1번 바꿀 때와 2번 바꿀 때를 각각 생각한다.
따라서 5_6_3=90

7. 조합 67
각 경계선을 l¡, l™, l£, y, l¶이라 하면 480-1 480-2 480-m
479 Cm= _ _y_ 이므로
1 2 m
1 1 영역을 붉은색으로 칠할 경우
480-m
색깔을 한 번 바꾸는 것은 l¡, l™, l£, y, l¶ 중 하나를 고르는 이 짝수가 되는 m의 최솟값을 구하면 된다.
m
것과 같으므로 이때 경우의 수는 7C1=7
480-m
이 짝수이려면 m이 짝수라야 하므로
색깔을 두 번 바꾸는 것은 l¡, l™, l£, y, l¶ 중 둘을 고르는 것 m
과 같으므로 이때 경우의 수는 7C2=21, 즉 7+21=28 m=2a_b (a는 1 이상의 정수, b는 홀수)라 하자.
2 1 영역을 파란색으로 칠할 경우 그 경우의 수는 1에서 구한 480=2—_15이므로 a<4이면
것과 같으므로 28 480-m=2a(2—-a_15-b)에서 2—-a_15-b가
1, 2에서 조건에 맞는 경우의 수는 28+28=56 480-m
홀수이므로 이 홀수가 된다.
m
참고 a>5이면 480-m=2—(15-2a-5_b)에서
8에서 색칠이 끝나므로 8과 1 사이의 경계선은 색을 바꾸는 경계선이 아 a=5일 때 480-m이 짝수이다.
니다. 따라서 m의 최솟값은 a=5, b=1일 때이므로
2—_1=32이다.

04 ^ 80
GUIDE
f( f( f(x)))=x는 어떤 경우에 성립하는지 생각한다.

f( f( f(x)))=x가 되려면 aGA에 대하여 f(a)=a이거나 (…㉠)


a a
06
a a
^ 31
a, b, cGA에 대하여 b b 또는 b b 과 같이 세 원소가
GUIDE
c c c c
a¡=7이고 r=2이므로 1이 놓이는 위치에서 an>an+1이 성립한다.
서로 순환하며 대응하면 된다. (… ㉡) 따라서 1의 위치를 기준으로 경우를 따진다.
문제의 조건에서 f(a)+a인 a가 존재한다고 했으므로 함수 f 는
a¡=7, a™+1, r=2인 7자리 자연수이려면
항등함수는 아니다. 따라서 다음과 같이 나누어 생각할 수 있다.
a¡a™a£a¢aa§a¶에서 7=a¡>a™이므로 an>an+1이 되는
1 ㉠과 ㉡이 섞인 경우:정의역의 원소 3개는 ㉡처럼 자기 자신
n(n=2, 3, 4, 5, 6)은 한 개뿐이어야 한다.
이 아닌 공역의 원소에 순환하며 대응해야 한다.
즉 a™<a£<y<an, an+1<an+2<y<a¶이도록 an을 결정하면
이때 ㉡에서 패턴이 2가지이므로 경우의 수는 6C3_2=40
되므로 2, 3, 4, 5, 6 중에서 (n-1)개를 뽑아 작은 것부터 a™,
나머지 세 개는 ㉠처럼 대응하면 된다.
a£, y, an을 결정하면 a™<a£<y<an이 성립한다.
2 ㉡만 있는 경우:1~6을 3개, 3개로 분할하면 된다.
또 an+1=1이라 하고, 뽑히지 않은 수 중에서 작은 것부터 an+2,
이때 패턴이 각각 2가지이므로 경우의 수는
y, a¶을 결정하면 된다.
1
C3_3C3_
6 _2_2=40 2, 3, 4, 5, 6 중에서 (n-1)개를 뽑는 경우의 수는
2!
따라서 40+40=80 5 C1+5C2+5C3+5C4+5C5=31

1등급 NOTE 다른 풀이

f n=f n-1@f 라 할 때 f:A !# A에서 f n(x)=x가 되는 경우는 n의 a¡=7, a™+1, r=2가 되도록 7자리 자연수를 만드는 경우를 다
약수 k에 대하여 k개의 원소가 서로 순환하며 대응해야 한다. 예를 들어
음과 같이 생각할 수 있다.
f 6(x)=x가 되는 경우는 다음과 같다.
7◯1◯◯◯◯ ⇨ 5C1_4C4
1 f(a)=a (항등함수) 2 2개 원소가 서로 대응
3 3개 원소가 서로 순환하며 대응 4 6개 원소가 서로 순환하며 대응 7◯◯1◯◯◯ ⇨ 5C2_3C3
7◯◯◯1◯◯ ⇨ 5C3_2C2
7◯◯◯◯1◯ ⇨ 5C4_1C1
05 ^ 32
7◯◯◯◯◯1 ⇨ 5C5
GUIDE
n! n(n-1)y(n-r+1)  5C1+5C2+5C3+5C4+5C5=31
nCr= = 이므로
r!(n-r)! r!
n 대신 479=480-1을 대입하면
480-1 480-2 480-m
479 Cm= _ _y_ 이다.
1 2 m

68 정답과 풀이
07 ^ 432 참고

GUIDE ⑶에서 점 6개가 한 원 위에 있는 경우는 2가지


가로 줄 각각에 적어도 하나의 검은색 유리 상자가 들어가야 하므로, 한
줄에는 검은색 유리 상자가 2개 있어야 한다. 또 세로 줄 각각에 적어도
하나의 검은색 유리 상자가 들어가야 한다.

1 검은색 유리 상자가 5개 있으므로 4개의 가로 줄 중에서 검은


점 4개가 한 원 위에 있는 경우는
색 유리 상자 2개가 포함될 줄 하나를 택하는 경우가 4가지이 ➊ 직사각형을 이루는 점 네 개를 지나는 원 18개 (전체 20개에서 가로
고, 이줄에 있는 유리 상자 4개 중에서 검은색 유리 상자로 바 3, 세로 1인 것은 점 6개가 한 원 위에 있는 경우이므로 제외)
뀔 상자 2개를 택하는 경우의 수는 4C2=6이므로 ➋ 아래와 같이 직사각형이 아닌 사다리꼴의 네 꼭짓점을 지나는 경우도
6개
4_6=24(가지)
2 1의 경우 중 하나가 오른쪽과 같다고
하자. a d g j
① a, b, c에서 2개를 선택하고 j, k, b e h k
l 에서 1개를 선택하는 경우는 a, b, c f i l
c에서 2개를 선택하면 j, k, l 중 하
나가 자동으로 결정되므로 3가지
② j, k, l 에서 2개를 선택하고 a, b, c에서 1개를 선택하는 경
따라서 네 점을 지나는 원은 모두 18+6=24(개)
우는 ①과 마찬가지로 3가지
※ 직사각형의 네 꼭짓점, 등변사다리꼴의 네 꼭짓점은 원 위의 네 점이
③ a, b, c에서 1개를 선택하고 j, k, l 중에서 a, b, c에서 선
될 수 있다. (∵ 크기가 같은 두 원주각)
택된 것과 다른 가로줄에 있는 것을 선택하면 빈 가로줄의
가운데 두 개 중 하나를 선택해야 하므로
3_2_2=12(가지)
1, 2에서 24(3+3+12)=432

참고

①에서 a, l 을 먼저 선택했다면 a d g j

e, h 중 하나를 선택해야 한다. b e h k


c f i l

08 ^ (1) 11 (2) 200 (3) 90


GUIDE
⑵ 한 직선 위에서 점 3개를 뽑는 경우를 제외한다.
⑶에서 삼각형 하나가 외접원 하나를 결정함을 이용한다. 이때 점 4개가
한 원 위에 있는 경우, 점 6개가 한 원 위에 있는 경우를 제외한다.

⑴ 가로 방향 3개, 세로 방향 4개, 대각선 방향 4개


따라서 3+4+4=11
⑵ 12개 점 중에서 3개를 선택하는 경우 중 한 직선
위에 세 점이 존재하는 경우를 제외하면 된다.
12 C3-3_4C3-4_3C3-4_3C3
=220-12-4-4=200
⑶ 삼각형 하나에 외접원 하나가 결정되고,
점 6개가 한 원 위에 있는 경우가 2가지,
점 4개가 한 원 위에 있는 경우가 24가지이므로
200-(6C3-1)_2-(4C3-1)_24
=200-38-72=90

7. 조합 69
memo
memo
memo

You might also like